Feat: DM pour les TST3
continuous-integration/drone/push Build is passing Details

This commit is contained in:
Bertrand Benjamin 2021-02-04 11:19:45 +01:00
parent ef6c26a51c
commit 3fa915e643
43 changed files with 8346 additions and 0 deletions

View File

@ -0,0 +1,203 @@
\documentclass[a5paper,10pt]{article}
\usepackage{myXsim}
\usepackage{tasks}
% Title Page
\title{DM2 \hfill ASAIDI Sophian}
\tribe{TST}
\date{\hfillÀ render pour le Mercredi 24 février}
\xsimsetup{
solution/print = false
}
\begin{document}
\maketitle
\begin{exercise}[subtitle={Loi binomiale}]
Trois personnes s'apprêtent à passer le portique de sécurité. On suppose que pour chaque personne la probabilité que le portique sonne est égale à $0.19$.
Soit $X$ la variable aléatoire donnant le nombre de personnes faisant sonner le portique, parmi les 3 personnes de ce groupe.
\begin{enumerate}
\item Tracer l'arbre représentant le situation.
\item Justifier que $X$ suit une loi binomiale dont on précisera les paramètres.
\item Quelle est la probabilité qu'une seule personne fasse sonner le portique?
\item Calculer puis interpréter les probabilités suivantes
\[
P(X = 0) \qquad \qquad P(X \geq 2)
\]
\item Calculer l'espérance de $X$ et interpréter le résultat.
\end{enumerate}
\end{exercise}
\begin{solution}
\begin{enumerate}
\item
\begin{tikzpicture}[sloped]
\node {.}
child {node {$0$}
child {node {$0$}
child {node {$0$}
edge from parent
node[above] {0.81}
}
child {node {$1$}
edge from parent
node[above] {0.19}
}
edge from parent
node[above] {0.81}
}
child[missing] {}
child {node {$1$}
child {node {$0$}
edge from parent
node[above] {0.81}
}
child {node {$1$}
edge from parent
node[above] {0.19}
}
edge from parent
node[above] {0.81}
}
edge from parent
node[above] {0.81}
}
child[missing] {}
child[missing] {}
child[missing] {}
child { node {$1$}
child {node {$0$}
child {node {$0$}
edge from parent
node[above] {0.81}
}
child {node {$1$}
edge from parent
node[above] {0.19}
}
edge from parent
node[above] {0.81}
}
child[missing] {}
child {node {$1$}
child {node {$0$}
edge from parent
node[above] {0.81}
}
child {node {$1$}
edge from parent
node[above] {0.19}
}
edge from parent
node[above] {0.81}
}
edge from parent
node[above] {0.19}
} ;
\end{tikzpicture}
\item Chaque personne a 2 possibilités (1: fait sonner ou 2: ne fait pas sonner) et l'on fait passer 3 personnes ce qui correspond à une répétition identique et aléatoire. On peut donc modéliser la situation par une loi binomiale.
\[
X \sim \mathcal{B}(3; 0.76)
\]
\item Probabilité qu'une seule personne fasse sonner le portique. On voit qu'il y a 3 branches qui correspondent à cette situation dont
\[
P(X = 1) = 3 \times 0.19^1 \times 0.81^2 \approx 0.374
\]
\item
\[
P(X = 0) = 0.81^3 \approx 0.531
\]
\[
P(X \geq 2) = P(X = 2) + P(X = 3) = 3 \times 0.19^2 \times 0.81^1 + 0.19^3 \approx 0.095
\]
\item Il faut d'abord tracer le tableau résumant la loi de probabilité:
\begin{center}
\begin{tabular}{|c|*{4}{c|}}
\hline
Valeur & 0 & 1 & 2 & 3 \\
\hline
Probabilité & $0.531$ & $0.374$ & $0.088$ &$0.007$ \\
\hline
\end{tabular}
\end{center}
On peut alors calculer l'espérance
\[
E[X] = 0 \times 0.531 + 1 \times 0.374 + 2 \times 0.088 + 3 \times 0.007 = 0.57
\]
On peut donc estimer qu'il y aura en moyenne $0.57$ personnes qui feront sonner le portique sur les 3 personnes.
\end{enumerate}
\end{solution}
\begin{exercise}[subtitle={Équation puissance}]
Résoudre les équations et inéquations suivantes
\begin{multicols}{2}
\begin{enumerate}
\item $10^x = 7$
\item $3^x = 35$
\item $0.8^x \leq 2$
\item $4 \times 0.06^x = 49$
\end{enumerate}
\end{multicols}
\end{exercise}
\begin{solution}
Les solutions ci-dessous ne sont pas justifiée car l'ordinateur ne sait pas faire. Par contre, vous vous devez savoir justifier vos réponses!
\begin{enumerate}
\item $x = \log(7)$
\item $x = \frac{\log(35)}{\log(3)}$
\item Il faut faire attention quand on divise par un log car ce dernier peut être négatif ce qui est le cas ici. Il faut donc pense à changer le sens de l'inégalité.
$x \geq \frac{\log(2)}{\log(0.8)}$
\item Il faut penser à faire la division à par $4$ avant d'utiliser le log car sinon, on ne peut pas utiliser la formule $\log(a^n) = n\times \log(a)$.
$x = \frac{\log(12.25)}{\log(0.06)}$
\end{enumerate}
\end{solution}
\begin{exercise}[subtitle={Étude de fonctions}]
Soit $f(x) = 5x^3 - 202.5x^2 - 2970x + 24$ une fonction définie sur $\R$.
\begin{enumerate}
\item Calculer $f'(x)$ la dérivée de $f(x)$.
\item Calculer $f'(33)$ et $f'(-6)$.
\item En déduire une forme factorisée de $f'(x)$.
\item Étudier le signe de $f'(x)$ et en déduire les variations de $f(x)$.
\item Est-ce que la fonction $f(x)$ admet un maximum ou un minimum? Si oui, calculer sa valeur.
\end{enumerate}
\end{exercise}
\begin{solution}
\begin{enumerate}
\item Dérivée de $f(x)$: $f'(x) = 15x^2 - 405x - 2970$
\item
\begin{align*}
f'(33) &= 15 \times 33^{2} - 405 \times 33 - 2970\\&= 15 \times 1089 - 13365 - 2970\\&= 16335 - 16335\\&= 0
\end{align*}
\begin{align*}
f'(-6) &= 15 \times - 6^{2} - 405(- 6) - 2970\\&= 15 \times 36 + 2430 - 2970\\&= 540 - 540\\&= 0
\end{align*}
Donc $x = 33$ et $x=-6$ sont des racines de $f'(x) = 15x^2 - 405x - 2970$.
\item On en déduit la forme factorisée suivante
\[
f'(x) = 15 (x - 33)(x--6)
\]
\item Pas de correction disponible
\item À causes des branches extérieurs, la fonction $f(x)$ n'a pas de maximum ou de minimum.
\end{enumerate}
\end{solution}
%\printsolutionstype{exercise}
\end{document}
%%% Local Variables:
%%% mode: latex
%%% TeX-master: "master"
%%% End:

View File

@ -0,0 +1,203 @@
\documentclass[a5paper,10pt]{article}
\usepackage{myXsim}
\usepackage{tasks}
% Title Page
\title{DM2 \hfill BELARBI Samira}
\tribe{TST}
\date{\hfillÀ render pour le Mercredi 24 février}
\xsimsetup{
solution/print = false
}
\begin{document}
\maketitle
\begin{exercise}[subtitle={Loi binomiale}]
Trois personnes s'apprêtent à passer le portique de sécurité. On suppose que pour chaque personne la probabilité que le portique sonne est égale à $0.58$.
Soit $X$ la variable aléatoire donnant le nombre de personnes faisant sonner le portique, parmi les 3 personnes de ce groupe.
\begin{enumerate}
\item Tracer l'arbre représentant le situation.
\item Justifier que $X$ suit une loi binomiale dont on précisera les paramètres.
\item Quelle est la probabilité qu'une seule personne fasse sonner le portique?
\item Calculer puis interpréter les probabilités suivantes
\[
P(X = 0) \qquad \qquad P(X \geq 2)
\]
\item Calculer l'espérance de $X$ et interpréter le résultat.
\end{enumerate}
\end{exercise}
\begin{solution}
\begin{enumerate}
\item
\begin{tikzpicture}[sloped]
\node {.}
child {node {$0$}
child {node {$0$}
child {node {$0$}
edge from parent
node[above] {0.42}
}
child {node {$1$}
edge from parent
node[above] {0.58}
}
edge from parent
node[above] {0.42}
}
child[missing] {}
child {node {$1$}
child {node {$0$}
edge from parent
node[above] {0.42}
}
child {node {$1$}
edge from parent
node[above] {0.58}
}
edge from parent
node[above] {0.42}
}
edge from parent
node[above] {0.42}
}
child[missing] {}
child[missing] {}
child[missing] {}
child { node {$1$}
child {node {$0$}
child {node {$0$}
edge from parent
node[above] {0.42}
}
child {node {$1$}
edge from parent
node[above] {0.58}
}
edge from parent
node[above] {0.42}
}
child[missing] {}
child {node {$1$}
child {node {$0$}
edge from parent
node[above] {0.42}
}
child {node {$1$}
edge from parent
node[above] {0.58}
}
edge from parent
node[above] {0.42}
}
edge from parent
node[above] {0.58}
} ;
\end{tikzpicture}
\item Chaque personne a 2 possibilités (1: fait sonner ou 2: ne fait pas sonner) et l'on fait passer 3 personnes ce qui correspond à une répétition identique et aléatoire. On peut donc modéliser la situation par une loi binomiale.
\[
X \sim \mathcal{B}(3; 0.76)
\]
\item Probabilité qu'une seule personne fasse sonner le portique. On voit qu'il y a 3 branches qui correspondent à cette situation dont
\[
P(X = 1) = 3 \times 0.58^1 \times 0.42^2 \approx 0.307
\]
\item
\[
P(X = 0) = 0.42^3 \approx 0.074
\]
\[
P(X \geq 2) = P(X = 2) + P(X = 3) = 3 \times 0.58^2 \times 0.42^1 + 0.58^3 \approx 0.619
\]
\item Il faut d'abord tracer le tableau résumant la loi de probabilité:
\begin{center}
\begin{tabular}{|c|*{4}{c|}}
\hline
Valeur & 0 & 1 & 2 & 3 \\
\hline
Probabilité & $0.074$ & $0.307$ & $0.424$ &$0.195$ \\
\hline
\end{tabular}
\end{center}
On peut alors calculer l'espérance
\[
E[X] = 0 \times 0.074 + 1 \times 0.307 + 2 \times 0.424 + 3 \times 0.195 = 1.74
\]
On peut donc estimer qu'il y aura en moyenne $1.74$ personnes qui feront sonner le portique sur les 3 personnes.
\end{enumerate}
\end{solution}
\begin{exercise}[subtitle={Équation puissance}]
Résoudre les équations et inéquations suivantes
\begin{multicols}{2}
\begin{enumerate}
\item $10^x = 6$
\item $10^x = 31$
\item $0.32^x \leq 15$
\item $5 \times 0.06^x = 9$
\end{enumerate}
\end{multicols}
\end{exercise}
\begin{solution}
Les solutions ci-dessous ne sont pas justifiée car l'ordinateur ne sait pas faire. Par contre, vous vous devez savoir justifier vos réponses!
\begin{enumerate}
\item $x = \log(6)$
\item $x = \frac{\log(31)}{\log(10)}$
\item Il faut faire attention quand on divise par un log car ce dernier peut être négatif ce qui est le cas ici. Il faut donc pense à changer le sens de l'inégalité.
$x \geq \frac{\log(15)}{\log(0.32)}$
\item Il faut penser à faire la division à par $5$ avant d'utiliser le log car sinon, on ne peut pas utiliser la formule $\log(a^n) = n\times \log(a)$.
$x = \frac{\log(1.8)}{\log(0.06)}$
\end{enumerate}
\end{solution}
\begin{exercise}[subtitle={Étude de fonctions}]
Soit $f(x) = - 4x^3 + 126x^2 + 2784x - 17$ une fonction définie sur $\R$.
\begin{enumerate}
\item Calculer $f'(x)$ la dérivée de $f(x)$.
\item Calculer $f'(29)$ et $f'(-8)$.
\item En déduire une forme factorisée de $f'(x)$.
\item Étudier le signe de $f'(x)$ et en déduire les variations de $f(x)$.
\item Est-ce que la fonction $f(x)$ admet un maximum ou un minimum? Si oui, calculer sa valeur.
\end{enumerate}
\end{exercise}
\begin{solution}
\begin{enumerate}
\item Dérivée de $f(x)$: $f'(x) = - 12x^2 + 252x + 2784$
\item
\begin{align*}
f'(29) &= - 12 \times 29^{2} + 252 \times 29 + 2784\\&= - 12 \times 841 + 7308 + 2784\\&= - 10092 + 10092\\&= 0
\end{align*}
\begin{align*}
f'(-8) &= - 12 \times - 8^{2} + 252(- 8) + 2784\\&= - 12 \times 64 - 2016 + 2784\\&= - 768 + 768\\&= 0
\end{align*}
Donc $x = 29$ et $x=-8$ sont des racines de $f'(x) = - 12x^2 + 252x + 2784$.
\item On en déduit la forme factorisée suivante
\[
f'(x) = -12 (x - 29)(x--8)
\]
\item Pas de correction disponible
\item À causes des branches extérieurs, la fonction $f(x)$ n'a pas de maximum ou de minimum.
\end{enumerate}
\end{solution}
%\printsolutionstype{exercise}
\end{document}
%%% Local Variables:
%%% mode: latex
%%% TeX-master: "master"
%%% End:

View File

@ -0,0 +1,203 @@
\documentclass[a5paper,10pt]{article}
\usepackage{myXsim}
\usepackage{tasks}
% Title Page
\title{DM2 \hfill BERTAN Ufuk}
\tribe{TST}
\date{\hfillÀ render pour le Mercredi 24 février}
\xsimsetup{
solution/print = false
}
\begin{document}
\maketitle
\begin{exercise}[subtitle={Loi binomiale}]
Trois personnes s'apprêtent à passer le portique de sécurité. On suppose que pour chaque personne la probabilité que le portique sonne est égale à $0.04$.
Soit $X$ la variable aléatoire donnant le nombre de personnes faisant sonner le portique, parmi les 3 personnes de ce groupe.
\begin{enumerate}
\item Tracer l'arbre représentant le situation.
\item Justifier que $X$ suit une loi binomiale dont on précisera les paramètres.
\item Quelle est la probabilité qu'une seule personne fasse sonner le portique?
\item Calculer puis interpréter les probabilités suivantes
\[
P(X = 0) \qquad \qquad P(X \geq 2)
\]
\item Calculer l'espérance de $X$ et interpréter le résultat.
\end{enumerate}
\end{exercise}
\begin{solution}
\begin{enumerate}
\item
\begin{tikzpicture}[sloped]
\node {.}
child {node {$0$}
child {node {$0$}
child {node {$0$}
edge from parent
node[above] {0.96}
}
child {node {$1$}
edge from parent
node[above] {0.04}
}
edge from parent
node[above] {0.96}
}
child[missing] {}
child {node {$1$}
child {node {$0$}
edge from parent
node[above] {0.96}
}
child {node {$1$}
edge from parent
node[above] {0.04}
}
edge from parent
node[above] {0.96}
}
edge from parent
node[above] {0.96}
}
child[missing] {}
child[missing] {}
child[missing] {}
child { node {$1$}
child {node {$0$}
child {node {$0$}
edge from parent
node[above] {0.96}
}
child {node {$1$}
edge from parent
node[above] {0.04}
}
edge from parent
node[above] {0.96}
}
child[missing] {}
child {node {$1$}
child {node {$0$}
edge from parent
node[above] {0.96}
}
child {node {$1$}
edge from parent
node[above] {0.04}
}
edge from parent
node[above] {0.96}
}
edge from parent
node[above] {0.04}
} ;
\end{tikzpicture}
\item Chaque personne a 2 possibilités (1: fait sonner ou 2: ne fait pas sonner) et l'on fait passer 3 personnes ce qui correspond à une répétition identique et aléatoire. On peut donc modéliser la situation par une loi binomiale.
\[
X \sim \mathcal{B}(3; 0.76)
\]
\item Probabilité qu'une seule personne fasse sonner le portique. On voit qu'il y a 3 branches qui correspondent à cette situation dont
\[
P(X = 1) = 3 \times 0.04^1 \times 0.96^2 \approx 0.111
\]
\item
\[
P(X = 0) = 0.96^3 \approx 0.885
\]
\[
P(X \geq 2) = P(X = 2) + P(X = 3) = 3 \times 0.04^2 \times 0.96^1 + 0.04^3 \approx 0.005
\]
\item Il faut d'abord tracer le tableau résumant la loi de probabilité:
\begin{center}
\begin{tabular}{|c|*{4}{c|}}
\hline
Valeur & 0 & 1 & 2 & 3 \\
\hline
Probabilité & $0.885$ & $0.111$ & $0.005$ &$0.0$ \\
\hline
\end{tabular}
\end{center}
On peut alors calculer l'espérance
\[
E[X] = 0 \times 0.885 + 1 \times 0.111 + 2 \times 0.005 + 3 \times 0.0 = 0.12
\]
On peut donc estimer qu'il y aura en moyenne $0.12$ personnes qui feront sonner le portique sur les 3 personnes.
\end{enumerate}
\end{solution}
\begin{exercise}[subtitle={Équation puissance}]
Résoudre les équations et inéquations suivantes
\begin{multicols}{2}
\begin{enumerate}
\item $10^x = 14$
\item $11^x = 35$
\item $0.05^x \leq 24$
\item $4 \times 0.92^x = 47$
\end{enumerate}
\end{multicols}
\end{exercise}
\begin{solution}
Les solutions ci-dessous ne sont pas justifiée car l'ordinateur ne sait pas faire. Par contre, vous vous devez savoir justifier vos réponses!
\begin{enumerate}
\item $x = \log(14)$
\item $x = \frac{\log(35)}{\log(11)}$
\item Il faut faire attention quand on divise par un log car ce dernier peut être négatif ce qui est le cas ici. Il faut donc pense à changer le sens de l'inégalité.
$x \geq \frac{\log(24)}{\log(0.05)}$
\item Il faut penser à faire la division à par $4$ avant d'utiliser le log car sinon, on ne peut pas utiliser la formule $\log(a^n) = n\times \log(a)$.
$x = \frac{\log(11.75)}{\log(0.92)}$
\end{enumerate}
\end{solution}
\begin{exercise}[subtitle={Étude de fonctions}]
Soit $f(x) = 4x^3 - 306x^2 + 4128x + 39$ une fonction définie sur $\R$.
\begin{enumerate}
\item Calculer $f'(x)$ la dérivée de $f(x)$.
\item Calculer $f'(43)$ et $f'(8)$.
\item En déduire une forme factorisée de $f'(x)$.
\item Étudier le signe de $f'(x)$ et en déduire les variations de $f(x)$.
\item Est-ce que la fonction $f(x)$ admet un maximum ou un minimum? Si oui, calculer sa valeur.
\end{enumerate}
\end{exercise}
\begin{solution}
\begin{enumerate}
\item Dérivée de $f(x)$: $f'(x) = 12x^2 - 612x + 4128$
\item
\begin{align*}
f'(43) &= 12 \times 43^{2} - 612 \times 43 + 4128\\&= 12 \times 1849 - 26316 + 4128\\&= 22188 - 22188\\&= 0
\end{align*}
\begin{align*}
f'(8) &= 12 \times 8^{2} - 612 \times 8 + 4128\\&= 12 \times 64 - 4896 + 4128\\&= 768 - 768\\&= 0
\end{align*}
Donc $x = 43$ et $x=8$ sont des racines de $f'(x) = 12x^2 - 612x + 4128$.
\item On en déduit la forme factorisée suivante
\[
f'(x) = 12 (x - 43)(x-8)
\]
\item Pas de correction disponible
\item À causes des branches extérieurs, la fonction $f(x)$ n'a pas de maximum ou de minimum.
\end{enumerate}
\end{solution}
%\printsolutionstype{exercise}
\end{document}
%%% Local Variables:
%%% mode: latex
%%% TeX-master: "master"
%%% End:

View File

@ -0,0 +1,203 @@
\documentclass[a5paper,10pt]{article}
\usepackage{myXsim}
\usepackage{tasks}
% Title Page
\title{DM2 \hfill BOUALIA Bilel}
\tribe{TST}
\date{\hfillÀ render pour le Mercredi 24 février}
\xsimsetup{
solution/print = false
}
\begin{document}
\maketitle
\begin{exercise}[subtitle={Loi binomiale}]
Trois personnes s'apprêtent à passer le portique de sécurité. On suppose que pour chaque personne la probabilité que le portique sonne est égale à $0.68$.
Soit $X$ la variable aléatoire donnant le nombre de personnes faisant sonner le portique, parmi les 3 personnes de ce groupe.
\begin{enumerate}
\item Tracer l'arbre représentant le situation.
\item Justifier que $X$ suit une loi binomiale dont on précisera les paramètres.
\item Quelle est la probabilité qu'une seule personne fasse sonner le portique?
\item Calculer puis interpréter les probabilités suivantes
\[
P(X = 0) \qquad \qquad P(X \geq 2)
\]
\item Calculer l'espérance de $X$ et interpréter le résultat.
\end{enumerate}
\end{exercise}
\begin{solution}
\begin{enumerate}
\item
\begin{tikzpicture}[sloped]
\node {.}
child {node {$0$}
child {node {$0$}
child {node {$0$}
edge from parent
node[above] {0.32}
}
child {node {$1$}
edge from parent
node[above] {0.68}
}
edge from parent
node[above] {0.32}
}
child[missing] {}
child {node {$1$}
child {node {$0$}
edge from parent
node[above] {0.32}
}
child {node {$1$}
edge from parent
node[above] {0.68}
}
edge from parent
node[above] {0.32}
}
edge from parent
node[above] {0.32}
}
child[missing] {}
child[missing] {}
child[missing] {}
child { node {$1$}
child {node {$0$}
child {node {$0$}
edge from parent
node[above] {0.32}
}
child {node {$1$}
edge from parent
node[above] {0.68}
}
edge from parent
node[above] {0.32}
}
child[missing] {}
child {node {$1$}
child {node {$0$}
edge from parent
node[above] {0.32}
}
child {node {$1$}
edge from parent
node[above] {0.68}
}
edge from parent
node[above] {0.32}
}
edge from parent
node[above] {0.68}
} ;
\end{tikzpicture}
\item Chaque personne a 2 possibilités (1: fait sonner ou 2: ne fait pas sonner) et l'on fait passer 3 personnes ce qui correspond à une répétition identique et aléatoire. On peut donc modéliser la situation par une loi binomiale.
\[
X \sim \mathcal{B}(3; 0.76)
\]
\item Probabilité qu'une seule personne fasse sonner le portique. On voit qu'il y a 3 branches qui correspondent à cette situation dont
\[
P(X = 1) = 3 \times 0.68^1 \times 0.32^2 \approx 0.209
\]
\item
\[
P(X = 0) = 0.32^3 \approx 0.033
\]
\[
P(X \geq 2) = P(X = 2) + P(X = 3) = 3 \times 0.68^2 \times 0.32^1 + 0.68^3 \approx 0.758
\]
\item Il faut d'abord tracer le tableau résumant la loi de probabilité:
\begin{center}
\begin{tabular}{|c|*{4}{c|}}
\hline
Valeur & 0 & 1 & 2 & 3 \\
\hline
Probabilité & $0.033$ & $0.209$ & $0.444$ &$0.314$ \\
\hline
\end{tabular}
\end{center}
On peut alors calculer l'espérance
\[
E[X] = 0 \times 0.033 + 1 \times 0.209 + 2 \times 0.444 + 3 \times 0.314 = 2.04
\]
On peut donc estimer qu'il y aura en moyenne $2.04$ personnes qui feront sonner le portique sur les 3 personnes.
\end{enumerate}
\end{solution}
\begin{exercise}[subtitle={Équation puissance}]
Résoudre les équations et inéquations suivantes
\begin{multicols}{2}
\begin{enumerate}
\item $10^x = 45$
\item $10^x = 5$
\item $0.69^x \leq 42$
\item $4 \times 0.04^x = 21$
\end{enumerate}
\end{multicols}
\end{exercise}
\begin{solution}
Les solutions ci-dessous ne sont pas justifiée car l'ordinateur ne sait pas faire. Par contre, vous vous devez savoir justifier vos réponses!
\begin{enumerate}
\item $x = \log(45)$
\item $x = \frac{\log(5)}{\log(10)}$
\item Il faut faire attention quand on divise par un log car ce dernier peut être négatif ce qui est le cas ici. Il faut donc pense à changer le sens de l'inégalité.
$x \geq \frac{\log(42)}{\log(0.69)}$
\item Il faut penser à faire la division à par $4$ avant d'utiliser le log car sinon, on ne peut pas utiliser la formule $\log(a^n) = n\times \log(a)$.
$x = \frac{\log(5.25)}{\log(0.04)}$
\end{enumerate}
\end{solution}
\begin{exercise}[subtitle={Étude de fonctions}]
Soit $f(x) = 10x^3 - 645x^2 - 7200x - 16$ une fonction définie sur $\R$.
\begin{enumerate}
\item Calculer $f'(x)$ la dérivée de $f(x)$.
\item Calculer $f'(48)$ et $f'(-5)$.
\item En déduire une forme factorisée de $f'(x)$.
\item Étudier le signe de $f'(x)$ et en déduire les variations de $f(x)$.
\item Est-ce que la fonction $f(x)$ admet un maximum ou un minimum? Si oui, calculer sa valeur.
\end{enumerate}
\end{exercise}
\begin{solution}
\begin{enumerate}
\item Dérivée de $f(x)$: $f'(x) = 30x^2 - 1290x - 7200$
\item
\begin{align*}
f'(48) &= 30 \times 48^{2} - 1290 \times 48 - 7200\\&= 30 \times 2304 - 61920 - 7200\\&= 69120 - 69120\\&= 0
\end{align*}
\begin{align*}
f'(-5) &= 30 \times - 5^{2} - 1290(- 5) - 7200\\&= 30 \times 25 + 6450 - 7200\\&= 750 - 750\\&= 0
\end{align*}
Donc $x = 48$ et $x=-5$ sont des racines de $f'(x) = 30x^2 - 1290x - 7200$.
\item On en déduit la forme factorisée suivante
\[
f'(x) = 30 (x - 48)(x--5)
\]
\item Pas de correction disponible
\item À causes des branches extérieurs, la fonction $f(x)$ n'a pas de maximum ou de minimum.
\end{enumerate}
\end{solution}
%\printsolutionstype{exercise}
\end{document}
%%% Local Variables:
%%% mode: latex
%%% TeX-master: "master"
%%% End:

View File

@ -0,0 +1,203 @@
\documentclass[a5paper,10pt]{article}
\usepackage{myXsim}
\usepackage{tasks}
% Title Page
\title{DM2 \hfill BOUCHOUX Kevin}
\tribe{TST}
\date{\hfillÀ render pour le Mercredi 24 février}
\xsimsetup{
solution/print = false
}
\begin{document}
\maketitle
\begin{exercise}[subtitle={Loi binomiale}]
Trois personnes s'apprêtent à passer le portique de sécurité. On suppose que pour chaque personne la probabilité que le portique sonne est égale à $0.7$.
Soit $X$ la variable aléatoire donnant le nombre de personnes faisant sonner le portique, parmi les 3 personnes de ce groupe.
\begin{enumerate}
\item Tracer l'arbre représentant le situation.
\item Justifier que $X$ suit une loi binomiale dont on précisera les paramètres.
\item Quelle est la probabilité qu'une seule personne fasse sonner le portique?
\item Calculer puis interpréter les probabilités suivantes
\[
P(X = 0) \qquad \qquad P(X \geq 2)
\]
\item Calculer l'espérance de $X$ et interpréter le résultat.
\end{enumerate}
\end{exercise}
\begin{solution}
\begin{enumerate}
\item
\begin{tikzpicture}[sloped]
\node {.}
child {node {$0$}
child {node {$0$}
child {node {$0$}
edge from parent
node[above] {0.3}
}
child {node {$1$}
edge from parent
node[above] {0.7}
}
edge from parent
node[above] {0.3}
}
child[missing] {}
child {node {$1$}
child {node {$0$}
edge from parent
node[above] {0.3}
}
child {node {$1$}
edge from parent
node[above] {0.7}
}
edge from parent
node[above] {0.3}
}
edge from parent
node[above] {0.3}
}
child[missing] {}
child[missing] {}
child[missing] {}
child { node {$1$}
child {node {$0$}
child {node {$0$}
edge from parent
node[above] {0.3}
}
child {node {$1$}
edge from parent
node[above] {0.7}
}
edge from parent
node[above] {0.3}
}
child[missing] {}
child {node {$1$}
child {node {$0$}
edge from parent
node[above] {0.3}
}
child {node {$1$}
edge from parent
node[above] {0.7}
}
edge from parent
node[above] {0.3}
}
edge from parent
node[above] {0.7}
} ;
\end{tikzpicture}
\item Chaque personne a 2 possibilités (1: fait sonner ou 2: ne fait pas sonner) et l'on fait passer 3 personnes ce qui correspond à une répétition identique et aléatoire. On peut donc modéliser la situation par une loi binomiale.
\[
X \sim \mathcal{B}(3; 0.76)
\]
\item Probabilité qu'une seule personne fasse sonner le portique. On voit qu'il y a 3 branches qui correspondent à cette situation dont
\[
P(X = 1) = 3 \times 0.7^1 \times 0.3^2 \approx 0.189
\]
\item
\[
P(X = 0) = 0.3^3 \approx 0.027
\]
\[
P(X \geq 2) = P(X = 2) + P(X = 3) = 3 \times 0.7^2 \times 0.3^1 + 0.7^3 \approx 0.784
\]
\item Il faut d'abord tracer le tableau résumant la loi de probabilité:
\begin{center}
\begin{tabular}{|c|*{4}{c|}}
\hline
Valeur & 0 & 1 & 2 & 3 \\
\hline
Probabilité & $0.027$ & $0.189$ & $0.441$ &$0.343$ \\
\hline
\end{tabular}
\end{center}
On peut alors calculer l'espérance
\[
E[X] = 0 \times 0.027 + 1 \times 0.189 + 2 \times 0.441 + 3 \times 0.343 = 2.1
\]
On peut donc estimer qu'il y aura en moyenne $2.1$ personnes qui feront sonner le portique sur les 3 personnes.
\end{enumerate}
\end{solution}
\begin{exercise}[subtitle={Équation puissance}]
Résoudre les équations et inéquations suivantes
\begin{multicols}{2}
\begin{enumerate}
\item $10^x = 4$
\item $7^x = 14$
\item $0.44^x \leq 29$
\item $6 \times 0.27^x = 10$
\end{enumerate}
\end{multicols}
\end{exercise}
\begin{solution}
Les solutions ci-dessous ne sont pas justifiée car l'ordinateur ne sait pas faire. Par contre, vous vous devez savoir justifier vos réponses!
\begin{enumerate}
\item $x = \log(4)$
\item $x = \frac{\log(14)}{\log(7)}$
\item Il faut faire attention quand on divise par un log car ce dernier peut être négatif ce qui est le cas ici. Il faut donc pense à changer le sens de l'inégalité.
$x \geq \frac{\log(29)}{\log(0.44)}$
\item Il faut penser à faire la division à par $6$ avant d'utiliser le log car sinon, on ne peut pas utiliser la formule $\log(a^n) = n\times \log(a)$.
$x = \frac{\log(1.67)}{\log(0.27)}$
\end{enumerate}
\end{solution}
\begin{exercise}[subtitle={Étude de fonctions}]
Soit $f(x) = 10x^3 - 840x^2 + 18450x - 1$ une fonction définie sur $\R$.
\begin{enumerate}
\item Calculer $f'(x)$ la dérivée de $f(x)$.
\item Calculer $f'(41)$ et $f'(15)$.
\item En déduire une forme factorisée de $f'(x)$.
\item Étudier le signe de $f'(x)$ et en déduire les variations de $f(x)$.
\item Est-ce que la fonction $f(x)$ admet un maximum ou un minimum? Si oui, calculer sa valeur.
\end{enumerate}
\end{exercise}
\begin{solution}
\begin{enumerate}
\item Dérivée de $f(x)$: $f'(x) = 30x^2 - 1680x + 18450$
\item
\begin{align*}
f'(41) &= 30 \times 41^{2} - 1680 \times 41 + 18450\\&= 30 \times 1681 - 68880 + 18450\\&= 50430 - 50430\\&= 0
\end{align*}
\begin{align*}
f'(15) &= 30 \times 15^{2} - 1680 \times 15 + 18450\\&= 30 \times 225 - 25200 + 18450\\&= 6750 - 6750\\&= 0
\end{align*}
Donc $x = 41$ et $x=15$ sont des racines de $f'(x) = 30x^2 - 1680x + 18450$.
\item On en déduit la forme factorisée suivante
\[
f'(x) = 30 (x - 41)(x-15)
\]
\item Pas de correction disponible
\item À causes des branches extérieurs, la fonction $f(x)$ n'a pas de maximum ou de minimum.
\end{enumerate}
\end{solution}
%\printsolutionstype{exercise}
\end{document}
%%% Local Variables:
%%% mode: latex
%%% TeX-master: "master"
%%% End:

View File

@ -0,0 +1,203 @@
\documentclass[a5paper,10pt]{article}
\usepackage{myXsim}
\usepackage{tasks}
% Title Page
\title{DM2 \hfill BUDIN Nathan}
\tribe{TST}
\date{\hfillÀ render pour le Mercredi 24 février}
\xsimsetup{
solution/print = false
}
\begin{document}
\maketitle
\begin{exercise}[subtitle={Loi binomiale}]
Trois personnes s'apprêtent à passer le portique de sécurité. On suppose que pour chaque personne la probabilité que le portique sonne est égale à $0.37$.
Soit $X$ la variable aléatoire donnant le nombre de personnes faisant sonner le portique, parmi les 3 personnes de ce groupe.
\begin{enumerate}
\item Tracer l'arbre représentant le situation.
\item Justifier que $X$ suit une loi binomiale dont on précisera les paramètres.
\item Quelle est la probabilité qu'une seule personne fasse sonner le portique?
\item Calculer puis interpréter les probabilités suivantes
\[
P(X = 0) \qquad \qquad P(X \geq 2)
\]
\item Calculer l'espérance de $X$ et interpréter le résultat.
\end{enumerate}
\end{exercise}
\begin{solution}
\begin{enumerate}
\item
\begin{tikzpicture}[sloped]
\node {.}
child {node {$0$}
child {node {$0$}
child {node {$0$}
edge from parent
node[above] {0.63}
}
child {node {$1$}
edge from parent
node[above] {0.37}
}
edge from parent
node[above] {0.63}
}
child[missing] {}
child {node {$1$}
child {node {$0$}
edge from parent
node[above] {0.63}
}
child {node {$1$}
edge from parent
node[above] {0.37}
}
edge from parent
node[above] {0.63}
}
edge from parent
node[above] {0.63}
}
child[missing] {}
child[missing] {}
child[missing] {}
child { node {$1$}
child {node {$0$}
child {node {$0$}
edge from parent
node[above] {0.63}
}
child {node {$1$}
edge from parent
node[above] {0.37}
}
edge from parent
node[above] {0.63}
}
child[missing] {}
child {node {$1$}
child {node {$0$}
edge from parent
node[above] {0.63}
}
child {node {$1$}
edge from parent
node[above] {0.37}
}
edge from parent
node[above] {0.63}
}
edge from parent
node[above] {0.37}
} ;
\end{tikzpicture}
\item Chaque personne a 2 possibilités (1: fait sonner ou 2: ne fait pas sonner) et l'on fait passer 3 personnes ce qui correspond à une répétition identique et aléatoire. On peut donc modéliser la situation par une loi binomiale.
\[
X \sim \mathcal{B}(3; 0.76)
\]
\item Probabilité qu'une seule personne fasse sonner le portique. On voit qu'il y a 3 branches qui correspondent à cette situation dont
\[
P(X = 1) = 3 \times 0.37^1 \times 0.63^2 \approx 0.441
\]
\item
\[
P(X = 0) = 0.63^3 \approx 0.25
\]
\[
P(X \geq 2) = P(X = 2) + P(X = 3) = 3 \times 0.37^2 \times 0.63^1 + 0.37^3 \approx 0.31
\]
\item Il faut d'abord tracer le tableau résumant la loi de probabilité:
\begin{center}
\begin{tabular}{|c|*{4}{c|}}
\hline
Valeur & 0 & 1 & 2 & 3 \\
\hline
Probabilité & $0.25$ & $0.441$ & $0.259$ &$0.051$ \\
\hline
\end{tabular}
\end{center}
On peut alors calculer l'espérance
\[
E[X] = 0 \times 0.25 + 1 \times 0.441 + 2 \times 0.259 + 3 \times 0.051 = 1.11
\]
On peut donc estimer qu'il y aura en moyenne $1.11$ personnes qui feront sonner le portique sur les 3 personnes.
\end{enumerate}
\end{solution}
\begin{exercise}[subtitle={Équation puissance}]
Résoudre les équations et inéquations suivantes
\begin{multicols}{2}
\begin{enumerate}
\item $10^x = 44$
\item $2^x = 33$
\item $0.94^x \leq 43$
\item $9 \times 0.17^x = 30$
\end{enumerate}
\end{multicols}
\end{exercise}
\begin{solution}
Les solutions ci-dessous ne sont pas justifiée car l'ordinateur ne sait pas faire. Par contre, vous vous devez savoir justifier vos réponses!
\begin{enumerate}
\item $x = \log(44)$
\item $x = \frac{\log(33)}{\log(2)}$
\item Il faut faire attention quand on divise par un log car ce dernier peut être négatif ce qui est le cas ici. Il faut donc pense à changer le sens de l'inégalité.
$x \geq \frac{\log(43)}{\log(0.94)}$
\item Il faut penser à faire la division à par $9$ avant d'utiliser le log car sinon, on ne peut pas utiliser la formule $\log(a^n) = n\times \log(a)$.
$x = \frac{\log(3.33)}{\log(0.17)}$
\end{enumerate}
\end{solution}
\begin{exercise}[subtitle={Étude de fonctions}]
Soit $f(x) = - 2x^3 + 99x^2 + 2580x + 46$ une fonction définie sur $\R$.
\begin{enumerate}
\item Calculer $f'(x)$ la dérivée de $f(x)$.
\item Calculer $f'(43)$ et $f'(-10)$.
\item En déduire une forme factorisée de $f'(x)$.
\item Étudier le signe de $f'(x)$ et en déduire les variations de $f(x)$.
\item Est-ce que la fonction $f(x)$ admet un maximum ou un minimum? Si oui, calculer sa valeur.
\end{enumerate}
\end{exercise}
\begin{solution}
\begin{enumerate}
\item Dérivée de $f(x)$: $f'(x) = - 6x^2 + 198x + 2580$
\item
\begin{align*}
f'(43) &= - 6 \times 43^{2} + 198 \times 43 + 2580\\&= - 6 \times 1849 + 8514 + 2580\\&= - 11094 + 11094\\&= 0
\end{align*}
\begin{align*}
f'(-10) &= - 6 \times - 10^{2} + 198(- 10) + 2580\\&= - 6 \times 100 - 1980 + 2580\\&= - 600 + 600\\&= 0
\end{align*}
Donc $x = 43$ et $x=-10$ sont des racines de $f'(x) = - 6x^2 + 198x + 2580$.
\item On en déduit la forme factorisée suivante
\[
f'(x) = -6 (x - 43)(x--10)
\]
\item Pas de correction disponible
\item À causes des branches extérieurs, la fonction $f(x)$ n'a pas de maximum ou de minimum.
\end{enumerate}
\end{solution}
%\printsolutionstype{exercise}
\end{document}
%%% Local Variables:
%%% mode: latex
%%% TeX-master: "master"
%%% End:

View File

@ -0,0 +1,203 @@
\documentclass[a5paper,10pt]{article}
\usepackage{myXsim}
\usepackage{tasks}
% Title Page
\title{DM2 \hfill CAGLAR Rojin}
\tribe{TST}
\date{\hfillÀ render pour le Mercredi 24 février}
\xsimsetup{
solution/print = false
}
\begin{document}
\maketitle
\begin{exercise}[subtitle={Loi binomiale}]
Trois personnes s'apprêtent à passer le portique de sécurité. On suppose que pour chaque personne la probabilité que le portique sonne est égale à $0.86$.
Soit $X$ la variable aléatoire donnant le nombre de personnes faisant sonner le portique, parmi les 3 personnes de ce groupe.
\begin{enumerate}
\item Tracer l'arbre représentant le situation.
\item Justifier que $X$ suit une loi binomiale dont on précisera les paramètres.
\item Quelle est la probabilité qu'une seule personne fasse sonner le portique?
\item Calculer puis interpréter les probabilités suivantes
\[
P(X = 0) \qquad \qquad P(X \geq 2)
\]
\item Calculer l'espérance de $X$ et interpréter le résultat.
\end{enumerate}
\end{exercise}
\begin{solution}
\begin{enumerate}
\item
\begin{tikzpicture}[sloped]
\node {.}
child {node {$0$}
child {node {$0$}
child {node {$0$}
edge from parent
node[above] {0.14}
}
child {node {$1$}
edge from parent
node[above] {0.86}
}
edge from parent
node[above] {0.14}
}
child[missing] {}
child {node {$1$}
child {node {$0$}
edge from parent
node[above] {0.14}
}
child {node {$1$}
edge from parent
node[above] {0.86}
}
edge from parent
node[above] {0.14}
}
edge from parent
node[above] {0.14}
}
child[missing] {}
child[missing] {}
child[missing] {}
child { node {$1$}
child {node {$0$}
child {node {$0$}
edge from parent
node[above] {0.14}
}
child {node {$1$}
edge from parent
node[above] {0.86}
}
edge from parent
node[above] {0.14}
}
child[missing] {}
child {node {$1$}
child {node {$0$}
edge from parent
node[above] {0.14}
}
child {node {$1$}
edge from parent
node[above] {0.86}
}
edge from parent
node[above] {0.14}
}
edge from parent
node[above] {0.86}
} ;
\end{tikzpicture}
\item Chaque personne a 2 possibilités (1: fait sonner ou 2: ne fait pas sonner) et l'on fait passer 3 personnes ce qui correspond à une répétition identique et aléatoire. On peut donc modéliser la situation par une loi binomiale.
\[
X \sim \mathcal{B}(3; 0.76)
\]
\item Probabilité qu'une seule personne fasse sonner le portique. On voit qu'il y a 3 branches qui correspondent à cette situation dont
\[
P(X = 1) = 3 \times 0.86^1 \times 0.14^2 \approx 0.051
\]
\item
\[
P(X = 0) = 0.14^3 \approx 0.003
\]
\[
P(X \geq 2) = P(X = 2) + P(X = 3) = 3 \times 0.86^2 \times 0.14^1 + 0.86^3 \approx 0.947
\]
\item Il faut d'abord tracer le tableau résumant la loi de probabilité:
\begin{center}
\begin{tabular}{|c|*{4}{c|}}
\hline
Valeur & 0 & 1 & 2 & 3 \\
\hline
Probabilité & $0.003$ & $0.051$ & $0.311$ &$0.636$ \\
\hline
\end{tabular}
\end{center}
On peut alors calculer l'espérance
\[
E[X] = 0 \times 0.003 + 1 \times 0.051 + 2 \times 0.311 + 3 \times 0.636 = 2.58
\]
On peut donc estimer qu'il y aura en moyenne $2.58$ personnes qui feront sonner le portique sur les 3 personnes.
\end{enumerate}
\end{solution}
\begin{exercise}[subtitle={Équation puissance}]
Résoudre les équations et inéquations suivantes
\begin{multicols}{2}
\begin{enumerate}
\item $10^x = 14$
\item $2^x = 38$
\item $0.15^x \leq 40$
\item $9 \times 0.56^x = 29$
\end{enumerate}
\end{multicols}
\end{exercise}
\begin{solution}
Les solutions ci-dessous ne sont pas justifiée car l'ordinateur ne sait pas faire. Par contre, vous vous devez savoir justifier vos réponses!
\begin{enumerate}
\item $x = \log(14)$
\item $x = \frac{\log(38)}{\log(2)}$
\item Il faut faire attention quand on divise par un log car ce dernier peut être négatif ce qui est le cas ici. Il faut donc pense à changer le sens de l'inégalité.
$x \geq \frac{\log(40)}{\log(0.15)}$
\item Il faut penser à faire la division à par $9$ avant d'utiliser le log car sinon, on ne peut pas utiliser la formule $\log(a^n) = n\times \log(a)$.
$x = \frac{\log(3.22)}{\log(0.56)}$
\end{enumerate}
\end{solution}
\begin{exercise}[subtitle={Étude de fonctions}]
Soit $f(x) = - x^3 + 67.5x^2 - 258x - 11$ une fonction définie sur $\R$.
\begin{enumerate}
\item Calculer $f'(x)$ la dérivée de $f(x)$.
\item Calculer $f'(43)$ et $f'(2)$.
\item En déduire une forme factorisée de $f'(x)$.
\item Étudier le signe de $f'(x)$ et en déduire les variations de $f(x)$.
\item Est-ce que la fonction $f(x)$ admet un maximum ou un minimum? Si oui, calculer sa valeur.
\end{enumerate}
\end{exercise}
\begin{solution}
\begin{enumerate}
\item Dérivée de $f(x)$: $f'(x) = - 3x^2 + 135x - 258$
\item
\begin{align*}
f'(43) &= - 3 \times 43^{2} + 135 \times 43 - 258\\&= - 3 \times 1849 + 5805 - 258\\&= - 5547 + 5547\\&= 0
\end{align*}
\begin{align*}
f'(2) &= - 3 \times 2^{2} + 135 \times 2 - 258\\&= - 3 \times 4 + 270 - 258\\&= - 12 + 12\\&= 0
\end{align*}
Donc $x = 43$ et $x=2$ sont des racines de $f'(x) = - 3x^2 + 135x - 258$.
\item On en déduit la forme factorisée suivante
\[
f'(x) = -3 (x - 43)(x-2)
\]
\item Pas de correction disponible
\item À causes des branches extérieurs, la fonction $f(x)$ n'a pas de maximum ou de minimum.
\end{enumerate}
\end{solution}
%\printsolutionstype{exercise}
\end{document}
%%% Local Variables:
%%% mode: latex
%%% TeX-master: "master"
%%% End:

View File

@ -0,0 +1,203 @@
\documentclass[a5paper,10pt]{article}
\usepackage{myXsim}
\usepackage{tasks}
% Title Page
\title{DM2 \hfill DARICHE Kaïs}
\tribe{TST}
\date{\hfillÀ render pour le Mercredi 24 février}
\xsimsetup{
solution/print = false
}
\begin{document}
\maketitle
\begin{exercise}[subtitle={Loi binomiale}]
Trois personnes s'apprêtent à passer le portique de sécurité. On suppose que pour chaque personne la probabilité que le portique sonne est égale à $0.16$.
Soit $X$ la variable aléatoire donnant le nombre de personnes faisant sonner le portique, parmi les 3 personnes de ce groupe.
\begin{enumerate}
\item Tracer l'arbre représentant le situation.
\item Justifier que $X$ suit une loi binomiale dont on précisera les paramètres.
\item Quelle est la probabilité qu'une seule personne fasse sonner le portique?
\item Calculer puis interpréter les probabilités suivantes
\[
P(X = 0) \qquad \qquad P(X \geq 2)
\]
\item Calculer l'espérance de $X$ et interpréter le résultat.
\end{enumerate}
\end{exercise}
\begin{solution}
\begin{enumerate}
\item
\begin{tikzpicture}[sloped]
\node {.}
child {node {$0$}
child {node {$0$}
child {node {$0$}
edge from parent
node[above] {0.84}
}
child {node {$1$}
edge from parent
node[above] {0.16}
}
edge from parent
node[above] {0.84}
}
child[missing] {}
child {node {$1$}
child {node {$0$}
edge from parent
node[above] {0.84}
}
child {node {$1$}
edge from parent
node[above] {0.16}
}
edge from parent
node[above] {0.84}
}
edge from parent
node[above] {0.84}
}
child[missing] {}
child[missing] {}
child[missing] {}
child { node {$1$}
child {node {$0$}
child {node {$0$}
edge from parent
node[above] {0.84}
}
child {node {$1$}
edge from parent
node[above] {0.16}
}
edge from parent
node[above] {0.84}
}
child[missing] {}
child {node {$1$}
child {node {$0$}
edge from parent
node[above] {0.84}
}
child {node {$1$}
edge from parent
node[above] {0.16}
}
edge from parent
node[above] {0.84}
}
edge from parent
node[above] {0.16}
} ;
\end{tikzpicture}
\item Chaque personne a 2 possibilités (1: fait sonner ou 2: ne fait pas sonner) et l'on fait passer 3 personnes ce qui correspond à une répétition identique et aléatoire. On peut donc modéliser la situation par une loi binomiale.
\[
X \sim \mathcal{B}(3; 0.76)
\]
\item Probabilité qu'une seule personne fasse sonner le portique. On voit qu'il y a 3 branches qui correspondent à cette situation dont
\[
P(X = 1) = 3 \times 0.16^1 \times 0.84^2 \approx 0.339
\]
\item
\[
P(X = 0) = 0.84^3 \approx 0.593
\]
\[
P(X \geq 2) = P(X = 2) + P(X = 3) = 3 \times 0.16^2 \times 0.84^1 + 0.16^3 \approx 0.069
\]
\item Il faut d'abord tracer le tableau résumant la loi de probabilité:
\begin{center}
\begin{tabular}{|c|*{4}{c|}}
\hline
Valeur & 0 & 1 & 2 & 3 \\
\hline
Probabilité & $0.593$ & $0.339$ & $0.065$ &$0.004$ \\
\hline
\end{tabular}
\end{center}
On peut alors calculer l'espérance
\[
E[X] = 0 \times 0.593 + 1 \times 0.339 + 2 \times 0.065 + 3 \times 0.004 = 0.48
\]
On peut donc estimer qu'il y aura en moyenne $0.48$ personnes qui feront sonner le portique sur les 3 personnes.
\end{enumerate}
\end{solution}
\begin{exercise}[subtitle={Équation puissance}]
Résoudre les équations et inéquations suivantes
\begin{multicols}{2}
\begin{enumerate}
\item $10^x = 35$
\item $14^x = 11$
\item $0.39^x \leq 48$
\item $3 \times 0.07^x = 4$
\end{enumerate}
\end{multicols}
\end{exercise}
\begin{solution}
Les solutions ci-dessous ne sont pas justifiée car l'ordinateur ne sait pas faire. Par contre, vous vous devez savoir justifier vos réponses!
\begin{enumerate}
\item $x = \log(35)$
\item $x = \frac{\log(11)}{\log(14)}$
\item Il faut faire attention quand on divise par un log car ce dernier peut être négatif ce qui est le cas ici. Il faut donc pense à changer le sens de l'inégalité.
$x \geq \frac{\log(48)}{\log(0.39)}$
\item Il faut penser à faire la division à par $3$ avant d'utiliser le log car sinon, on ne peut pas utiliser la formule $\log(a^n) = n\times \log(a)$.
$x = \frac{\log(1.33)}{\log(0.07)}$
\end{enumerate}
\end{solution}
\begin{exercise}[subtitle={Étude de fonctions}]
Soit $f(x) = x^3 - 54x^2 + 780x + 36$ une fonction définie sur $\R$.
\begin{enumerate}
\item Calculer $f'(x)$ la dérivée de $f(x)$.
\item Calculer $f'(26)$ et $f'(10)$.
\item En déduire une forme factorisée de $f'(x)$.
\item Étudier le signe de $f'(x)$ et en déduire les variations de $f(x)$.
\item Est-ce que la fonction $f(x)$ admet un maximum ou un minimum? Si oui, calculer sa valeur.
\end{enumerate}
\end{exercise}
\begin{solution}
\begin{enumerate}
\item Dérivée de $f(x)$: $f'(x) = 3x^2 - 108x + 780$
\item
\begin{align*}
f'(26) &= 3 \times 26^{2} - 108 \times 26 + 780\\&= 3 \times 676 - 2808 + 780\\&= 2028 - 2028\\&= 0
\end{align*}
\begin{align*}
f'(10) &= 3 \times 10^{2} - 108 \times 10 + 780\\&= 3 \times 100 - 1080 + 780\\&= 300 - 300\\&= 0
\end{align*}
Donc $x = 26$ et $x=10$ sont des racines de $f'(x) = 3x^2 - 108x + 780$.
\item On en déduit la forme factorisée suivante
\[
f'(x) = 3 (x - 26)(x-10)
\]
\item Pas de correction disponible
\item À causes des branches extérieurs, la fonction $f(x)$ n'a pas de maximum ou de minimum.
\end{enumerate}
\end{solution}
%\printsolutionstype{exercise}
\end{document}
%%% Local Variables:
%%% mode: latex
%%% TeX-master: "master"
%%% End:

View File

@ -0,0 +1,203 @@
\documentclass[a5paper,10pt]{article}
\usepackage{myXsim}
\usepackage{tasks}
% Title Page
\title{DM2 \hfill DEBRAS Noémie}
\tribe{TST}
\date{\hfillÀ render pour le Mercredi 24 février}
\xsimsetup{
solution/print = false
}
\begin{document}
\maketitle
\begin{exercise}[subtitle={Loi binomiale}]
Trois personnes s'apprêtent à passer le portique de sécurité. On suppose que pour chaque personne la probabilité que le portique sonne est égale à $0.1$.
Soit $X$ la variable aléatoire donnant le nombre de personnes faisant sonner le portique, parmi les 3 personnes de ce groupe.
\begin{enumerate}
\item Tracer l'arbre représentant le situation.
\item Justifier que $X$ suit une loi binomiale dont on précisera les paramètres.
\item Quelle est la probabilité qu'une seule personne fasse sonner le portique?
\item Calculer puis interpréter les probabilités suivantes
\[
P(X = 0) \qquad \qquad P(X \geq 2)
\]
\item Calculer l'espérance de $X$ et interpréter le résultat.
\end{enumerate}
\end{exercise}
\begin{solution}
\begin{enumerate}
\item
\begin{tikzpicture}[sloped]
\node {.}
child {node {$0$}
child {node {$0$}
child {node {$0$}
edge from parent
node[above] {0.9}
}
child {node {$1$}
edge from parent
node[above] {0.1}
}
edge from parent
node[above] {0.9}
}
child[missing] {}
child {node {$1$}
child {node {$0$}
edge from parent
node[above] {0.9}
}
child {node {$1$}
edge from parent
node[above] {0.1}
}
edge from parent
node[above] {0.9}
}
edge from parent
node[above] {0.9}
}
child[missing] {}
child[missing] {}
child[missing] {}
child { node {$1$}
child {node {$0$}
child {node {$0$}
edge from parent
node[above] {0.9}
}
child {node {$1$}
edge from parent
node[above] {0.1}
}
edge from parent
node[above] {0.9}
}
child[missing] {}
child {node {$1$}
child {node {$0$}
edge from parent
node[above] {0.9}
}
child {node {$1$}
edge from parent
node[above] {0.1}
}
edge from parent
node[above] {0.9}
}
edge from parent
node[above] {0.1}
} ;
\end{tikzpicture}
\item Chaque personne a 2 possibilités (1: fait sonner ou 2: ne fait pas sonner) et l'on fait passer 3 personnes ce qui correspond à une répétition identique et aléatoire. On peut donc modéliser la situation par une loi binomiale.
\[
X \sim \mathcal{B}(3; 0.76)
\]
\item Probabilité qu'une seule personne fasse sonner le portique. On voit qu'il y a 3 branches qui correspondent à cette situation dont
\[
P(X = 1) = 3 \times 0.1^1 \times 0.9^2 \approx 0.243
\]
\item
\[
P(X = 0) = 0.9^3 \approx 0.729
\]
\[
P(X \geq 2) = P(X = 2) + P(X = 3) = 3 \times 0.1^2 \times 0.9^1 + 0.1^3 \approx 0.028
\]
\item Il faut d'abord tracer le tableau résumant la loi de probabilité:
\begin{center}
\begin{tabular}{|c|*{4}{c|}}
\hline
Valeur & 0 & 1 & 2 & 3 \\
\hline
Probabilité & $0.729$ & $0.243$ & $0.027$ &$0.001$ \\
\hline
\end{tabular}
\end{center}
On peut alors calculer l'espérance
\[
E[X] = 0 \times 0.729 + 1 \times 0.243 + 2 \times 0.027 + 3 \times 0.001 = 0.3
\]
On peut donc estimer qu'il y aura en moyenne $0.3$ personnes qui feront sonner le portique sur les 3 personnes.
\end{enumerate}
\end{solution}
\begin{exercise}[subtitle={Équation puissance}]
Résoudre les équations et inéquations suivantes
\begin{multicols}{2}
\begin{enumerate}
\item $10^x = 25$
\item $12^x = 21$
\item $0.77^x \leq 22$
\item $6 \times 0.4^x = 40$
\end{enumerate}
\end{multicols}
\end{exercise}
\begin{solution}
Les solutions ci-dessous ne sont pas justifiée car l'ordinateur ne sait pas faire. Par contre, vous vous devez savoir justifier vos réponses!
\begin{enumerate}
\item $x = \log(25)$
\item $x = \frac{\log(21)}{\log(12)}$
\item Il faut faire attention quand on divise par un log car ce dernier peut être négatif ce qui est le cas ici. Il faut donc pense à changer le sens de l'inégalité.
$x \geq \frac{\log(22)}{\log(0.77)}$
\item Il faut penser à faire la division à par $6$ avant d'utiliser le log car sinon, on ne peut pas utiliser la formule $\log(a^n) = n\times \log(a)$.
$x = \frac{\log(6.67)}{\log(0.4)}$
\end{enumerate}
\end{solution}
\begin{exercise}[subtitle={Étude de fonctions}]
Soit $f(x) = 4x^3 - 132x^2 - 5460x + 39$ une fonction définie sur $\R$.
\begin{enumerate}
\item Calculer $f'(x)$ la dérivée de $f(x)$.
\item Calculer $f'(35)$ et $f'(-13)$.
\item En déduire une forme factorisée de $f'(x)$.
\item Étudier le signe de $f'(x)$ et en déduire les variations de $f(x)$.
\item Est-ce que la fonction $f(x)$ admet un maximum ou un minimum? Si oui, calculer sa valeur.
\end{enumerate}
\end{exercise}
\begin{solution}
\begin{enumerate}
\item Dérivée de $f(x)$: $f'(x) = 12x^2 - 264x - 5460$
\item
\begin{align*}
f'(35) &= 12 \times 35^{2} - 264 \times 35 - 5460\\&= 12 \times 1225 - 9240 - 5460\\&= 14700 - 14700\\&= 0
\end{align*}
\begin{align*}
f'(-13) &= 12 \times - 13^{2} - 264(- 13) - 5460\\&= 12 \times 169 + 3432 - 5460\\&= 2028 - 2028\\&= 0
\end{align*}
Donc $x = 35$ et $x=-13$ sont des racines de $f'(x) = 12x^2 - 264x - 5460$.
\item On en déduit la forme factorisée suivante
\[
f'(x) = 12 (x - 35)(x--13)
\]
\item Pas de correction disponible
\item À causes des branches extérieurs, la fonction $f(x)$ n'a pas de maximum ou de minimum.
\end{enumerate}
\end{solution}
%\printsolutionstype{exercise}
\end{document}
%%% Local Variables:
%%% mode: latex
%%% TeX-master: "master"
%%% End:

View File

@ -0,0 +1,203 @@
\documentclass[a5paper,10pt]{article}
\usepackage{myXsim}
\usepackage{tasks}
% Title Page
\title{DM2 \hfill GERMAIN Anaïs}
\tribe{TST}
\date{\hfillÀ render pour le Mercredi 24 février}
\xsimsetup{
solution/print = false
}
\begin{document}
\maketitle
\begin{exercise}[subtitle={Loi binomiale}]
Trois personnes s'apprêtent à passer le portique de sécurité. On suppose que pour chaque personne la probabilité que le portique sonne est égale à $0.68$.
Soit $X$ la variable aléatoire donnant le nombre de personnes faisant sonner le portique, parmi les 3 personnes de ce groupe.
\begin{enumerate}
\item Tracer l'arbre représentant le situation.
\item Justifier que $X$ suit une loi binomiale dont on précisera les paramètres.
\item Quelle est la probabilité qu'une seule personne fasse sonner le portique?
\item Calculer puis interpréter les probabilités suivantes
\[
P(X = 0) \qquad \qquad P(X \geq 2)
\]
\item Calculer l'espérance de $X$ et interpréter le résultat.
\end{enumerate}
\end{exercise}
\begin{solution}
\begin{enumerate}
\item
\begin{tikzpicture}[sloped]
\node {.}
child {node {$0$}
child {node {$0$}
child {node {$0$}
edge from parent
node[above] {0.32}
}
child {node {$1$}
edge from parent
node[above] {0.68}
}
edge from parent
node[above] {0.32}
}
child[missing] {}
child {node {$1$}
child {node {$0$}
edge from parent
node[above] {0.32}
}
child {node {$1$}
edge from parent
node[above] {0.68}
}
edge from parent
node[above] {0.32}
}
edge from parent
node[above] {0.32}
}
child[missing] {}
child[missing] {}
child[missing] {}
child { node {$1$}
child {node {$0$}
child {node {$0$}
edge from parent
node[above] {0.32}
}
child {node {$1$}
edge from parent
node[above] {0.68}
}
edge from parent
node[above] {0.32}
}
child[missing] {}
child {node {$1$}
child {node {$0$}
edge from parent
node[above] {0.32}
}
child {node {$1$}
edge from parent
node[above] {0.68}
}
edge from parent
node[above] {0.32}
}
edge from parent
node[above] {0.68}
} ;
\end{tikzpicture}
\item Chaque personne a 2 possibilités (1: fait sonner ou 2: ne fait pas sonner) et l'on fait passer 3 personnes ce qui correspond à une répétition identique et aléatoire. On peut donc modéliser la situation par une loi binomiale.
\[
X \sim \mathcal{B}(3; 0.76)
\]
\item Probabilité qu'une seule personne fasse sonner le portique. On voit qu'il y a 3 branches qui correspondent à cette situation dont
\[
P(X = 1) = 3 \times 0.68^1 \times 0.32^2 \approx 0.209
\]
\item
\[
P(X = 0) = 0.32^3 \approx 0.033
\]
\[
P(X \geq 2) = P(X = 2) + P(X = 3) = 3 \times 0.68^2 \times 0.32^1 + 0.68^3 \approx 0.758
\]
\item Il faut d'abord tracer le tableau résumant la loi de probabilité:
\begin{center}
\begin{tabular}{|c|*{4}{c|}}
\hline
Valeur & 0 & 1 & 2 & 3 \\
\hline
Probabilité & $0.033$ & $0.209$ & $0.444$ &$0.314$ \\
\hline
\end{tabular}
\end{center}
On peut alors calculer l'espérance
\[
E[X] = 0 \times 0.033 + 1 \times 0.209 + 2 \times 0.444 + 3 \times 0.314 = 2.04
\]
On peut donc estimer qu'il y aura en moyenne $2.04$ personnes qui feront sonner le portique sur les 3 personnes.
\end{enumerate}
\end{solution}
\begin{exercise}[subtitle={Équation puissance}]
Résoudre les équations et inéquations suivantes
\begin{multicols}{2}
\begin{enumerate}
\item $10^x = 48$
\item $2^x = 42$
\item $0.47^x \leq 46$
\item $7 \times 0.37^x = 45$
\end{enumerate}
\end{multicols}
\end{exercise}
\begin{solution}
Les solutions ci-dessous ne sont pas justifiée car l'ordinateur ne sait pas faire. Par contre, vous vous devez savoir justifier vos réponses!
\begin{enumerate}
\item $x = \log(48)$
\item $x = \frac{\log(42)}{\log(2)}$
\item Il faut faire attention quand on divise par un log car ce dernier peut être négatif ce qui est le cas ici. Il faut donc pense à changer le sens de l'inégalité.
$x \geq \frac{\log(46)}{\log(0.47)}$
\item Il faut penser à faire la division à par $7$ avant d'utiliser le log car sinon, on ne peut pas utiliser la formule $\log(a^n) = n\times \log(a)$.
$x = \frac{\log(6.43)}{\log(0.37)}$
\end{enumerate}
\end{solution}
\begin{exercise}[subtitle={Étude de fonctions}]
Soit $f(x) = - 7x^3 + 472.5x^2 + 4116x + 48$ une fonction définie sur $\R$.
\begin{enumerate}
\item Calculer $f'(x)$ la dérivée de $f(x)$.
\item Calculer $f'(49)$ et $f'(-4)$.
\item En déduire une forme factorisée de $f'(x)$.
\item Étudier le signe de $f'(x)$ et en déduire les variations de $f(x)$.
\item Est-ce que la fonction $f(x)$ admet un maximum ou un minimum? Si oui, calculer sa valeur.
\end{enumerate}
\end{exercise}
\begin{solution}
\begin{enumerate}
\item Dérivée de $f(x)$: $f'(x) = - 21x^2 + 945x + 4116$
\item
\begin{align*}
f'(49) &= - 21 \times 49^{2} + 945 \times 49 + 4116\\&= - 21 \times 2401 + 46305 + 4116\\&= - 50421 + 50421\\&= 0
\end{align*}
\begin{align*}
f'(-4) &= - 21 \times - 4^{2} + 945(- 4) + 4116\\&= - 21 \times 16 - 3780 + 4116\\&= - 336 + 336\\&= 0
\end{align*}
Donc $x = 49$ et $x=-4$ sont des racines de $f'(x) = - 21x^2 + 945x + 4116$.
\item On en déduit la forme factorisée suivante
\[
f'(x) = -21 (x - 49)(x--4)
\]
\item Pas de correction disponible
\item À causes des branches extérieurs, la fonction $f(x)$ n'a pas de maximum ou de minimum.
\end{enumerate}
\end{solution}
%\printsolutionstype{exercise}
\end{document}
%%% Local Variables:
%%% mode: latex
%%% TeX-master: "master"
%%% End:

View File

@ -0,0 +1,203 @@
\documentclass[a5paper,10pt]{article}
\usepackage{myXsim}
\usepackage{tasks}
% Title Page
\title{DM2 \hfill HADJRAS Mohcine}
\tribe{TST}
\date{\hfillÀ render pour le Mercredi 24 février}
\xsimsetup{
solution/print = false
}
\begin{document}
\maketitle
\begin{exercise}[subtitle={Loi binomiale}]
Trois personnes s'apprêtent à passer le portique de sécurité. On suppose que pour chaque personne la probabilité que le portique sonne est égale à $0.31$.
Soit $X$ la variable aléatoire donnant le nombre de personnes faisant sonner le portique, parmi les 3 personnes de ce groupe.
\begin{enumerate}
\item Tracer l'arbre représentant le situation.
\item Justifier que $X$ suit une loi binomiale dont on précisera les paramètres.
\item Quelle est la probabilité qu'une seule personne fasse sonner le portique?
\item Calculer puis interpréter les probabilités suivantes
\[
P(X = 0) \qquad \qquad P(X \geq 2)
\]
\item Calculer l'espérance de $X$ et interpréter le résultat.
\end{enumerate}
\end{exercise}
\begin{solution}
\begin{enumerate}
\item
\begin{tikzpicture}[sloped]
\node {.}
child {node {$0$}
child {node {$0$}
child {node {$0$}
edge from parent
node[above] {0.69}
}
child {node {$1$}
edge from parent
node[above] {0.31}
}
edge from parent
node[above] {0.69}
}
child[missing] {}
child {node {$1$}
child {node {$0$}
edge from parent
node[above] {0.69}
}
child {node {$1$}
edge from parent
node[above] {0.31}
}
edge from parent
node[above] {0.69}
}
edge from parent
node[above] {0.69}
}
child[missing] {}
child[missing] {}
child[missing] {}
child { node {$1$}
child {node {$0$}
child {node {$0$}
edge from parent
node[above] {0.69}
}
child {node {$1$}
edge from parent
node[above] {0.31}
}
edge from parent
node[above] {0.69}
}
child[missing] {}
child {node {$1$}
child {node {$0$}
edge from parent
node[above] {0.69}
}
child {node {$1$}
edge from parent
node[above] {0.31}
}
edge from parent
node[above] {0.69}
}
edge from parent
node[above] {0.31}
} ;
\end{tikzpicture}
\item Chaque personne a 2 possibilités (1: fait sonner ou 2: ne fait pas sonner) et l'on fait passer 3 personnes ce qui correspond à une répétition identique et aléatoire. On peut donc modéliser la situation par une loi binomiale.
\[
X \sim \mathcal{B}(3; 0.76)
\]
\item Probabilité qu'une seule personne fasse sonner le portique. On voit qu'il y a 3 branches qui correspondent à cette situation dont
\[
P(X = 1) = 3 \times 0.31^1 \times 0.69^2 \approx 0.443
\]
\item
\[
P(X = 0) = 0.69^3 \approx 0.329
\]
\[
P(X \geq 2) = P(X = 2) + P(X = 3) = 3 \times 0.31^2 \times 0.69^1 + 0.31^3 \approx 0.229
\]
\item Il faut d'abord tracer le tableau résumant la loi de probabilité:
\begin{center}
\begin{tabular}{|c|*{4}{c|}}
\hline
Valeur & 0 & 1 & 2 & 3 \\
\hline
Probabilité & $0.329$ & $0.443$ & $0.199$ &$0.03$ \\
\hline
\end{tabular}
\end{center}
On peut alors calculer l'espérance
\[
E[X] = 0 \times 0.329 + 1 \times 0.443 + 2 \times 0.199 + 3 \times 0.03 = 0.93
\]
On peut donc estimer qu'il y aura en moyenne $0.93$ personnes qui feront sonner le portique sur les 3 personnes.
\end{enumerate}
\end{solution}
\begin{exercise}[subtitle={Équation puissance}]
Résoudre les équations et inéquations suivantes
\begin{multicols}{2}
\begin{enumerate}
\item $10^x = 10$
\item $11^x = 20$
\item $0.09^x \leq 22$
\item $6 \times 0.16^x = 45$
\end{enumerate}
\end{multicols}
\end{exercise}
\begin{solution}
Les solutions ci-dessous ne sont pas justifiée car l'ordinateur ne sait pas faire. Par contre, vous vous devez savoir justifier vos réponses!
\begin{enumerate}
\item $x = \log(10)$
\item $x = \frac{\log(20)}{\log(11)}$
\item Il faut faire attention quand on divise par un log car ce dernier peut être négatif ce qui est le cas ici. Il faut donc pense à changer le sens de l'inégalité.
$x \geq \frac{\log(22)}{\log(0.09)}$
\item Il faut penser à faire la division à par $6$ avant d'utiliser le log car sinon, on ne peut pas utiliser la formule $\log(a^n) = n\times \log(a)$.
$x = \frac{\log(7.5)}{\log(0.16)}$
\end{enumerate}
\end{solution}
\begin{exercise}[subtitle={Étude de fonctions}]
Soit $f(x) = 3x^3 - 225x^2 + 4896x + 16$ une fonction définie sur $\R$.
\begin{enumerate}
\item Calculer $f'(x)$ la dérivée de $f(x)$.
\item Calculer $f'(34)$ et $f'(16)$.
\item En déduire une forme factorisée de $f'(x)$.
\item Étudier le signe de $f'(x)$ et en déduire les variations de $f(x)$.
\item Est-ce que la fonction $f(x)$ admet un maximum ou un minimum? Si oui, calculer sa valeur.
\end{enumerate}
\end{exercise}
\begin{solution}
\begin{enumerate}
\item Dérivée de $f(x)$: $f'(x) = 9x^2 - 450x + 4896$
\item
\begin{align*}
f'(34) &= 9 \times 34^{2} - 450 \times 34 + 4896\\&= 9 \times 1156 - 15300 + 4896\\&= 10404 - 10404\\&= 0
\end{align*}
\begin{align*}
f'(16) &= 9 \times 16^{2} - 450 \times 16 + 4896\\&= 9 \times 256 - 7200 + 4896\\&= 2304 - 2304\\&= 0
\end{align*}
Donc $x = 34$ et $x=16$ sont des racines de $f'(x) = 9x^2 - 450x + 4896$.
\item On en déduit la forme factorisée suivante
\[
f'(x) = 9 (x - 34)(x-16)
\]
\item Pas de correction disponible
\item À causes des branches extérieurs, la fonction $f(x)$ n'a pas de maximum ou de minimum.
\end{enumerate}
\end{solution}
%\printsolutionstype{exercise}
\end{document}
%%% Local Variables:
%%% mode: latex
%%% TeX-master: "master"
%%% End:

View File

@ -0,0 +1,203 @@
\documentclass[a5paper,10pt]{article}
\usepackage{myXsim}
\usepackage{tasks}
% Title Page
\title{DM2 \hfill HENRIST Maxime}
\tribe{TST}
\date{\hfillÀ render pour le Mercredi 24 février}
\xsimsetup{
solution/print = false
}
\begin{document}
\maketitle
\begin{exercise}[subtitle={Loi binomiale}]
Trois personnes s'apprêtent à passer le portique de sécurité. On suppose que pour chaque personne la probabilité que le portique sonne est égale à $0.41$.
Soit $X$ la variable aléatoire donnant le nombre de personnes faisant sonner le portique, parmi les 3 personnes de ce groupe.
\begin{enumerate}
\item Tracer l'arbre représentant le situation.
\item Justifier que $X$ suit une loi binomiale dont on précisera les paramètres.
\item Quelle est la probabilité qu'une seule personne fasse sonner le portique?
\item Calculer puis interpréter les probabilités suivantes
\[
P(X = 0) \qquad \qquad P(X \geq 2)
\]
\item Calculer l'espérance de $X$ et interpréter le résultat.
\end{enumerate}
\end{exercise}
\begin{solution}
\begin{enumerate}
\item
\begin{tikzpicture}[sloped]
\node {.}
child {node {$0$}
child {node {$0$}
child {node {$0$}
edge from parent
node[above] {0.59}
}
child {node {$1$}
edge from parent
node[above] {0.41}
}
edge from parent
node[above] {0.59}
}
child[missing] {}
child {node {$1$}
child {node {$0$}
edge from parent
node[above] {0.59}
}
child {node {$1$}
edge from parent
node[above] {0.41}
}
edge from parent
node[above] {0.59}
}
edge from parent
node[above] {0.59}
}
child[missing] {}
child[missing] {}
child[missing] {}
child { node {$1$}
child {node {$0$}
child {node {$0$}
edge from parent
node[above] {0.59}
}
child {node {$1$}
edge from parent
node[above] {0.41}
}
edge from parent
node[above] {0.59}
}
child[missing] {}
child {node {$1$}
child {node {$0$}
edge from parent
node[above] {0.59}
}
child {node {$1$}
edge from parent
node[above] {0.41}
}
edge from parent
node[above] {0.59}
}
edge from parent
node[above] {0.41}
} ;
\end{tikzpicture}
\item Chaque personne a 2 possibilités (1: fait sonner ou 2: ne fait pas sonner) et l'on fait passer 3 personnes ce qui correspond à une répétition identique et aléatoire. On peut donc modéliser la situation par une loi binomiale.
\[
X \sim \mathcal{B}(3; 0.76)
\]
\item Probabilité qu'une seule personne fasse sonner le portique. On voit qu'il y a 3 branches qui correspondent à cette situation dont
\[
P(X = 1) = 3 \times 0.41^1 \times 0.59^2 \approx 0.428
\]
\item
\[
P(X = 0) = 0.59^3 \approx 0.205
\]
\[
P(X \geq 2) = P(X = 2) + P(X = 3) = 3 \times 0.41^2 \times 0.59^1 + 0.41^3 \approx 0.367
\]
\item Il faut d'abord tracer le tableau résumant la loi de probabilité:
\begin{center}
\begin{tabular}{|c|*{4}{c|}}
\hline
Valeur & 0 & 1 & 2 & 3 \\
\hline
Probabilité & $0.205$ & $0.428$ & $0.298$ &$0.069$ \\
\hline
\end{tabular}
\end{center}
On peut alors calculer l'espérance
\[
E[X] = 0 \times 0.205 + 1 \times 0.428 + 2 \times 0.298 + 3 \times 0.069 = 1.23
\]
On peut donc estimer qu'il y aura en moyenne $1.23$ personnes qui feront sonner le portique sur les 3 personnes.
\end{enumerate}
\end{solution}
\begin{exercise}[subtitle={Équation puissance}]
Résoudre les équations et inéquations suivantes
\begin{multicols}{2}
\begin{enumerate}
\item $10^x = 24$
\item $14^x = 16$
\item $0.35^x \leq 34$
\item $8 \times 0.25^x = 36$
\end{enumerate}
\end{multicols}
\end{exercise}
\begin{solution}
Les solutions ci-dessous ne sont pas justifiée car l'ordinateur ne sait pas faire. Par contre, vous vous devez savoir justifier vos réponses!
\begin{enumerate}
\item $x = \log(24)$
\item $x = \frac{\log(16)}{\log(14)}$
\item Il faut faire attention quand on divise par un log car ce dernier peut être négatif ce qui est le cas ici. Il faut donc pense à changer le sens de l'inégalité.
$x \geq \frac{\log(34)}{\log(0.35)}$
\item Il faut penser à faire la division à par $8$ avant d'utiliser le log car sinon, on ne peut pas utiliser la formule $\log(a^n) = n\times \log(a)$.
$x = \frac{\log(4.5)}{\log(0.25)}$
\end{enumerate}
\end{solution}
\begin{exercise}[subtitle={Étude de fonctions}]
Soit $f(x) = 7x^3 - 346.5x^2 - 4914x + 30$ une fonction définie sur $\R$.
\begin{enumerate}
\item Calculer $f'(x)$ la dérivée de $f(x)$.
\item Calculer $f'(39)$ et $f'(-6)$.
\item En déduire une forme factorisée de $f'(x)$.
\item Étudier le signe de $f'(x)$ et en déduire les variations de $f(x)$.
\item Est-ce que la fonction $f(x)$ admet un maximum ou un minimum? Si oui, calculer sa valeur.
\end{enumerate}
\end{exercise}
\begin{solution}
\begin{enumerate}
\item Dérivée de $f(x)$: $f'(x) = 21x^2 - 693x - 4914$
\item
\begin{align*}
f'(39) &= 21 \times 39^{2} - 693 \times 39 - 4914\\&= 21 \times 1521 - 27027 - 4914\\&= 31941 - 31941\\&= 0
\end{align*}
\begin{align*}
f'(-6) &= 21 \times - 6^{2} - 693(- 6) - 4914\\&= 21 \times 36 + 4158 - 4914\\&= 756 - 756\\&= 0
\end{align*}
Donc $x = 39$ et $x=-6$ sont des racines de $f'(x) = 21x^2 - 693x - 4914$.
\item On en déduit la forme factorisée suivante
\[
f'(x) = 21 (x - 39)(x--6)
\]
\item Pas de correction disponible
\item À causes des branches extérieurs, la fonction $f(x)$ n'a pas de maximum ou de minimum.
\end{enumerate}
\end{solution}
%\printsolutionstype{exercise}
\end{document}
%%% Local Variables:
%%% mode: latex
%%% TeX-master: "master"
%%% End:

View File

@ -0,0 +1,203 @@
\documentclass[a5paper,10pt]{article}
\usepackage{myXsim}
\usepackage{tasks}
% Title Page
\title{DM2 \hfill INFANTES Antoine}
\tribe{TST}
\date{\hfillÀ render pour le Mercredi 24 février}
\xsimsetup{
solution/print = false
}
\begin{document}
\maketitle
\begin{exercise}[subtitle={Loi binomiale}]
Trois personnes s'apprêtent à passer le portique de sécurité. On suppose que pour chaque personne la probabilité que le portique sonne est égale à $0.58$.
Soit $X$ la variable aléatoire donnant le nombre de personnes faisant sonner le portique, parmi les 3 personnes de ce groupe.
\begin{enumerate}
\item Tracer l'arbre représentant le situation.
\item Justifier que $X$ suit une loi binomiale dont on précisera les paramètres.
\item Quelle est la probabilité qu'une seule personne fasse sonner le portique?
\item Calculer puis interpréter les probabilités suivantes
\[
P(X = 0) \qquad \qquad P(X \geq 2)
\]
\item Calculer l'espérance de $X$ et interpréter le résultat.
\end{enumerate}
\end{exercise}
\begin{solution}
\begin{enumerate}
\item
\begin{tikzpicture}[sloped]
\node {.}
child {node {$0$}
child {node {$0$}
child {node {$0$}
edge from parent
node[above] {0.42}
}
child {node {$1$}
edge from parent
node[above] {0.58}
}
edge from parent
node[above] {0.42}
}
child[missing] {}
child {node {$1$}
child {node {$0$}
edge from parent
node[above] {0.42}
}
child {node {$1$}
edge from parent
node[above] {0.58}
}
edge from parent
node[above] {0.42}
}
edge from parent
node[above] {0.42}
}
child[missing] {}
child[missing] {}
child[missing] {}
child { node {$1$}
child {node {$0$}
child {node {$0$}
edge from parent
node[above] {0.42}
}
child {node {$1$}
edge from parent
node[above] {0.58}
}
edge from parent
node[above] {0.42}
}
child[missing] {}
child {node {$1$}
child {node {$0$}
edge from parent
node[above] {0.42}
}
child {node {$1$}
edge from parent
node[above] {0.58}
}
edge from parent
node[above] {0.42}
}
edge from parent
node[above] {0.58}
} ;
\end{tikzpicture}
\item Chaque personne a 2 possibilités (1: fait sonner ou 2: ne fait pas sonner) et l'on fait passer 3 personnes ce qui correspond à une répétition identique et aléatoire. On peut donc modéliser la situation par une loi binomiale.
\[
X \sim \mathcal{B}(3; 0.76)
\]
\item Probabilité qu'une seule personne fasse sonner le portique. On voit qu'il y a 3 branches qui correspondent à cette situation dont
\[
P(X = 1) = 3 \times 0.58^1 \times 0.42^2 \approx 0.307
\]
\item
\[
P(X = 0) = 0.42^3 \approx 0.074
\]
\[
P(X \geq 2) = P(X = 2) + P(X = 3) = 3 \times 0.58^2 \times 0.42^1 + 0.58^3 \approx 0.619
\]
\item Il faut d'abord tracer le tableau résumant la loi de probabilité:
\begin{center}
\begin{tabular}{|c|*{4}{c|}}
\hline
Valeur & 0 & 1 & 2 & 3 \\
\hline
Probabilité & $0.074$ & $0.307$ & $0.424$ &$0.195$ \\
\hline
\end{tabular}
\end{center}
On peut alors calculer l'espérance
\[
E[X] = 0 \times 0.074 + 1 \times 0.307 + 2 \times 0.424 + 3 \times 0.195 = 1.74
\]
On peut donc estimer qu'il y aura en moyenne $1.74$ personnes qui feront sonner le portique sur les 3 personnes.
\end{enumerate}
\end{solution}
\begin{exercise}[subtitle={Équation puissance}]
Résoudre les équations et inéquations suivantes
\begin{multicols}{2}
\begin{enumerate}
\item $10^x = 1$
\item $11^x = 10$
\item $0.52^x \leq 14$
\item $5 \times 0.49^x = 45$
\end{enumerate}
\end{multicols}
\end{exercise}
\begin{solution}
Les solutions ci-dessous ne sont pas justifiée car l'ordinateur ne sait pas faire. Par contre, vous vous devez savoir justifier vos réponses!
\begin{enumerate}
\item $x = \log(1)$
\item $x = \frac{\log(10)}{\log(11)}$
\item Il faut faire attention quand on divise par un log car ce dernier peut être négatif ce qui est le cas ici. Il faut donc pense à changer le sens de l'inégalité.
$x \geq \frac{\log(14)}{\log(0.52)}$
\item Il faut penser à faire la division à par $5$ avant d'utiliser le log car sinon, on ne peut pas utiliser la formule $\log(a^n) = n\times \log(a)$.
$x = \frac{\log(9.0)}{\log(0.49)}$
\end{enumerate}
\end{solution}
\begin{exercise}[subtitle={Étude de fonctions}]
Soit $f(x) = - 9x^3 + 364.5x^2 + 756x - 21$ une fonction définie sur $\R$.
\begin{enumerate}
\item Calculer $f'(x)$ la dérivée de $f(x)$.
\item Calculer $f'(28)$ et $f'(-1)$.
\item En déduire une forme factorisée de $f'(x)$.
\item Étudier le signe de $f'(x)$ et en déduire les variations de $f(x)$.
\item Est-ce que la fonction $f(x)$ admet un maximum ou un minimum? Si oui, calculer sa valeur.
\end{enumerate}
\end{exercise}
\begin{solution}
\begin{enumerate}
\item Dérivée de $f(x)$: $f'(x) = - 27x^2 + 729x + 756$
\item
\begin{align*}
f'(28) &= - 27 \times 28^{2} + 729 \times 28 + 756\\&= - 27 \times 784 + 20412 + 756\\&= - 21168 + 21168\\&= 0
\end{align*}
\begin{align*}
f'(-1) &= - 27 \times - 1^{2} + 729(- 1) + 756\\&= - 27 \times 1 - 729 + 756\\&= - 27 + 27\\&= 0
\end{align*}
Donc $x = 28$ et $x=-1$ sont des racines de $f'(x) = - 27x^2 + 729x + 756$.
\item On en déduit la forme factorisée suivante
\[
f'(x) = -27 (x - 28)(x--1)
\]
\item Pas de correction disponible
\item À causes des branches extérieurs, la fonction $f(x)$ n'a pas de maximum ou de minimum.
\end{enumerate}
\end{solution}
%\printsolutionstype{exercise}
\end{document}
%%% Local Variables:
%%% mode: latex
%%% TeX-master: "master"
%%% End:

View File

@ -0,0 +1,203 @@
\documentclass[a5paper,10pt]{article}
\usepackage{myXsim}
\usepackage{tasks}
% Title Page
\title{DM2 \hfill MAGRO Robin}
\tribe{TST}
\date{\hfillÀ render pour le Mercredi 24 février}
\xsimsetup{
solution/print = false
}
\begin{document}
\maketitle
\begin{exercise}[subtitle={Loi binomiale}]
Trois personnes s'apprêtent à passer le portique de sécurité. On suppose que pour chaque personne la probabilité que le portique sonne est égale à $0.59$.
Soit $X$ la variable aléatoire donnant le nombre de personnes faisant sonner le portique, parmi les 3 personnes de ce groupe.
\begin{enumerate}
\item Tracer l'arbre représentant le situation.
\item Justifier que $X$ suit une loi binomiale dont on précisera les paramètres.
\item Quelle est la probabilité qu'une seule personne fasse sonner le portique?
\item Calculer puis interpréter les probabilités suivantes
\[
P(X = 0) \qquad \qquad P(X \geq 2)
\]
\item Calculer l'espérance de $X$ et interpréter le résultat.
\end{enumerate}
\end{exercise}
\begin{solution}
\begin{enumerate}
\item
\begin{tikzpicture}[sloped]
\node {.}
child {node {$0$}
child {node {$0$}
child {node {$0$}
edge from parent
node[above] {0.41}
}
child {node {$1$}
edge from parent
node[above] {0.59}
}
edge from parent
node[above] {0.41}
}
child[missing] {}
child {node {$1$}
child {node {$0$}
edge from parent
node[above] {0.41}
}
child {node {$1$}
edge from parent
node[above] {0.59}
}
edge from parent
node[above] {0.41}
}
edge from parent
node[above] {0.41}
}
child[missing] {}
child[missing] {}
child[missing] {}
child { node {$1$}
child {node {$0$}
child {node {$0$}
edge from parent
node[above] {0.41}
}
child {node {$1$}
edge from parent
node[above] {0.59}
}
edge from parent
node[above] {0.41}
}
child[missing] {}
child {node {$1$}
child {node {$0$}
edge from parent
node[above] {0.41}
}
child {node {$1$}
edge from parent
node[above] {0.59}
}
edge from parent
node[above] {0.41}
}
edge from parent
node[above] {0.59}
} ;
\end{tikzpicture}
\item Chaque personne a 2 possibilités (1: fait sonner ou 2: ne fait pas sonner) et l'on fait passer 3 personnes ce qui correspond à une répétition identique et aléatoire. On peut donc modéliser la situation par une loi binomiale.
\[
X \sim \mathcal{B}(3; 0.76)
\]
\item Probabilité qu'une seule personne fasse sonner le portique. On voit qu'il y a 3 branches qui correspondent à cette situation dont
\[
P(X = 1) = 3 \times 0.59^1 \times 0.41^2 \approx 0.298
\]
\item
\[
P(X = 0) = 0.41^3 \approx 0.069
\]
\[
P(X \geq 2) = P(X = 2) + P(X = 3) = 3 \times 0.59^2 \times 0.41^1 + 0.59^3 \approx 0.633
\]
\item Il faut d'abord tracer le tableau résumant la loi de probabilité:
\begin{center}
\begin{tabular}{|c|*{4}{c|}}
\hline
Valeur & 0 & 1 & 2 & 3 \\
\hline
Probabilité & $0.069$ & $0.298$ & $0.428$ &$0.205$ \\
\hline
\end{tabular}
\end{center}
On peut alors calculer l'espérance
\[
E[X] = 0 \times 0.069 + 1 \times 0.298 + 2 \times 0.428 + 3 \times 0.205 = 1.77
\]
On peut donc estimer qu'il y aura en moyenne $1.77$ personnes qui feront sonner le portique sur les 3 personnes.
\end{enumerate}
\end{solution}
\begin{exercise}[subtitle={Équation puissance}]
Résoudre les équations et inéquations suivantes
\begin{multicols}{2}
\begin{enumerate}
\item $10^x = 5$
\item $15^x = 1$
\item $0.57^x \leq 45$
\item $3 \times 0.51^x = 21$
\end{enumerate}
\end{multicols}
\end{exercise}
\begin{solution}
Les solutions ci-dessous ne sont pas justifiée car l'ordinateur ne sait pas faire. Par contre, vous vous devez savoir justifier vos réponses!
\begin{enumerate}
\item $x = \log(5)$
\item $x = \frac{\log(1)}{\log(15)}$
\item Il faut faire attention quand on divise par un log car ce dernier peut être négatif ce qui est le cas ici. Il faut donc pense à changer le sens de l'inégalité.
$x \geq \frac{\log(45)}{\log(0.57)}$
\item Il faut penser à faire la division à par $3$ avant d'utiliser le log car sinon, on ne peut pas utiliser la formule $\log(a^n) = n\times \log(a)$.
$x = \frac{\log(7.0)}{\log(0.51)}$
\end{enumerate}
\end{solution}
\begin{exercise}[subtitle={Étude de fonctions}]
Soit $f(x) = - x^3 + 43.5x^2 + 510x - 37$ une fonction définie sur $\R$.
\begin{enumerate}
\item Calculer $f'(x)$ la dérivée de $f(x)$.
\item Calculer $f'(34)$ et $f'(-5)$.
\item En déduire une forme factorisée de $f'(x)$.
\item Étudier le signe de $f'(x)$ et en déduire les variations de $f(x)$.
\item Est-ce que la fonction $f(x)$ admet un maximum ou un minimum? Si oui, calculer sa valeur.
\end{enumerate}
\end{exercise}
\begin{solution}
\begin{enumerate}
\item Dérivée de $f(x)$: $f'(x) = - 3x^2 + 87x + 510$
\item
\begin{align*}
f'(34) &= - 3 \times 34^{2} + 87 \times 34 + 510\\&= - 3 \times 1156 + 2958 + 510\\&= - 3468 + 3468\\&= 0
\end{align*}
\begin{align*}
f'(-5) &= - 3 \times - 5^{2} + 87(- 5) + 510\\&= - 3 \times 25 - 435 + 510\\&= - 75 + 75\\&= 0
\end{align*}
Donc $x = 34$ et $x=-5$ sont des racines de $f'(x) = - 3x^2 + 87x + 510$.
\item On en déduit la forme factorisée suivante
\[
f'(x) = -3 (x - 34)(x--5)
\]
\item Pas de correction disponible
\item À causes des branches extérieurs, la fonction $f(x)$ n'a pas de maximum ou de minimum.
\end{enumerate}
\end{solution}
%\printsolutionstype{exercise}
\end{document}
%%% Local Variables:
%%% mode: latex
%%% TeX-master: "master"
%%% End:

View File

@ -0,0 +1,203 @@
\documentclass[a5paper,10pt]{article}
\usepackage{myXsim}
\usepackage{tasks}
% Title Page
\title{DM2 \hfill MORFIN Chloé}
\tribe{TST}
\date{\hfillÀ render pour le Mercredi 24 février}
\xsimsetup{
solution/print = false
}
\begin{document}
\maketitle
\begin{exercise}[subtitle={Loi binomiale}]
Trois personnes s'apprêtent à passer le portique de sécurité. On suppose que pour chaque personne la probabilité que le portique sonne est égale à $0.96$.
Soit $X$ la variable aléatoire donnant le nombre de personnes faisant sonner le portique, parmi les 3 personnes de ce groupe.
\begin{enumerate}
\item Tracer l'arbre représentant le situation.
\item Justifier que $X$ suit une loi binomiale dont on précisera les paramètres.
\item Quelle est la probabilité qu'une seule personne fasse sonner le portique?
\item Calculer puis interpréter les probabilités suivantes
\[
P(X = 0) \qquad \qquad P(X \geq 2)
\]
\item Calculer l'espérance de $X$ et interpréter le résultat.
\end{enumerate}
\end{exercise}
\begin{solution}
\begin{enumerate}
\item
\begin{tikzpicture}[sloped]
\node {.}
child {node {$0$}
child {node {$0$}
child {node {$0$}
edge from parent
node[above] {0.04}
}
child {node {$1$}
edge from parent
node[above] {0.96}
}
edge from parent
node[above] {0.04}
}
child[missing] {}
child {node {$1$}
child {node {$0$}
edge from parent
node[above] {0.04}
}
child {node {$1$}
edge from parent
node[above] {0.96}
}
edge from parent
node[above] {0.04}
}
edge from parent
node[above] {0.04}
}
child[missing] {}
child[missing] {}
child[missing] {}
child { node {$1$}
child {node {$0$}
child {node {$0$}
edge from parent
node[above] {0.04}
}
child {node {$1$}
edge from parent
node[above] {0.96}
}
edge from parent
node[above] {0.04}
}
child[missing] {}
child {node {$1$}
child {node {$0$}
edge from parent
node[above] {0.04}
}
child {node {$1$}
edge from parent
node[above] {0.96}
}
edge from parent
node[above] {0.04}
}
edge from parent
node[above] {0.96}
} ;
\end{tikzpicture}
\item Chaque personne a 2 possibilités (1: fait sonner ou 2: ne fait pas sonner) et l'on fait passer 3 personnes ce qui correspond à une répétition identique et aléatoire. On peut donc modéliser la situation par une loi binomiale.
\[
X \sim \mathcal{B}(3; 0.76)
\]
\item Probabilité qu'une seule personne fasse sonner le portique. On voit qu'il y a 3 branches qui correspondent à cette situation dont
\[
P(X = 1) = 3 \times 0.96^1 \times 0.04^2 \approx 0.005
\]
\item
\[
P(X = 0) = 0.04^3 \approx 0.0
\]
\[
P(X \geq 2) = P(X = 2) + P(X = 3) = 3 \times 0.96^2 \times 0.04^1 + 0.96^3 \approx 0.996
\]
\item Il faut d'abord tracer le tableau résumant la loi de probabilité:
\begin{center}
\begin{tabular}{|c|*{4}{c|}}
\hline
Valeur & 0 & 1 & 2 & 3 \\
\hline
Probabilité & $0.0$ & $0.005$ & $0.111$ &$0.885$ \\
\hline
\end{tabular}
\end{center}
On peut alors calculer l'espérance
\[
E[X] = 0 \times 0.0 + 1 \times 0.005 + 2 \times 0.111 + 3 \times 0.885 = 2.88
\]
On peut donc estimer qu'il y aura en moyenne $2.88$ personnes qui feront sonner le portique sur les 3 personnes.
\end{enumerate}
\end{solution}
\begin{exercise}[subtitle={Équation puissance}]
Résoudre les équations et inéquations suivantes
\begin{multicols}{2}
\begin{enumerate}
\item $10^x = 17$
\item $3^x = 31$
\item $0.44^x \leq 45$
\item $9 \times 1.0^x = 17$
\end{enumerate}
\end{multicols}
\end{exercise}
\begin{solution}
Les solutions ci-dessous ne sont pas justifiée car l'ordinateur ne sait pas faire. Par contre, vous vous devez savoir justifier vos réponses!
\begin{enumerate}
\item $x = \log(17)$
\item $x = \frac{\log(31)}{\log(3)}$
\item Il faut faire attention quand on divise par un log car ce dernier peut être négatif ce qui est le cas ici. Il faut donc pense à changer le sens de l'inégalité.
$x \geq \frac{\log(45)}{\log(0.44)}$
\item Il faut penser à faire la division à par $9$ avant d'utiliser le log car sinon, on ne peut pas utiliser la formule $\log(a^n) = n\times \log(a)$.
$x = \frac{\log(1.89)}{\log(1.0)}$
\end{enumerate}
\end{solution}
\begin{exercise}[subtitle={Étude de fonctions}]
Soit $f(x) = - 8x^3 + 324x^2 + 1392x - 9$ une fonction définie sur $\R$.
\begin{enumerate}
\item Calculer $f'(x)$ la dérivée de $f(x)$.
\item Calculer $f'(29)$ et $f'(-2)$.
\item En déduire une forme factorisée de $f'(x)$.
\item Étudier le signe de $f'(x)$ et en déduire les variations de $f(x)$.
\item Est-ce que la fonction $f(x)$ admet un maximum ou un minimum? Si oui, calculer sa valeur.
\end{enumerate}
\end{exercise}
\begin{solution}
\begin{enumerate}
\item Dérivée de $f(x)$: $f'(x) = - 24x^2 + 648x + 1392$
\item
\begin{align*}
f'(29) &= - 24 \times 29^{2} + 648 \times 29 + 1392\\&= - 24 \times 841 + 18792 + 1392\\&= - 20184 + 20184\\&= 0
\end{align*}
\begin{align*}
f'(-2) &= - 24 \times - 2^{2} + 648(- 2) + 1392\\&= - 24 \times 4 - 1296 + 1392\\&= - 96 + 96\\&= 0
\end{align*}
Donc $x = 29$ et $x=-2$ sont des racines de $f'(x) = - 24x^2 + 648x + 1392$.
\item On en déduit la forme factorisée suivante
\[
f'(x) = -24 (x - 29)(x--2)
\]
\item Pas de correction disponible
\item À causes des branches extérieurs, la fonction $f(x)$ n'a pas de maximum ou de minimum.
\end{enumerate}
\end{solution}
%\printsolutionstype{exercise}
\end{document}
%%% Local Variables:
%%% mode: latex
%%% TeX-master: "master"
%%% End:

View File

@ -0,0 +1,203 @@
\documentclass[a5paper,10pt]{article}
\usepackage{myXsim}
\usepackage{tasks}
% Title Page
\title{DM2 \hfill PERES RAMALHO Emeric}
\tribe{TST}
\date{\hfillÀ render pour le Mercredi 24 février}
\xsimsetup{
solution/print = false
}
\begin{document}
\maketitle
\begin{exercise}[subtitle={Loi binomiale}]
Trois personnes s'apprêtent à passer le portique de sécurité. On suppose que pour chaque personne la probabilité que le portique sonne est égale à $0.19$.
Soit $X$ la variable aléatoire donnant le nombre de personnes faisant sonner le portique, parmi les 3 personnes de ce groupe.
\begin{enumerate}
\item Tracer l'arbre représentant le situation.
\item Justifier que $X$ suit une loi binomiale dont on précisera les paramètres.
\item Quelle est la probabilité qu'une seule personne fasse sonner le portique?
\item Calculer puis interpréter les probabilités suivantes
\[
P(X = 0) \qquad \qquad P(X \geq 2)
\]
\item Calculer l'espérance de $X$ et interpréter le résultat.
\end{enumerate}
\end{exercise}
\begin{solution}
\begin{enumerate}
\item
\begin{tikzpicture}[sloped]
\node {.}
child {node {$0$}
child {node {$0$}
child {node {$0$}
edge from parent
node[above] {0.81}
}
child {node {$1$}
edge from parent
node[above] {0.19}
}
edge from parent
node[above] {0.81}
}
child[missing] {}
child {node {$1$}
child {node {$0$}
edge from parent
node[above] {0.81}
}
child {node {$1$}
edge from parent
node[above] {0.19}
}
edge from parent
node[above] {0.81}
}
edge from parent
node[above] {0.81}
}
child[missing] {}
child[missing] {}
child[missing] {}
child { node {$1$}
child {node {$0$}
child {node {$0$}
edge from parent
node[above] {0.81}
}
child {node {$1$}
edge from parent
node[above] {0.19}
}
edge from parent
node[above] {0.81}
}
child[missing] {}
child {node {$1$}
child {node {$0$}
edge from parent
node[above] {0.81}
}
child {node {$1$}
edge from parent
node[above] {0.19}
}
edge from parent
node[above] {0.81}
}
edge from parent
node[above] {0.19}
} ;
\end{tikzpicture}
\item Chaque personne a 2 possibilités (1: fait sonner ou 2: ne fait pas sonner) et l'on fait passer 3 personnes ce qui correspond à une répétition identique et aléatoire. On peut donc modéliser la situation par une loi binomiale.
\[
X \sim \mathcal{B}(3; 0.76)
\]
\item Probabilité qu'une seule personne fasse sonner le portique. On voit qu'il y a 3 branches qui correspondent à cette situation dont
\[
P(X = 1) = 3 \times 0.19^1 \times 0.81^2 \approx 0.374
\]
\item
\[
P(X = 0) = 0.81^3 \approx 0.531
\]
\[
P(X \geq 2) = P(X = 2) + P(X = 3) = 3 \times 0.19^2 \times 0.81^1 + 0.19^3 \approx 0.095
\]
\item Il faut d'abord tracer le tableau résumant la loi de probabilité:
\begin{center}
\begin{tabular}{|c|*{4}{c|}}
\hline
Valeur & 0 & 1 & 2 & 3 \\
\hline
Probabilité & $0.531$ & $0.374$ & $0.088$ &$0.007$ \\
\hline
\end{tabular}
\end{center}
On peut alors calculer l'espérance
\[
E[X] = 0 \times 0.531 + 1 \times 0.374 + 2 \times 0.088 + 3 \times 0.007 = 0.57
\]
On peut donc estimer qu'il y aura en moyenne $0.57$ personnes qui feront sonner le portique sur les 3 personnes.
\end{enumerate}
\end{solution}
\begin{exercise}[subtitle={Équation puissance}]
Résoudre les équations et inéquations suivantes
\begin{multicols}{2}
\begin{enumerate}
\item $10^x = 5$
\item $4^x = 39$
\item $0.95^x \leq 21$
\item $10 \times 0.74^x = 16$
\end{enumerate}
\end{multicols}
\end{exercise}
\begin{solution}
Les solutions ci-dessous ne sont pas justifiée car l'ordinateur ne sait pas faire. Par contre, vous vous devez savoir justifier vos réponses!
\begin{enumerate}
\item $x = \log(5)$
\item $x = \frac{\log(39)}{\log(4)}$
\item Il faut faire attention quand on divise par un log car ce dernier peut être négatif ce qui est le cas ici. Il faut donc pense à changer le sens de l'inégalité.
$x \geq \frac{\log(21)}{\log(0.95)}$
\item Il faut penser à faire la division à par $10$ avant d'utiliser le log car sinon, on ne peut pas utiliser la formule $\log(a^n) = n\times \log(a)$.
$x = \frac{\log(1.6)}{\log(0.74)}$
\end{enumerate}
\end{solution}
\begin{exercise}[subtitle={Étude de fonctions}]
Soit $f(x) = - 3x^3 + 108x^2 - 396x - 26$ une fonction définie sur $\R$.
\begin{enumerate}
\item Calculer $f'(x)$ la dérivée de $f(x)$.
\item Calculer $f'(22)$ et $f'(2)$.
\item En déduire une forme factorisée de $f'(x)$.
\item Étudier le signe de $f'(x)$ et en déduire les variations de $f(x)$.
\item Est-ce que la fonction $f(x)$ admet un maximum ou un minimum? Si oui, calculer sa valeur.
\end{enumerate}
\end{exercise}
\begin{solution}
\begin{enumerate}
\item Dérivée de $f(x)$: $f'(x) = - 9x^2 + 216x - 396$
\item
\begin{align*}
f'(22) &= - 9 \times 22^{2} + 216 \times 22 - 396\\&= - 9 \times 484 + 4752 - 396\\&= - 4356 + 4356\\&= 0
\end{align*}
\begin{align*}
f'(2) &= - 9 \times 2^{2} + 216 \times 2 - 396\\&= - 9 \times 4 + 432 - 396\\&= - 36 + 36\\&= 0
\end{align*}
Donc $x = 22$ et $x=2$ sont des racines de $f'(x) = - 9x^2 + 216x - 396$.
\item On en déduit la forme factorisée suivante
\[
f'(x) = -9 (x - 22)(x-2)
\]
\item Pas de correction disponible
\item À causes des branches extérieurs, la fonction $f(x)$ n'a pas de maximum ou de minimum.
\end{enumerate}
\end{solution}
%\printsolutionstype{exercise}
\end{document}
%%% Local Variables:
%%% mode: latex
%%% TeX-master: "master"
%%% End:

View File

@ -0,0 +1,203 @@
\documentclass[a5paper,10pt]{article}
\usepackage{myXsim}
\usepackage{tasks}
% Title Page
\title{DM2 \hfill RADOUAA Saleh}
\tribe{TST}
\date{\hfillÀ render pour le Mercredi 24 février}
\xsimsetup{
solution/print = false
}
\begin{document}
\maketitle
\begin{exercise}[subtitle={Loi binomiale}]
Trois personnes s'apprêtent à passer le portique de sécurité. On suppose que pour chaque personne la probabilité que le portique sonne est égale à $0.08$.
Soit $X$ la variable aléatoire donnant le nombre de personnes faisant sonner le portique, parmi les 3 personnes de ce groupe.
\begin{enumerate}
\item Tracer l'arbre représentant le situation.
\item Justifier que $X$ suit une loi binomiale dont on précisera les paramètres.
\item Quelle est la probabilité qu'une seule personne fasse sonner le portique?
\item Calculer puis interpréter les probabilités suivantes
\[
P(X = 0) \qquad \qquad P(X \geq 2)
\]
\item Calculer l'espérance de $X$ et interpréter le résultat.
\end{enumerate}
\end{exercise}
\begin{solution}
\begin{enumerate}
\item
\begin{tikzpicture}[sloped]
\node {.}
child {node {$0$}
child {node {$0$}
child {node {$0$}
edge from parent
node[above] {0.92}
}
child {node {$1$}
edge from parent
node[above] {0.08}
}
edge from parent
node[above] {0.92}
}
child[missing] {}
child {node {$1$}
child {node {$0$}
edge from parent
node[above] {0.92}
}
child {node {$1$}
edge from parent
node[above] {0.08}
}
edge from parent
node[above] {0.92}
}
edge from parent
node[above] {0.92}
}
child[missing] {}
child[missing] {}
child[missing] {}
child { node {$1$}
child {node {$0$}
child {node {$0$}
edge from parent
node[above] {0.92}
}
child {node {$1$}
edge from parent
node[above] {0.08}
}
edge from parent
node[above] {0.92}
}
child[missing] {}
child {node {$1$}
child {node {$0$}
edge from parent
node[above] {0.92}
}
child {node {$1$}
edge from parent
node[above] {0.08}
}
edge from parent
node[above] {0.92}
}
edge from parent
node[above] {0.08}
} ;
\end{tikzpicture}
\item Chaque personne a 2 possibilités (1: fait sonner ou 2: ne fait pas sonner) et l'on fait passer 3 personnes ce qui correspond à une répétition identique et aléatoire. On peut donc modéliser la situation par une loi binomiale.
\[
X \sim \mathcal{B}(3; 0.76)
\]
\item Probabilité qu'une seule personne fasse sonner le portique. On voit qu'il y a 3 branches qui correspondent à cette situation dont
\[
P(X = 1) = 3 \times 0.08^1 \times 0.92^2 \approx 0.203
\]
\item
\[
P(X = 0) = 0.92^3 \approx 0.779
\]
\[
P(X \geq 2) = P(X = 2) + P(X = 3) = 3 \times 0.08^2 \times 0.92^1 + 0.08^3 \approx 0.019
\]
\item Il faut d'abord tracer le tableau résumant la loi de probabilité:
\begin{center}
\begin{tabular}{|c|*{4}{c|}}
\hline
Valeur & 0 & 1 & 2 & 3 \\
\hline
Probabilité & $0.779$ & $0.203$ & $0.018$ &$0.001$ \\
\hline
\end{tabular}
\end{center}
On peut alors calculer l'espérance
\[
E[X] = 0 \times 0.779 + 1 \times 0.203 + 2 \times 0.018 + 3 \times 0.001 = 0.24
\]
On peut donc estimer qu'il y aura en moyenne $0.24$ personnes qui feront sonner le portique sur les 3 personnes.
\end{enumerate}
\end{solution}
\begin{exercise}[subtitle={Équation puissance}]
Résoudre les équations et inéquations suivantes
\begin{multicols}{2}
\begin{enumerate}
\item $10^x = 8$
\item $17^x = 11$
\item $0.84^x \leq 28$
\item $8 \times 0.96^x = 22$
\end{enumerate}
\end{multicols}
\end{exercise}
\begin{solution}
Les solutions ci-dessous ne sont pas justifiée car l'ordinateur ne sait pas faire. Par contre, vous vous devez savoir justifier vos réponses!
\begin{enumerate}
\item $x = \log(8)$
\item $x = \frac{\log(11)}{\log(17)}$
\item Il faut faire attention quand on divise par un log car ce dernier peut être négatif ce qui est le cas ici. Il faut donc pense à changer le sens de l'inégalité.
$x \geq \frac{\log(28)}{\log(0.84)}$
\item Il faut penser à faire la division à par $8$ avant d'utiliser le log car sinon, on ne peut pas utiliser la formule $\log(a^n) = n\times \log(a)$.
$x = \frac{\log(2.75)}{\log(0.96)}$
\end{enumerate}
\end{solution}
\begin{exercise}[subtitle={Étude de fonctions}]
Soit $f(x) = 3x^3 - 207x^2 + 1161x + 46$ une fonction définie sur $\R$.
\begin{enumerate}
\item Calculer $f'(x)$ la dérivée de $f(x)$.
\item Calculer $f'(43)$ et $f'(3)$.
\item En déduire une forme factorisée de $f'(x)$.
\item Étudier le signe de $f'(x)$ et en déduire les variations de $f(x)$.
\item Est-ce que la fonction $f(x)$ admet un maximum ou un minimum? Si oui, calculer sa valeur.
\end{enumerate}
\end{exercise}
\begin{solution}
\begin{enumerate}
\item Dérivée de $f(x)$: $f'(x) = 9x^2 - 414x + 1161$
\item
\begin{align*}
f'(43) &= 9 \times 43^{2} - 414 \times 43 + 1161\\&= 9 \times 1849 - 17802 + 1161\\&= 16641 - 16641\\&= 0
\end{align*}
\begin{align*}
f'(3) &= 9 \times 3^{2} - 414 \times 3 + 1161\\&= 9 \times 9 - 1242 + 1161\\&= 81 - 81\\&= 0
\end{align*}
Donc $x = 43$ et $x=3$ sont des racines de $f'(x) = 9x^2 - 414x + 1161$.
\item On en déduit la forme factorisée suivante
\[
f'(x) = 9 (x - 43)(x-3)
\]
\item Pas de correction disponible
\item À causes des branches extérieurs, la fonction $f(x)$ n'a pas de maximum ou de minimum.
\end{enumerate}
\end{solution}
%\printsolutionstype{exercise}
\end{document}
%%% Local Variables:
%%% mode: latex
%%% TeX-master: "master"
%%% End:

View File

@ -0,0 +1,203 @@
\documentclass[a5paper,10pt]{article}
\usepackage{myXsim}
\usepackage{tasks}
% Title Page
\title{DM2 \hfill TAY Ummuhan}
\tribe{TST}
\date{\hfillÀ render pour le Mercredi 24 février}
\xsimsetup{
solution/print = false
}
\begin{document}
\maketitle
\begin{exercise}[subtitle={Loi binomiale}]
Trois personnes s'apprêtent à passer le portique de sécurité. On suppose que pour chaque personne la probabilité que le portique sonne est égale à $0.72$.
Soit $X$ la variable aléatoire donnant le nombre de personnes faisant sonner le portique, parmi les 3 personnes de ce groupe.
\begin{enumerate}
\item Tracer l'arbre représentant le situation.
\item Justifier que $X$ suit une loi binomiale dont on précisera les paramètres.
\item Quelle est la probabilité qu'une seule personne fasse sonner le portique?
\item Calculer puis interpréter les probabilités suivantes
\[
P(X = 0) \qquad \qquad P(X \geq 2)
\]
\item Calculer l'espérance de $X$ et interpréter le résultat.
\end{enumerate}
\end{exercise}
\begin{solution}
\begin{enumerate}
\item
\begin{tikzpicture}[sloped]
\node {.}
child {node {$0$}
child {node {$0$}
child {node {$0$}
edge from parent
node[above] {0.28}
}
child {node {$1$}
edge from parent
node[above] {0.72}
}
edge from parent
node[above] {0.28}
}
child[missing] {}
child {node {$1$}
child {node {$0$}
edge from parent
node[above] {0.28}
}
child {node {$1$}
edge from parent
node[above] {0.72}
}
edge from parent
node[above] {0.28}
}
edge from parent
node[above] {0.28}
}
child[missing] {}
child[missing] {}
child[missing] {}
child { node {$1$}
child {node {$0$}
child {node {$0$}
edge from parent
node[above] {0.28}
}
child {node {$1$}
edge from parent
node[above] {0.72}
}
edge from parent
node[above] {0.28}
}
child[missing] {}
child {node {$1$}
child {node {$0$}
edge from parent
node[above] {0.28}
}
child {node {$1$}
edge from parent
node[above] {0.72}
}
edge from parent
node[above] {0.28}
}
edge from parent
node[above] {0.72}
} ;
\end{tikzpicture}
\item Chaque personne a 2 possibilités (1: fait sonner ou 2: ne fait pas sonner) et l'on fait passer 3 personnes ce qui correspond à une répétition identique et aléatoire. On peut donc modéliser la situation par une loi binomiale.
\[
X \sim \mathcal{B}(3; 0.76)
\]
\item Probabilité qu'une seule personne fasse sonner le portique. On voit qu'il y a 3 branches qui correspondent à cette situation dont
\[
P(X = 1) = 3 \times 0.72^1 \times 0.28^2 \approx 0.169
\]
\item
\[
P(X = 0) = 0.28^3 \approx 0.022
\]
\[
P(X \geq 2) = P(X = 2) + P(X = 3) = 3 \times 0.72^2 \times 0.28^1 + 0.72^3 \approx 0.808
\]
\item Il faut d'abord tracer le tableau résumant la loi de probabilité:
\begin{center}
\begin{tabular}{|c|*{4}{c|}}
\hline
Valeur & 0 & 1 & 2 & 3 \\
\hline
Probabilité & $0.022$ & $0.169$ & $0.435$ &$0.373$ \\
\hline
\end{tabular}
\end{center}
On peut alors calculer l'espérance
\[
E[X] = 0 \times 0.022 + 1 \times 0.169 + 2 \times 0.435 + 3 \times 0.373 = 2.16
\]
On peut donc estimer qu'il y aura en moyenne $2.16$ personnes qui feront sonner le portique sur les 3 personnes.
\end{enumerate}
\end{solution}
\begin{exercise}[subtitle={Équation puissance}]
Résoudre les équations et inéquations suivantes
\begin{multicols}{2}
\begin{enumerate}
\item $10^x = 41$
\item $2^x = 7$
\item $0.44^x \leq 20$
\item $3 \times 0.08^x = 24$
\end{enumerate}
\end{multicols}
\end{exercise}
\begin{solution}
Les solutions ci-dessous ne sont pas justifiée car l'ordinateur ne sait pas faire. Par contre, vous vous devez savoir justifier vos réponses!
\begin{enumerate}
\item $x = \log(41)$
\item $x = \frac{\log(7)}{\log(2)}$
\item Il faut faire attention quand on divise par un log car ce dernier peut être négatif ce qui est le cas ici. Il faut donc pense à changer le sens de l'inégalité.
$x \geq \frac{\log(20)}{\log(0.44)}$
\item Il faut penser à faire la division à par $3$ avant d'utiliser le log car sinon, on ne peut pas utiliser la formule $\log(a^n) = n\times \log(a)$.
$x = \frac{\log(8.0)}{\log(0.08)}$
\end{enumerate}
\end{solution}
\begin{exercise}[subtitle={Étude de fonctions}]
Soit $f(x) = 7x^3 - 472.5x^2 + 10374x + 15$ une fonction définie sur $\R$.
\begin{enumerate}
\item Calculer $f'(x)$ la dérivée de $f(x)$.
\item Calculer $f'(26)$ et $f'(19)$.
\item En déduire une forme factorisée de $f'(x)$.
\item Étudier le signe de $f'(x)$ et en déduire les variations de $f(x)$.
\item Est-ce que la fonction $f(x)$ admet un maximum ou un minimum? Si oui, calculer sa valeur.
\end{enumerate}
\end{exercise}
\begin{solution}
\begin{enumerate}
\item Dérivée de $f(x)$: $f'(x) = 21x^2 - 945x + 10374$
\item
\begin{align*}
f'(26) &= 21 \times 26^{2} - 945 \times 26 + 10374\\&= 21 \times 676 - 24570 + 10374\\&= 14196 - 14196\\&= 0
\end{align*}
\begin{align*}
f'(19) &= 21 \times 19^{2} - 945 \times 19 + 10374\\&= 21 \times 361 - 17955 + 10374\\&= 7581 - 7581\\&= 0
\end{align*}
Donc $x = 26$ et $x=19$ sont des racines de $f'(x) = 21x^2 - 945x + 10374$.
\item On en déduit la forme factorisée suivante
\[
f'(x) = 21 (x - 26)(x-19)
\]
\item Pas de correction disponible
\item À causes des branches extérieurs, la fonction $f(x)$ n'a pas de maximum ou de minimum.
\end{enumerate}
\end{solution}
%\printsolutionstype{exercise}
\end{document}
%%% Local Variables:
%%% mode: latex
%%% TeX-master: "master"
%%% End:

View File

@ -0,0 +1,203 @@
\documentclass[a5paper,10pt]{article}
\usepackage{myXsim}
\usepackage{tasks}
% Title Page
\title{DM2 \hfill VIALON-DUPERRON Victorien}
\tribe{TST}
\date{\hfillÀ render pour le Mercredi 24 février}
\xsimsetup{
solution/print = false
}
\begin{document}
\maketitle
\begin{exercise}[subtitle={Loi binomiale}]
Trois personnes s'apprêtent à passer le portique de sécurité. On suppose que pour chaque personne la probabilité que le portique sonne est égale à $0.18$.
Soit $X$ la variable aléatoire donnant le nombre de personnes faisant sonner le portique, parmi les 3 personnes de ce groupe.
\begin{enumerate}
\item Tracer l'arbre représentant le situation.
\item Justifier que $X$ suit une loi binomiale dont on précisera les paramètres.
\item Quelle est la probabilité qu'une seule personne fasse sonner le portique?
\item Calculer puis interpréter les probabilités suivantes
\[
P(X = 0) \qquad \qquad P(X \geq 2)
\]
\item Calculer l'espérance de $X$ et interpréter le résultat.
\end{enumerate}
\end{exercise}
\begin{solution}
\begin{enumerate}
\item
\begin{tikzpicture}[sloped]
\node {.}
child {node {$0$}
child {node {$0$}
child {node {$0$}
edge from parent
node[above] {0.82}
}
child {node {$1$}
edge from parent
node[above] {0.18}
}
edge from parent
node[above] {0.82}
}
child[missing] {}
child {node {$1$}
child {node {$0$}
edge from parent
node[above] {0.82}
}
child {node {$1$}
edge from parent
node[above] {0.18}
}
edge from parent
node[above] {0.82}
}
edge from parent
node[above] {0.82}
}
child[missing] {}
child[missing] {}
child[missing] {}
child { node {$1$}
child {node {$0$}
child {node {$0$}
edge from parent
node[above] {0.82}
}
child {node {$1$}
edge from parent
node[above] {0.18}
}
edge from parent
node[above] {0.82}
}
child[missing] {}
child {node {$1$}
child {node {$0$}
edge from parent
node[above] {0.82}
}
child {node {$1$}
edge from parent
node[above] {0.18}
}
edge from parent
node[above] {0.82}
}
edge from parent
node[above] {0.18}
} ;
\end{tikzpicture}
\item Chaque personne a 2 possibilités (1: fait sonner ou 2: ne fait pas sonner) et l'on fait passer 3 personnes ce qui correspond à une répétition identique et aléatoire. On peut donc modéliser la situation par une loi binomiale.
\[
X \sim \mathcal{B}(3; 0.76)
\]
\item Probabilité qu'une seule personne fasse sonner le portique. On voit qu'il y a 3 branches qui correspondent à cette situation dont
\[
P(X = 1) = 3 \times 0.18^1 \times 0.82^2 \approx 0.363
\]
\item
\[
P(X = 0) = 0.82^3 \approx 0.551
\]
\[
P(X \geq 2) = P(X = 2) + P(X = 3) = 3 \times 0.18^2 \times 0.82^1 + 0.18^3 \approx 0.086
\]
\item Il faut d'abord tracer le tableau résumant la loi de probabilité:
\begin{center}
\begin{tabular}{|c|*{4}{c|}}
\hline
Valeur & 0 & 1 & 2 & 3 \\
\hline
Probabilité & $0.551$ & $0.363$ & $0.08$ &$0.006$ \\
\hline
\end{tabular}
\end{center}
On peut alors calculer l'espérance
\[
E[X] = 0 \times 0.551 + 1 \times 0.363 + 2 \times 0.08 + 3 \times 0.006 = 0.54
\]
On peut donc estimer qu'il y aura en moyenne $0.54$ personnes qui feront sonner le portique sur les 3 personnes.
\end{enumerate}
\end{solution}
\begin{exercise}[subtitle={Équation puissance}]
Résoudre les équations et inéquations suivantes
\begin{multicols}{2}
\begin{enumerate}
\item $10^x = 22$
\item $4^x = 6$
\item $0.01^x \leq 35$
\item $10 \times 0.36^x = 19$
\end{enumerate}
\end{multicols}
\end{exercise}
\begin{solution}
Les solutions ci-dessous ne sont pas justifiée car l'ordinateur ne sait pas faire. Par contre, vous vous devez savoir justifier vos réponses!
\begin{enumerate}
\item $x = \log(22)$
\item $x = \frac{\log(6)}{\log(4)}$
\item Il faut faire attention quand on divise par un log car ce dernier peut être négatif ce qui est le cas ici. Il faut donc pense à changer le sens de l'inégalité.
$x \geq \frac{\log(35)}{\log(0.01)}$
\item Il faut penser à faire la division à par $10$ avant d'utiliser le log car sinon, on ne peut pas utiliser la formule $\log(a^n) = n\times \log(a)$.
$x = \frac{\log(1.9)}{\log(0.36)}$
\end{enumerate}
\end{solution}
\begin{exercise}[subtitle={Étude de fonctions}]
Soit $f(x) = - 4x^3 + 72x^2 + 1296x - 22$ une fonction définie sur $\R$.
\begin{enumerate}
\item Calculer $f'(x)$ la dérivée de $f(x)$.
\item Calculer $f'(18)$ et $f'(-6)$.
\item En déduire une forme factorisée de $f'(x)$.
\item Étudier le signe de $f'(x)$ et en déduire les variations de $f(x)$.
\item Est-ce que la fonction $f(x)$ admet un maximum ou un minimum? Si oui, calculer sa valeur.
\end{enumerate}
\end{exercise}
\begin{solution}
\begin{enumerate}
\item Dérivée de $f(x)$: $f'(x) = - 12x^2 + 144x + 1296$
\item
\begin{align*}
f'(18) &= - 12 \times 18^{2} + 144 \times 18 + 1296\\&= - 12 \times 324 + 2592 + 1296\\&= - 3888 + 3888\\&= 0
\end{align*}
\begin{align*}
f'(-6) &= - 12 \times - 6^{2} + 144(- 6) + 1296\\&= - 12 \times 36 - 864 + 1296\\&= - 432 + 432\\&= 0
\end{align*}
Donc $x = 18$ et $x=-6$ sont des racines de $f'(x) = - 12x^2 + 144x + 1296$.
\item On en déduit la forme factorisée suivante
\[
f'(x) = -12 (x - 18)(x--6)
\]
\item Pas de correction disponible
\item À causes des branches extérieurs, la fonction $f(x)$ n'a pas de maximum ou de minimum.
\end{enumerate}
\end{solution}
%\printsolutionstype{exercise}
\end{document}
%%% Local Variables:
%%% mode: latex
%%% TeX-master: "master"
%%% End:

View File

@ -0,0 +1,203 @@
\documentclass[a5paper,10pt]{article}
\usepackage{myXsim}
\usepackage{tasks}
% Title Page
\title{DM2 \hfill ZENAGUI Yanis}
\tribe{TST}
\date{\hfillÀ render pour le Mercredi 24 février}
\xsimsetup{
solution/print = false
}
\begin{document}
\maketitle
\begin{exercise}[subtitle={Loi binomiale}]
Trois personnes s'apprêtent à passer le portique de sécurité. On suppose que pour chaque personne la probabilité que le portique sonne est égale à $0.23$.
Soit $X$ la variable aléatoire donnant le nombre de personnes faisant sonner le portique, parmi les 3 personnes de ce groupe.
\begin{enumerate}
\item Tracer l'arbre représentant le situation.
\item Justifier que $X$ suit une loi binomiale dont on précisera les paramètres.
\item Quelle est la probabilité qu'une seule personne fasse sonner le portique?
\item Calculer puis interpréter les probabilités suivantes
\[
P(X = 0) \qquad \qquad P(X \geq 2)
\]
\item Calculer l'espérance de $X$ et interpréter le résultat.
\end{enumerate}
\end{exercise}
\begin{solution}
\begin{enumerate}
\item
\begin{tikzpicture}[sloped]
\node {.}
child {node {$0$}
child {node {$0$}
child {node {$0$}
edge from parent
node[above] {0.77}
}
child {node {$1$}
edge from parent
node[above] {0.23}
}
edge from parent
node[above] {0.77}
}
child[missing] {}
child {node {$1$}
child {node {$0$}
edge from parent
node[above] {0.77}
}
child {node {$1$}
edge from parent
node[above] {0.23}
}
edge from parent
node[above] {0.77}
}
edge from parent
node[above] {0.77}
}
child[missing] {}
child[missing] {}
child[missing] {}
child { node {$1$}
child {node {$0$}
child {node {$0$}
edge from parent
node[above] {0.77}
}
child {node {$1$}
edge from parent
node[above] {0.23}
}
edge from parent
node[above] {0.77}
}
child[missing] {}
child {node {$1$}
child {node {$0$}
edge from parent
node[above] {0.77}
}
child {node {$1$}
edge from parent
node[above] {0.23}
}
edge from parent
node[above] {0.77}
}
edge from parent
node[above] {0.23}
} ;
\end{tikzpicture}
\item Chaque personne a 2 possibilités (1: fait sonner ou 2: ne fait pas sonner) et l'on fait passer 3 personnes ce qui correspond à une répétition identique et aléatoire. On peut donc modéliser la situation par une loi binomiale.
\[
X \sim \mathcal{B}(3; 0.76)
\]
\item Probabilité qu'une seule personne fasse sonner le portique. On voit qu'il y a 3 branches qui correspondent à cette situation dont
\[
P(X = 1) = 3 \times 0.23^1 \times 0.77^2 \approx 0.409
\]
\item
\[
P(X = 0) = 0.77^3 \approx 0.457
\]
\[
P(X \geq 2) = P(X = 2) + P(X = 3) = 3 \times 0.23^2 \times 0.77^1 + 0.23^3 \approx 0.134
\]
\item Il faut d'abord tracer le tableau résumant la loi de probabilité:
\begin{center}
\begin{tabular}{|c|*{4}{c|}}
\hline
Valeur & 0 & 1 & 2 & 3 \\
\hline
Probabilité & $0.457$ & $0.409$ & $0.122$ &$0.012$ \\
\hline
\end{tabular}
\end{center}
On peut alors calculer l'espérance
\[
E[X] = 0 \times 0.457 + 1 \times 0.409 + 2 \times 0.122 + 3 \times 0.012 = 0.69
\]
On peut donc estimer qu'il y aura en moyenne $0.69$ personnes qui feront sonner le portique sur les 3 personnes.
\end{enumerate}
\end{solution}
\begin{exercise}[subtitle={Équation puissance}]
Résoudre les équations et inéquations suivantes
\begin{multicols}{2}
\begin{enumerate}
\item $10^x = 15$
\item $7^x = 38$
\item $0.66^x \leq 14$
\item $9 \times 0.36^x = 21$
\end{enumerate}
\end{multicols}
\end{exercise}
\begin{solution}
Les solutions ci-dessous ne sont pas justifiée car l'ordinateur ne sait pas faire. Par contre, vous vous devez savoir justifier vos réponses!
\begin{enumerate}
\item $x = \log(15)$
\item $x = \frac{\log(38)}{\log(7)}$
\item Il faut faire attention quand on divise par un log car ce dernier peut être négatif ce qui est le cas ici. Il faut donc pense à changer le sens de l'inégalité.
$x \geq \frac{\log(14)}{\log(0.66)}$
\item Il faut penser à faire la division à par $9$ avant d'utiliser le log car sinon, on ne peut pas utiliser la formule $\log(a^n) = n\times \log(a)$.
$x = \frac{\log(2.33)}{\log(0.36)}$
\end{enumerate}
\end{solution}
\begin{exercise}[subtitle={Étude de fonctions}]
Soit $f(x) = - 5x^3 + 375x^2 - 4515x + 9$ une fonction définie sur $\R$.
\begin{enumerate}
\item Calculer $f'(x)$ la dérivée de $f(x)$.
\item Calculer $f'(43)$ et $f'(7)$.
\item En déduire une forme factorisée de $f'(x)$.
\item Étudier le signe de $f'(x)$ et en déduire les variations de $f(x)$.
\item Est-ce que la fonction $f(x)$ admet un maximum ou un minimum? Si oui, calculer sa valeur.
\end{enumerate}
\end{exercise}
\begin{solution}
\begin{enumerate}
\item Dérivée de $f(x)$: $f'(x) = - 15x^2 + 750x - 4515$
\item
\begin{align*}
f'(43) &= - 15 \times 43^{2} + 750 \times 43 - 4515\\&= - 15 \times 1849 + 32250 - 4515\\&= - 27735 + 27735\\&= 0
\end{align*}
\begin{align*}
f'(7) &= - 15 \times 7^{2} + 750 \times 7 - 4515\\&= - 15 \times 49 + 5250 - 4515\\&= - 735 + 735\\&= 0
\end{align*}
Donc $x = 43$ et $x=7$ sont des racines de $f'(x) = - 15x^2 + 750x - 4515$.
\item On en déduit la forme factorisée suivante
\[
f'(x) = -15 (x - 43)(x-7)
\]
\item Pas de correction disponible
\item À causes des branches extérieurs, la fonction $f(x)$ n'a pas de maximum ou de minimum.
\end{enumerate}
\end{solution}
%\printsolutionstype{exercise}
\end{document}
%%% Local Variables:
%%% mode: latex
%%% TeX-master: "master"
%%% End:

Binary file not shown.

View File

@ -0,0 +1,203 @@
\documentclass[a5paper,10pt]{article}
\usepackage{myXsim}
\usepackage{tasks}
% Title Page
\title{DM2 \hfill ASAIDI Sophian}
\tribe{TST}
\date{\hfillÀ render pour le Mercredi 24 février}
\xsimsetup{
solution/print = true
}
\begin{document}
\maketitle
\begin{exercise}[subtitle={Loi binomiale}]
Trois personnes s'apprêtent à passer le portique de sécurité. On suppose que pour chaque personne la probabilité que le portique sonne est égale à $0.19$.
Soit $X$ la variable aléatoire donnant le nombre de personnes faisant sonner le portique, parmi les 3 personnes de ce groupe.
\begin{enumerate}
\item Tracer l'arbre représentant le situation.
\item Justifier que $X$ suit une loi binomiale dont on précisera les paramètres.
\item Quelle est la probabilité qu'une seule personne fasse sonner le portique?
\item Calculer puis interpréter les probabilités suivantes
\[
P(X = 0) \qquad \qquad P(X \geq 2)
\]
\item Calculer l'espérance de $X$ et interpréter le résultat.
\end{enumerate}
\end{exercise}
\begin{solution}
\begin{enumerate}
\item
\begin{tikzpicture}[sloped]
\node {.}
child {node {$0$}
child {node {$0$}
child {node {$0$}
edge from parent
node[above] {0.81}
}
child {node {$1$}
edge from parent
node[above] {0.19}
}
edge from parent
node[above] {0.81}
}
child[missing] {}
child {node {$1$}
child {node {$0$}
edge from parent
node[above] {0.81}
}
child {node {$1$}
edge from parent
node[above] {0.19}
}
edge from parent
node[above] {0.81}
}
edge from parent
node[above] {0.81}
}
child[missing] {}
child[missing] {}
child[missing] {}
child { node {$1$}
child {node {$0$}
child {node {$0$}
edge from parent
node[above] {0.81}
}
child {node {$1$}
edge from parent
node[above] {0.19}
}
edge from parent
node[above] {0.81}
}
child[missing] {}
child {node {$1$}
child {node {$0$}
edge from parent
node[above] {0.81}
}
child {node {$1$}
edge from parent
node[above] {0.19}
}
edge from parent
node[above] {0.81}
}
edge from parent
node[above] {0.19}
} ;
\end{tikzpicture}
\item Chaque personne a 2 possibilités (1: fait sonner ou 2: ne fait pas sonner) et l'on fait passer 3 personnes ce qui correspond à une répétition identique et aléatoire. On peut donc modéliser la situation par une loi binomiale.
\[
X \sim \mathcal{B}(3; 0.76)
\]
\item Probabilité qu'une seule personne fasse sonner le portique. On voit qu'il y a 3 branches qui correspondent à cette situation dont
\[
P(X = 1) = 3 \times 0.19^1 \times 0.81^2 \approx 0.374
\]
\item
\[
P(X = 0) = 0.81^3 \approx 0.531
\]
\[
P(X \geq 2) = P(X = 2) + P(X = 3) = 3 \times 0.19^2 \times 0.81^1 + 0.19^3 \approx 0.095
\]
\item Il faut d'abord tracer le tableau résumant la loi de probabilité:
\begin{center}
\begin{tabular}{|c|*{4}{c|}}
\hline
Valeur & 0 & 1 & 2 & 3 \\
\hline
Probabilité & $0.531$ & $0.374$ & $0.088$ &$0.007$ \\
\hline
\end{tabular}
\end{center}
On peut alors calculer l'espérance
\[
E[X] = 0 \times 0.531 + 1 \times 0.374 + 2 \times 0.088 + 3 \times 0.007 = 0.57
\]
On peut donc estimer qu'il y aura en moyenne $0.57$ personnes qui feront sonner le portique sur les 3 personnes.
\end{enumerate}
\end{solution}
\begin{exercise}[subtitle={Équation puissance}]
Résoudre les équations et inéquations suivantes
\begin{multicols}{2}
\begin{enumerate}
\item $10^x = 7$
\item $3^x = 35$
\item $0.8^x \leq 2$
\item $4 \times 0.06^x = 49$
\end{enumerate}
\end{multicols}
\end{exercise}
\begin{solution}
Les solutions ci-dessous ne sont pas justifiée car l'ordinateur ne sait pas faire. Par contre, vous vous devez savoir justifier vos réponses!
\begin{enumerate}
\item $x = \log(7)$
\item $x = \frac{\log(35)}{\log(3)}$
\item Il faut faire attention quand on divise par un log car ce dernier peut être négatif ce qui est le cas ici. Il faut donc pense à changer le sens de l'inégalité.
$x \geq \frac{\log(2)}{\log(0.8)}$
\item Il faut penser à faire la division à par $4$ avant d'utiliser le log car sinon, on ne peut pas utiliser la formule $\log(a^n) = n\times \log(a)$.
$x = \frac{\log(12.25)}{\log(0.06)}$
\end{enumerate}
\end{solution}
\begin{exercise}[subtitle={Étude de fonctions}]
Soit $f(x) = 5x^3 - 202.5x^2 - 2970x + 24$ une fonction définie sur $\R$.
\begin{enumerate}
\item Calculer $f'(x)$ la dérivée de $f(x)$.
\item Calculer $f'(33)$ et $f'(-6)$.
\item En déduire une forme factorisée de $f'(x)$.
\item Étudier le signe de $f'(x)$ et en déduire les variations de $f(x)$.
\item Est-ce que la fonction $f(x)$ admet un maximum ou un minimum? Si oui, calculer sa valeur.
\end{enumerate}
\end{exercise}
\begin{solution}
\begin{enumerate}
\item Dérivée de $f(x)$: $f'(x) = 15x^2 - 405x - 2970$
\item
\begin{align*}
f'(33) &= 15 \times 33^{2} - 405 \times 33 - 2970\\&= 15 \times 1089 - 13365 - 2970\\&= 16335 - 16335\\&= 0
\end{align*}
\begin{align*}
f'(-6) &= 15 \times - 6^{2} - 405(- 6) - 2970\\&= 15 \times 36 + 2430 - 2970\\&= 540 - 540\\&= 0
\end{align*}
Donc $x = 33$ et $x=-6$ sont des racines de $f'(x) = 15x^2 - 405x - 2970$.
\item On en déduit la forme factorisée suivante
\[
f'(x) = 15 (x - 33)(x--6)
\]
\item Pas de correction disponible
\item À causes des branches extérieurs, la fonction $f(x)$ n'a pas de maximum ou de minimum.
\end{enumerate}
\end{solution}
%\printsolutionstype{exercise}
\end{document}
%%% Local Variables:
%%% mode: latex
%%% TeX-master: "master"
%%% End:

View File

@ -0,0 +1,203 @@
\documentclass[a5paper,10pt]{article}
\usepackage{myXsim}
\usepackage{tasks}
% Title Page
\title{DM2 \hfill BELARBI Samira}
\tribe{TST}
\date{\hfillÀ render pour le Mercredi 24 février}
\xsimsetup{
solution/print = true
}
\begin{document}
\maketitle
\begin{exercise}[subtitle={Loi binomiale}]
Trois personnes s'apprêtent à passer le portique de sécurité. On suppose que pour chaque personne la probabilité que le portique sonne est égale à $0.58$.
Soit $X$ la variable aléatoire donnant le nombre de personnes faisant sonner le portique, parmi les 3 personnes de ce groupe.
\begin{enumerate}
\item Tracer l'arbre représentant le situation.
\item Justifier que $X$ suit une loi binomiale dont on précisera les paramètres.
\item Quelle est la probabilité qu'une seule personne fasse sonner le portique?
\item Calculer puis interpréter les probabilités suivantes
\[
P(X = 0) \qquad \qquad P(X \geq 2)
\]
\item Calculer l'espérance de $X$ et interpréter le résultat.
\end{enumerate}
\end{exercise}
\begin{solution}
\begin{enumerate}
\item
\begin{tikzpicture}[sloped]
\node {.}
child {node {$0$}
child {node {$0$}
child {node {$0$}
edge from parent
node[above] {0.42}
}
child {node {$1$}
edge from parent
node[above] {0.58}
}
edge from parent
node[above] {0.42}
}
child[missing] {}
child {node {$1$}
child {node {$0$}
edge from parent
node[above] {0.42}
}
child {node {$1$}
edge from parent
node[above] {0.58}
}
edge from parent
node[above] {0.42}
}
edge from parent
node[above] {0.42}
}
child[missing] {}
child[missing] {}
child[missing] {}
child { node {$1$}
child {node {$0$}
child {node {$0$}
edge from parent
node[above] {0.42}
}
child {node {$1$}
edge from parent
node[above] {0.58}
}
edge from parent
node[above] {0.42}
}
child[missing] {}
child {node {$1$}
child {node {$0$}
edge from parent
node[above] {0.42}
}
child {node {$1$}
edge from parent
node[above] {0.58}
}
edge from parent
node[above] {0.42}
}
edge from parent
node[above] {0.58}
} ;
\end{tikzpicture}
\item Chaque personne a 2 possibilités (1: fait sonner ou 2: ne fait pas sonner) et l'on fait passer 3 personnes ce qui correspond à une répétition identique et aléatoire. On peut donc modéliser la situation par une loi binomiale.
\[
X \sim \mathcal{B}(3; 0.76)
\]
\item Probabilité qu'une seule personne fasse sonner le portique. On voit qu'il y a 3 branches qui correspondent à cette situation dont
\[
P(X = 1) = 3 \times 0.58^1 \times 0.42^2 \approx 0.307
\]
\item
\[
P(X = 0) = 0.42^3 \approx 0.074
\]
\[
P(X \geq 2) = P(X = 2) + P(X = 3) = 3 \times 0.58^2 \times 0.42^1 + 0.58^3 \approx 0.619
\]
\item Il faut d'abord tracer le tableau résumant la loi de probabilité:
\begin{center}
\begin{tabular}{|c|*{4}{c|}}
\hline
Valeur & 0 & 1 & 2 & 3 \\
\hline
Probabilité & $0.074$ & $0.307$ & $0.424$ &$0.195$ \\
\hline
\end{tabular}
\end{center}
On peut alors calculer l'espérance
\[
E[X] = 0 \times 0.074 + 1 \times 0.307 + 2 \times 0.424 + 3 \times 0.195 = 1.74
\]
On peut donc estimer qu'il y aura en moyenne $1.74$ personnes qui feront sonner le portique sur les 3 personnes.
\end{enumerate}
\end{solution}
\begin{exercise}[subtitle={Équation puissance}]
Résoudre les équations et inéquations suivantes
\begin{multicols}{2}
\begin{enumerate}
\item $10^x = 6$
\item $10^x = 31$
\item $0.32^x \leq 15$
\item $5 \times 0.06^x = 9$
\end{enumerate}
\end{multicols}
\end{exercise}
\begin{solution}
Les solutions ci-dessous ne sont pas justifiée car l'ordinateur ne sait pas faire. Par contre, vous vous devez savoir justifier vos réponses!
\begin{enumerate}
\item $x = \log(6)$
\item $x = \frac{\log(31)}{\log(10)}$
\item Il faut faire attention quand on divise par un log car ce dernier peut être négatif ce qui est le cas ici. Il faut donc pense à changer le sens de l'inégalité.
$x \geq \frac{\log(15)}{\log(0.32)}$
\item Il faut penser à faire la division à par $5$ avant d'utiliser le log car sinon, on ne peut pas utiliser la formule $\log(a^n) = n\times \log(a)$.
$x = \frac{\log(1.8)}{\log(0.06)}$
\end{enumerate}
\end{solution}
\begin{exercise}[subtitle={Étude de fonctions}]
Soit $f(x) = - 4x^3 + 126x^2 + 2784x - 17$ une fonction définie sur $\R$.
\begin{enumerate}
\item Calculer $f'(x)$ la dérivée de $f(x)$.
\item Calculer $f'(29)$ et $f'(-8)$.
\item En déduire une forme factorisée de $f'(x)$.
\item Étudier le signe de $f'(x)$ et en déduire les variations de $f(x)$.
\item Est-ce que la fonction $f(x)$ admet un maximum ou un minimum? Si oui, calculer sa valeur.
\end{enumerate}
\end{exercise}
\begin{solution}
\begin{enumerate}
\item Dérivée de $f(x)$: $f'(x) = - 12x^2 + 252x + 2784$
\item
\begin{align*}
f'(29) &= - 12 \times 29^{2} + 252 \times 29 + 2784\\&= - 12 \times 841 + 7308 + 2784\\&= - 10092 + 10092\\&= 0
\end{align*}
\begin{align*}
f'(-8) &= - 12 \times - 8^{2} + 252(- 8) + 2784\\&= - 12 \times 64 - 2016 + 2784\\&= - 768 + 768\\&= 0
\end{align*}
Donc $x = 29$ et $x=-8$ sont des racines de $f'(x) = - 12x^2 + 252x + 2784$.
\item On en déduit la forme factorisée suivante
\[
f'(x) = -12 (x - 29)(x--8)
\]
\item Pas de correction disponible
\item À causes des branches extérieurs, la fonction $f(x)$ n'a pas de maximum ou de minimum.
\end{enumerate}
\end{solution}
%\printsolutionstype{exercise}
\end{document}
%%% Local Variables:
%%% mode: latex
%%% TeX-master: "master"
%%% End:

View File

@ -0,0 +1,203 @@
\documentclass[a5paper,10pt]{article}
\usepackage{myXsim}
\usepackage{tasks}
% Title Page
\title{DM2 \hfill BERTAN Ufuk}
\tribe{TST}
\date{\hfillÀ render pour le Mercredi 24 février}
\xsimsetup{
solution/print = true
}
\begin{document}
\maketitle
\begin{exercise}[subtitle={Loi binomiale}]
Trois personnes s'apprêtent à passer le portique de sécurité. On suppose que pour chaque personne la probabilité que le portique sonne est égale à $0.04$.
Soit $X$ la variable aléatoire donnant le nombre de personnes faisant sonner le portique, parmi les 3 personnes de ce groupe.
\begin{enumerate}
\item Tracer l'arbre représentant le situation.
\item Justifier que $X$ suit une loi binomiale dont on précisera les paramètres.
\item Quelle est la probabilité qu'une seule personne fasse sonner le portique?
\item Calculer puis interpréter les probabilités suivantes
\[
P(X = 0) \qquad \qquad P(X \geq 2)
\]
\item Calculer l'espérance de $X$ et interpréter le résultat.
\end{enumerate}
\end{exercise}
\begin{solution}
\begin{enumerate}
\item
\begin{tikzpicture}[sloped]
\node {.}
child {node {$0$}
child {node {$0$}
child {node {$0$}
edge from parent
node[above] {0.96}
}
child {node {$1$}
edge from parent
node[above] {0.04}
}
edge from parent
node[above] {0.96}
}
child[missing] {}
child {node {$1$}
child {node {$0$}
edge from parent
node[above] {0.96}
}
child {node {$1$}
edge from parent
node[above] {0.04}
}
edge from parent
node[above] {0.96}
}
edge from parent
node[above] {0.96}
}
child[missing] {}
child[missing] {}
child[missing] {}
child { node {$1$}
child {node {$0$}
child {node {$0$}
edge from parent
node[above] {0.96}
}
child {node {$1$}
edge from parent
node[above] {0.04}
}
edge from parent
node[above] {0.96}
}
child[missing] {}
child {node {$1$}
child {node {$0$}
edge from parent
node[above] {0.96}
}
child {node {$1$}
edge from parent
node[above] {0.04}
}
edge from parent
node[above] {0.96}
}
edge from parent
node[above] {0.04}
} ;
\end{tikzpicture}
\item Chaque personne a 2 possibilités (1: fait sonner ou 2: ne fait pas sonner) et l'on fait passer 3 personnes ce qui correspond à une répétition identique et aléatoire. On peut donc modéliser la situation par une loi binomiale.
\[
X \sim \mathcal{B}(3; 0.76)
\]
\item Probabilité qu'une seule personne fasse sonner le portique. On voit qu'il y a 3 branches qui correspondent à cette situation dont
\[
P(X = 1) = 3 \times 0.04^1 \times 0.96^2 \approx 0.111
\]
\item
\[
P(X = 0) = 0.96^3 \approx 0.885
\]
\[
P(X \geq 2) = P(X = 2) + P(X = 3) = 3 \times 0.04^2 \times 0.96^1 + 0.04^3 \approx 0.005
\]
\item Il faut d'abord tracer le tableau résumant la loi de probabilité:
\begin{center}
\begin{tabular}{|c|*{4}{c|}}
\hline
Valeur & 0 & 1 & 2 & 3 \\
\hline
Probabilité & $0.885$ & $0.111$ & $0.005$ &$0.0$ \\
\hline
\end{tabular}
\end{center}
On peut alors calculer l'espérance
\[
E[X] = 0 \times 0.885 + 1 \times 0.111 + 2 \times 0.005 + 3 \times 0.0 = 0.12
\]
On peut donc estimer qu'il y aura en moyenne $0.12$ personnes qui feront sonner le portique sur les 3 personnes.
\end{enumerate}
\end{solution}
\begin{exercise}[subtitle={Équation puissance}]
Résoudre les équations et inéquations suivantes
\begin{multicols}{2}
\begin{enumerate}
\item $10^x = 14$
\item $11^x = 35$
\item $0.05^x \leq 24$
\item $4 \times 0.92^x = 47$
\end{enumerate}
\end{multicols}
\end{exercise}
\begin{solution}
Les solutions ci-dessous ne sont pas justifiée car l'ordinateur ne sait pas faire. Par contre, vous vous devez savoir justifier vos réponses!
\begin{enumerate}
\item $x = \log(14)$
\item $x = \frac{\log(35)}{\log(11)}$
\item Il faut faire attention quand on divise par un log car ce dernier peut être négatif ce qui est le cas ici. Il faut donc pense à changer le sens de l'inégalité.
$x \geq \frac{\log(24)}{\log(0.05)}$
\item Il faut penser à faire la division à par $4$ avant d'utiliser le log car sinon, on ne peut pas utiliser la formule $\log(a^n) = n\times \log(a)$.
$x = \frac{\log(11.75)}{\log(0.92)}$
\end{enumerate}
\end{solution}
\begin{exercise}[subtitle={Étude de fonctions}]
Soit $f(x) = 4x^3 - 306x^2 + 4128x + 39$ une fonction définie sur $\R$.
\begin{enumerate}
\item Calculer $f'(x)$ la dérivée de $f(x)$.
\item Calculer $f'(43)$ et $f'(8)$.
\item En déduire une forme factorisée de $f'(x)$.
\item Étudier le signe de $f'(x)$ et en déduire les variations de $f(x)$.
\item Est-ce que la fonction $f(x)$ admet un maximum ou un minimum? Si oui, calculer sa valeur.
\end{enumerate}
\end{exercise}
\begin{solution}
\begin{enumerate}
\item Dérivée de $f(x)$: $f'(x) = 12x^2 - 612x + 4128$
\item
\begin{align*}
f'(43) &= 12 \times 43^{2} - 612 \times 43 + 4128\\&= 12 \times 1849 - 26316 + 4128\\&= 22188 - 22188\\&= 0
\end{align*}
\begin{align*}
f'(8) &= 12 \times 8^{2} - 612 \times 8 + 4128\\&= 12 \times 64 - 4896 + 4128\\&= 768 - 768\\&= 0
\end{align*}
Donc $x = 43$ et $x=8$ sont des racines de $f'(x) = 12x^2 - 612x + 4128$.
\item On en déduit la forme factorisée suivante
\[
f'(x) = 12 (x - 43)(x-8)
\]
\item Pas de correction disponible
\item À causes des branches extérieurs, la fonction $f(x)$ n'a pas de maximum ou de minimum.
\end{enumerate}
\end{solution}
%\printsolutionstype{exercise}
\end{document}
%%% Local Variables:
%%% mode: latex
%%% TeX-master: "master"
%%% End:

View File

@ -0,0 +1,203 @@
\documentclass[a5paper,10pt]{article}
\usepackage{myXsim}
\usepackage{tasks}
% Title Page
\title{DM2 \hfill BOUALIA Bilel}
\tribe{TST}
\date{\hfillÀ render pour le Mercredi 24 février}
\xsimsetup{
solution/print = true
}
\begin{document}
\maketitle
\begin{exercise}[subtitle={Loi binomiale}]
Trois personnes s'apprêtent à passer le portique de sécurité. On suppose que pour chaque personne la probabilité que le portique sonne est égale à $0.68$.
Soit $X$ la variable aléatoire donnant le nombre de personnes faisant sonner le portique, parmi les 3 personnes de ce groupe.
\begin{enumerate}
\item Tracer l'arbre représentant le situation.
\item Justifier que $X$ suit une loi binomiale dont on précisera les paramètres.
\item Quelle est la probabilité qu'une seule personne fasse sonner le portique?
\item Calculer puis interpréter les probabilités suivantes
\[
P(X = 0) \qquad \qquad P(X \geq 2)
\]
\item Calculer l'espérance de $X$ et interpréter le résultat.
\end{enumerate}
\end{exercise}
\begin{solution}
\begin{enumerate}
\item
\begin{tikzpicture}[sloped]
\node {.}
child {node {$0$}
child {node {$0$}
child {node {$0$}
edge from parent
node[above] {0.32}
}
child {node {$1$}
edge from parent
node[above] {0.68}
}
edge from parent
node[above] {0.32}
}
child[missing] {}
child {node {$1$}
child {node {$0$}
edge from parent
node[above] {0.32}
}
child {node {$1$}
edge from parent
node[above] {0.68}
}
edge from parent
node[above] {0.32}
}
edge from parent
node[above] {0.32}
}
child[missing] {}
child[missing] {}
child[missing] {}
child { node {$1$}
child {node {$0$}
child {node {$0$}
edge from parent
node[above] {0.32}
}
child {node {$1$}
edge from parent
node[above] {0.68}
}
edge from parent
node[above] {0.32}
}
child[missing] {}
child {node {$1$}
child {node {$0$}
edge from parent
node[above] {0.32}
}
child {node {$1$}
edge from parent
node[above] {0.68}
}
edge from parent
node[above] {0.32}
}
edge from parent
node[above] {0.68}
} ;
\end{tikzpicture}
\item Chaque personne a 2 possibilités (1: fait sonner ou 2: ne fait pas sonner) et l'on fait passer 3 personnes ce qui correspond à une répétition identique et aléatoire. On peut donc modéliser la situation par une loi binomiale.
\[
X \sim \mathcal{B}(3; 0.76)
\]
\item Probabilité qu'une seule personne fasse sonner le portique. On voit qu'il y a 3 branches qui correspondent à cette situation dont
\[
P(X = 1) = 3 \times 0.68^1 \times 0.32^2 \approx 0.209
\]
\item
\[
P(X = 0) = 0.32^3 \approx 0.033
\]
\[
P(X \geq 2) = P(X = 2) + P(X = 3) = 3 \times 0.68^2 \times 0.32^1 + 0.68^3 \approx 0.758
\]
\item Il faut d'abord tracer le tableau résumant la loi de probabilité:
\begin{center}
\begin{tabular}{|c|*{4}{c|}}
\hline
Valeur & 0 & 1 & 2 & 3 \\
\hline
Probabilité & $0.033$ & $0.209$ & $0.444$ &$0.314$ \\
\hline
\end{tabular}
\end{center}
On peut alors calculer l'espérance
\[
E[X] = 0 \times 0.033 + 1 \times 0.209 + 2 \times 0.444 + 3 \times 0.314 = 2.04
\]
On peut donc estimer qu'il y aura en moyenne $2.04$ personnes qui feront sonner le portique sur les 3 personnes.
\end{enumerate}
\end{solution}
\begin{exercise}[subtitle={Équation puissance}]
Résoudre les équations et inéquations suivantes
\begin{multicols}{2}
\begin{enumerate}
\item $10^x = 45$
\item $10^x = 5$
\item $0.69^x \leq 42$
\item $4 \times 0.04^x = 21$
\end{enumerate}
\end{multicols}
\end{exercise}
\begin{solution}
Les solutions ci-dessous ne sont pas justifiée car l'ordinateur ne sait pas faire. Par contre, vous vous devez savoir justifier vos réponses!
\begin{enumerate}
\item $x = \log(45)$
\item $x = \frac{\log(5)}{\log(10)}$
\item Il faut faire attention quand on divise par un log car ce dernier peut être négatif ce qui est le cas ici. Il faut donc pense à changer le sens de l'inégalité.
$x \geq \frac{\log(42)}{\log(0.69)}$
\item Il faut penser à faire la division à par $4$ avant d'utiliser le log car sinon, on ne peut pas utiliser la formule $\log(a^n) = n\times \log(a)$.
$x = \frac{\log(5.25)}{\log(0.04)}$
\end{enumerate}
\end{solution}
\begin{exercise}[subtitle={Étude de fonctions}]
Soit $f(x) = 10x^3 - 645x^2 - 7200x - 16$ une fonction définie sur $\R$.
\begin{enumerate}
\item Calculer $f'(x)$ la dérivée de $f(x)$.
\item Calculer $f'(48)$ et $f'(-5)$.
\item En déduire une forme factorisée de $f'(x)$.
\item Étudier le signe de $f'(x)$ et en déduire les variations de $f(x)$.
\item Est-ce que la fonction $f(x)$ admet un maximum ou un minimum? Si oui, calculer sa valeur.
\end{enumerate}
\end{exercise}
\begin{solution}
\begin{enumerate}
\item Dérivée de $f(x)$: $f'(x) = 30x^2 - 1290x - 7200$
\item
\begin{align*}
f'(48) &= 30 \times 48^{2} - 1290 \times 48 - 7200\\&= 30 \times 2304 - 61920 - 7200\\&= 69120 - 69120\\&= 0
\end{align*}
\begin{align*}
f'(-5) &= 30 \times - 5^{2} - 1290(- 5) - 7200\\&= 30 \times 25 + 6450 - 7200\\&= 750 - 750\\&= 0
\end{align*}
Donc $x = 48$ et $x=-5$ sont des racines de $f'(x) = 30x^2 - 1290x - 7200$.
\item On en déduit la forme factorisée suivante
\[
f'(x) = 30 (x - 48)(x--5)
\]
\item Pas de correction disponible
\item À causes des branches extérieurs, la fonction $f(x)$ n'a pas de maximum ou de minimum.
\end{enumerate}
\end{solution}
%\printsolutionstype{exercise}
\end{document}
%%% Local Variables:
%%% mode: latex
%%% TeX-master: "master"
%%% End:

View File

@ -0,0 +1,203 @@
\documentclass[a5paper,10pt]{article}
\usepackage{myXsim}
\usepackage{tasks}
% Title Page
\title{DM2 \hfill BOUCHOUX Kevin}
\tribe{TST}
\date{\hfillÀ render pour le Mercredi 24 février}
\xsimsetup{
solution/print = true
}
\begin{document}
\maketitle
\begin{exercise}[subtitle={Loi binomiale}]
Trois personnes s'apprêtent à passer le portique de sécurité. On suppose que pour chaque personne la probabilité que le portique sonne est égale à $0.7$.
Soit $X$ la variable aléatoire donnant le nombre de personnes faisant sonner le portique, parmi les 3 personnes de ce groupe.
\begin{enumerate}
\item Tracer l'arbre représentant le situation.
\item Justifier que $X$ suit une loi binomiale dont on précisera les paramètres.
\item Quelle est la probabilité qu'une seule personne fasse sonner le portique?
\item Calculer puis interpréter les probabilités suivantes
\[
P(X = 0) \qquad \qquad P(X \geq 2)
\]
\item Calculer l'espérance de $X$ et interpréter le résultat.
\end{enumerate}
\end{exercise}
\begin{solution}
\begin{enumerate}
\item
\begin{tikzpicture}[sloped]
\node {.}
child {node {$0$}
child {node {$0$}
child {node {$0$}
edge from parent
node[above] {0.3}
}
child {node {$1$}
edge from parent
node[above] {0.7}
}
edge from parent
node[above] {0.3}
}
child[missing] {}
child {node {$1$}
child {node {$0$}
edge from parent
node[above] {0.3}
}
child {node {$1$}
edge from parent
node[above] {0.7}
}
edge from parent
node[above] {0.3}
}
edge from parent
node[above] {0.3}
}
child[missing] {}
child[missing] {}
child[missing] {}
child { node {$1$}
child {node {$0$}
child {node {$0$}
edge from parent
node[above] {0.3}
}
child {node {$1$}
edge from parent
node[above] {0.7}
}
edge from parent
node[above] {0.3}
}
child[missing] {}
child {node {$1$}
child {node {$0$}
edge from parent
node[above] {0.3}
}
child {node {$1$}
edge from parent
node[above] {0.7}
}
edge from parent
node[above] {0.3}
}
edge from parent
node[above] {0.7}
} ;
\end{tikzpicture}
\item Chaque personne a 2 possibilités (1: fait sonner ou 2: ne fait pas sonner) et l'on fait passer 3 personnes ce qui correspond à une répétition identique et aléatoire. On peut donc modéliser la situation par une loi binomiale.
\[
X \sim \mathcal{B}(3; 0.76)
\]
\item Probabilité qu'une seule personne fasse sonner le portique. On voit qu'il y a 3 branches qui correspondent à cette situation dont
\[
P(X = 1) = 3 \times 0.7^1 \times 0.3^2 \approx 0.189
\]
\item
\[
P(X = 0) = 0.3^3 \approx 0.027
\]
\[
P(X \geq 2) = P(X = 2) + P(X = 3) = 3 \times 0.7^2 \times 0.3^1 + 0.7^3 \approx 0.784
\]
\item Il faut d'abord tracer le tableau résumant la loi de probabilité:
\begin{center}
\begin{tabular}{|c|*{4}{c|}}
\hline
Valeur & 0 & 1 & 2 & 3 \\
\hline
Probabilité & $0.027$ & $0.189$ & $0.441$ &$0.343$ \\
\hline
\end{tabular}
\end{center}
On peut alors calculer l'espérance
\[
E[X] = 0 \times 0.027 + 1 \times 0.189 + 2 \times 0.441 + 3 \times 0.343 = 2.1
\]
On peut donc estimer qu'il y aura en moyenne $2.1$ personnes qui feront sonner le portique sur les 3 personnes.
\end{enumerate}
\end{solution}
\begin{exercise}[subtitle={Équation puissance}]
Résoudre les équations et inéquations suivantes
\begin{multicols}{2}
\begin{enumerate}
\item $10^x = 4$
\item $7^x = 14$
\item $0.44^x \leq 29$
\item $6 \times 0.27^x = 10$
\end{enumerate}
\end{multicols}
\end{exercise}
\begin{solution}
Les solutions ci-dessous ne sont pas justifiée car l'ordinateur ne sait pas faire. Par contre, vous vous devez savoir justifier vos réponses!
\begin{enumerate}
\item $x = \log(4)$
\item $x = \frac{\log(14)}{\log(7)}$
\item Il faut faire attention quand on divise par un log car ce dernier peut être négatif ce qui est le cas ici. Il faut donc pense à changer le sens de l'inégalité.
$x \geq \frac{\log(29)}{\log(0.44)}$
\item Il faut penser à faire la division à par $6$ avant d'utiliser le log car sinon, on ne peut pas utiliser la formule $\log(a^n) = n\times \log(a)$.
$x = \frac{\log(1.67)}{\log(0.27)}$
\end{enumerate}
\end{solution}
\begin{exercise}[subtitle={Étude de fonctions}]
Soit $f(x) = 10x^3 - 840x^2 + 18450x - 1$ une fonction définie sur $\R$.
\begin{enumerate}
\item Calculer $f'(x)$ la dérivée de $f(x)$.
\item Calculer $f'(41)$ et $f'(15)$.
\item En déduire une forme factorisée de $f'(x)$.
\item Étudier le signe de $f'(x)$ et en déduire les variations de $f(x)$.
\item Est-ce que la fonction $f(x)$ admet un maximum ou un minimum? Si oui, calculer sa valeur.
\end{enumerate}
\end{exercise}
\begin{solution}
\begin{enumerate}
\item Dérivée de $f(x)$: $f'(x) = 30x^2 - 1680x + 18450$
\item
\begin{align*}
f'(41) &= 30 \times 41^{2} - 1680 \times 41 + 18450\\&= 30 \times 1681 - 68880 + 18450\\&= 50430 - 50430\\&= 0
\end{align*}
\begin{align*}
f'(15) &= 30 \times 15^{2} - 1680 \times 15 + 18450\\&= 30 \times 225 - 25200 + 18450\\&= 6750 - 6750\\&= 0
\end{align*}
Donc $x = 41$ et $x=15$ sont des racines de $f'(x) = 30x^2 - 1680x + 18450$.
\item On en déduit la forme factorisée suivante
\[
f'(x) = 30 (x - 41)(x-15)
\]
\item Pas de correction disponible
\item À causes des branches extérieurs, la fonction $f(x)$ n'a pas de maximum ou de minimum.
\end{enumerate}
\end{solution}
%\printsolutionstype{exercise}
\end{document}
%%% Local Variables:
%%% mode: latex
%%% TeX-master: "master"
%%% End:

View File

@ -0,0 +1,203 @@
\documentclass[a5paper,10pt]{article}
\usepackage{myXsim}
\usepackage{tasks}
% Title Page
\title{DM2 \hfill BUDIN Nathan}
\tribe{TST}
\date{\hfillÀ render pour le Mercredi 24 février}
\xsimsetup{
solution/print = true
}
\begin{document}
\maketitle
\begin{exercise}[subtitle={Loi binomiale}]
Trois personnes s'apprêtent à passer le portique de sécurité. On suppose que pour chaque personne la probabilité que le portique sonne est égale à $0.37$.
Soit $X$ la variable aléatoire donnant le nombre de personnes faisant sonner le portique, parmi les 3 personnes de ce groupe.
\begin{enumerate}
\item Tracer l'arbre représentant le situation.
\item Justifier que $X$ suit une loi binomiale dont on précisera les paramètres.
\item Quelle est la probabilité qu'une seule personne fasse sonner le portique?
\item Calculer puis interpréter les probabilités suivantes
\[
P(X = 0) \qquad \qquad P(X \geq 2)
\]
\item Calculer l'espérance de $X$ et interpréter le résultat.
\end{enumerate}
\end{exercise}
\begin{solution}
\begin{enumerate}
\item
\begin{tikzpicture}[sloped]
\node {.}
child {node {$0$}
child {node {$0$}
child {node {$0$}
edge from parent
node[above] {0.63}
}
child {node {$1$}
edge from parent
node[above] {0.37}
}
edge from parent
node[above] {0.63}
}
child[missing] {}
child {node {$1$}
child {node {$0$}
edge from parent
node[above] {0.63}
}
child {node {$1$}
edge from parent
node[above] {0.37}
}
edge from parent
node[above] {0.63}
}
edge from parent
node[above] {0.63}
}
child[missing] {}
child[missing] {}
child[missing] {}
child { node {$1$}
child {node {$0$}
child {node {$0$}
edge from parent
node[above] {0.63}
}
child {node {$1$}
edge from parent
node[above] {0.37}
}
edge from parent
node[above] {0.63}
}
child[missing] {}
child {node {$1$}
child {node {$0$}
edge from parent
node[above] {0.63}
}
child {node {$1$}
edge from parent
node[above] {0.37}
}
edge from parent
node[above] {0.63}
}
edge from parent
node[above] {0.37}
} ;
\end{tikzpicture}
\item Chaque personne a 2 possibilités (1: fait sonner ou 2: ne fait pas sonner) et l'on fait passer 3 personnes ce qui correspond à une répétition identique et aléatoire. On peut donc modéliser la situation par une loi binomiale.
\[
X \sim \mathcal{B}(3; 0.76)
\]
\item Probabilité qu'une seule personne fasse sonner le portique. On voit qu'il y a 3 branches qui correspondent à cette situation dont
\[
P(X = 1) = 3 \times 0.37^1 \times 0.63^2 \approx 0.441
\]
\item
\[
P(X = 0) = 0.63^3 \approx 0.25
\]
\[
P(X \geq 2) = P(X = 2) + P(X = 3) = 3 \times 0.37^2 \times 0.63^1 + 0.37^3 \approx 0.31
\]
\item Il faut d'abord tracer le tableau résumant la loi de probabilité:
\begin{center}
\begin{tabular}{|c|*{4}{c|}}
\hline
Valeur & 0 & 1 & 2 & 3 \\
\hline
Probabilité & $0.25$ & $0.441$ & $0.259$ &$0.051$ \\
\hline
\end{tabular}
\end{center}
On peut alors calculer l'espérance
\[
E[X] = 0 \times 0.25 + 1 \times 0.441 + 2 \times 0.259 + 3 \times 0.051 = 1.11
\]
On peut donc estimer qu'il y aura en moyenne $1.11$ personnes qui feront sonner le portique sur les 3 personnes.
\end{enumerate}
\end{solution}
\begin{exercise}[subtitle={Équation puissance}]
Résoudre les équations et inéquations suivantes
\begin{multicols}{2}
\begin{enumerate}
\item $10^x = 44$
\item $2^x = 33$
\item $0.94^x \leq 43$
\item $9 \times 0.17^x = 30$
\end{enumerate}
\end{multicols}
\end{exercise}
\begin{solution}
Les solutions ci-dessous ne sont pas justifiée car l'ordinateur ne sait pas faire. Par contre, vous vous devez savoir justifier vos réponses!
\begin{enumerate}
\item $x = \log(44)$
\item $x = \frac{\log(33)}{\log(2)}$
\item Il faut faire attention quand on divise par un log car ce dernier peut être négatif ce qui est le cas ici. Il faut donc pense à changer le sens de l'inégalité.
$x \geq \frac{\log(43)}{\log(0.94)}$
\item Il faut penser à faire la division à par $9$ avant d'utiliser le log car sinon, on ne peut pas utiliser la formule $\log(a^n) = n\times \log(a)$.
$x = \frac{\log(3.33)}{\log(0.17)}$
\end{enumerate}
\end{solution}
\begin{exercise}[subtitle={Étude de fonctions}]
Soit $f(x) = - 2x^3 + 99x^2 + 2580x + 46$ une fonction définie sur $\R$.
\begin{enumerate}
\item Calculer $f'(x)$ la dérivée de $f(x)$.
\item Calculer $f'(43)$ et $f'(-10)$.
\item En déduire une forme factorisée de $f'(x)$.
\item Étudier le signe de $f'(x)$ et en déduire les variations de $f(x)$.
\item Est-ce que la fonction $f(x)$ admet un maximum ou un minimum? Si oui, calculer sa valeur.
\end{enumerate}
\end{exercise}
\begin{solution}
\begin{enumerate}
\item Dérivée de $f(x)$: $f'(x) = - 6x^2 + 198x + 2580$
\item
\begin{align*}
f'(43) &= - 6 \times 43^{2} + 198 \times 43 + 2580\\&= - 6 \times 1849 + 8514 + 2580\\&= - 11094 + 11094\\&= 0
\end{align*}
\begin{align*}
f'(-10) &= - 6 \times - 10^{2} + 198(- 10) + 2580\\&= - 6 \times 100 - 1980 + 2580\\&= - 600 + 600\\&= 0
\end{align*}
Donc $x = 43$ et $x=-10$ sont des racines de $f'(x) = - 6x^2 + 198x + 2580$.
\item On en déduit la forme factorisée suivante
\[
f'(x) = -6 (x - 43)(x--10)
\]
\item Pas de correction disponible
\item À causes des branches extérieurs, la fonction $f(x)$ n'a pas de maximum ou de minimum.
\end{enumerate}
\end{solution}
%\printsolutionstype{exercise}
\end{document}
%%% Local Variables:
%%% mode: latex
%%% TeX-master: "master"
%%% End:

View File

@ -0,0 +1,203 @@
\documentclass[a5paper,10pt]{article}
\usepackage{myXsim}
\usepackage{tasks}
% Title Page
\title{DM2 \hfill CAGLAR Rojin}
\tribe{TST}
\date{\hfillÀ render pour le Mercredi 24 février}
\xsimsetup{
solution/print = true
}
\begin{document}
\maketitle
\begin{exercise}[subtitle={Loi binomiale}]
Trois personnes s'apprêtent à passer le portique de sécurité. On suppose que pour chaque personne la probabilité que le portique sonne est égale à $0.86$.
Soit $X$ la variable aléatoire donnant le nombre de personnes faisant sonner le portique, parmi les 3 personnes de ce groupe.
\begin{enumerate}
\item Tracer l'arbre représentant le situation.
\item Justifier que $X$ suit une loi binomiale dont on précisera les paramètres.
\item Quelle est la probabilité qu'une seule personne fasse sonner le portique?
\item Calculer puis interpréter les probabilités suivantes
\[
P(X = 0) \qquad \qquad P(X \geq 2)
\]
\item Calculer l'espérance de $X$ et interpréter le résultat.
\end{enumerate}
\end{exercise}
\begin{solution}
\begin{enumerate}
\item
\begin{tikzpicture}[sloped]
\node {.}
child {node {$0$}
child {node {$0$}
child {node {$0$}
edge from parent
node[above] {0.14}
}
child {node {$1$}
edge from parent
node[above] {0.86}
}
edge from parent
node[above] {0.14}
}
child[missing] {}
child {node {$1$}
child {node {$0$}
edge from parent
node[above] {0.14}
}
child {node {$1$}
edge from parent
node[above] {0.86}
}
edge from parent
node[above] {0.14}
}
edge from parent
node[above] {0.14}
}
child[missing] {}
child[missing] {}
child[missing] {}
child { node {$1$}
child {node {$0$}
child {node {$0$}
edge from parent
node[above] {0.14}
}
child {node {$1$}
edge from parent
node[above] {0.86}
}
edge from parent
node[above] {0.14}
}
child[missing] {}
child {node {$1$}
child {node {$0$}
edge from parent
node[above] {0.14}
}
child {node {$1$}
edge from parent
node[above] {0.86}
}
edge from parent
node[above] {0.14}
}
edge from parent
node[above] {0.86}
} ;
\end{tikzpicture}
\item Chaque personne a 2 possibilités (1: fait sonner ou 2: ne fait pas sonner) et l'on fait passer 3 personnes ce qui correspond à une répétition identique et aléatoire. On peut donc modéliser la situation par une loi binomiale.
\[
X \sim \mathcal{B}(3; 0.76)
\]
\item Probabilité qu'une seule personne fasse sonner le portique. On voit qu'il y a 3 branches qui correspondent à cette situation dont
\[
P(X = 1) = 3 \times 0.86^1 \times 0.14^2 \approx 0.051
\]
\item
\[
P(X = 0) = 0.14^3 \approx 0.003
\]
\[
P(X \geq 2) = P(X = 2) + P(X = 3) = 3 \times 0.86^2 \times 0.14^1 + 0.86^3 \approx 0.947
\]
\item Il faut d'abord tracer le tableau résumant la loi de probabilité:
\begin{center}
\begin{tabular}{|c|*{4}{c|}}
\hline
Valeur & 0 & 1 & 2 & 3 \\
\hline
Probabilité & $0.003$ & $0.051$ & $0.311$ &$0.636$ \\
\hline
\end{tabular}
\end{center}
On peut alors calculer l'espérance
\[
E[X] = 0 \times 0.003 + 1 \times 0.051 + 2 \times 0.311 + 3 \times 0.636 = 2.58
\]
On peut donc estimer qu'il y aura en moyenne $2.58$ personnes qui feront sonner le portique sur les 3 personnes.
\end{enumerate}
\end{solution}
\begin{exercise}[subtitle={Équation puissance}]
Résoudre les équations et inéquations suivantes
\begin{multicols}{2}
\begin{enumerate}
\item $10^x = 14$
\item $2^x = 38$
\item $0.15^x \leq 40$
\item $9 \times 0.56^x = 29$
\end{enumerate}
\end{multicols}
\end{exercise}
\begin{solution}
Les solutions ci-dessous ne sont pas justifiée car l'ordinateur ne sait pas faire. Par contre, vous vous devez savoir justifier vos réponses!
\begin{enumerate}
\item $x = \log(14)$
\item $x = \frac{\log(38)}{\log(2)}$
\item Il faut faire attention quand on divise par un log car ce dernier peut être négatif ce qui est le cas ici. Il faut donc pense à changer le sens de l'inégalité.
$x \geq \frac{\log(40)}{\log(0.15)}$
\item Il faut penser à faire la division à par $9$ avant d'utiliser le log car sinon, on ne peut pas utiliser la formule $\log(a^n) = n\times \log(a)$.
$x = \frac{\log(3.22)}{\log(0.56)}$
\end{enumerate}
\end{solution}
\begin{exercise}[subtitle={Étude de fonctions}]
Soit $f(x) = - x^3 + 67.5x^2 - 258x - 11$ une fonction définie sur $\R$.
\begin{enumerate}
\item Calculer $f'(x)$ la dérivée de $f(x)$.
\item Calculer $f'(43)$ et $f'(2)$.
\item En déduire une forme factorisée de $f'(x)$.
\item Étudier le signe de $f'(x)$ et en déduire les variations de $f(x)$.
\item Est-ce que la fonction $f(x)$ admet un maximum ou un minimum? Si oui, calculer sa valeur.
\end{enumerate}
\end{exercise}
\begin{solution}
\begin{enumerate}
\item Dérivée de $f(x)$: $f'(x) = - 3x^2 + 135x - 258$
\item
\begin{align*}
f'(43) &= - 3 \times 43^{2} + 135 \times 43 - 258\\&= - 3 \times 1849 + 5805 - 258\\&= - 5547 + 5547\\&= 0
\end{align*}
\begin{align*}
f'(2) &= - 3 \times 2^{2} + 135 \times 2 - 258\\&= - 3 \times 4 + 270 - 258\\&= - 12 + 12\\&= 0
\end{align*}
Donc $x = 43$ et $x=2$ sont des racines de $f'(x) = - 3x^2 + 135x - 258$.
\item On en déduit la forme factorisée suivante
\[
f'(x) = -3 (x - 43)(x-2)
\]
\item Pas de correction disponible
\item À causes des branches extérieurs, la fonction $f(x)$ n'a pas de maximum ou de minimum.
\end{enumerate}
\end{solution}
%\printsolutionstype{exercise}
\end{document}
%%% Local Variables:
%%% mode: latex
%%% TeX-master: "master"
%%% End:

View File

@ -0,0 +1,203 @@
\documentclass[a5paper,10pt]{article}
\usepackage{myXsim}
\usepackage{tasks}
% Title Page
\title{DM2 \hfill DARICHE Kaïs}
\tribe{TST}
\date{\hfillÀ render pour le Mercredi 24 février}
\xsimsetup{
solution/print = true
}
\begin{document}
\maketitle
\begin{exercise}[subtitle={Loi binomiale}]
Trois personnes s'apprêtent à passer le portique de sécurité. On suppose que pour chaque personne la probabilité que le portique sonne est égale à $0.16$.
Soit $X$ la variable aléatoire donnant le nombre de personnes faisant sonner le portique, parmi les 3 personnes de ce groupe.
\begin{enumerate}
\item Tracer l'arbre représentant le situation.
\item Justifier que $X$ suit une loi binomiale dont on précisera les paramètres.
\item Quelle est la probabilité qu'une seule personne fasse sonner le portique?
\item Calculer puis interpréter les probabilités suivantes
\[
P(X = 0) \qquad \qquad P(X \geq 2)
\]
\item Calculer l'espérance de $X$ et interpréter le résultat.
\end{enumerate}
\end{exercise}
\begin{solution}
\begin{enumerate}
\item
\begin{tikzpicture}[sloped]
\node {.}
child {node {$0$}
child {node {$0$}
child {node {$0$}
edge from parent
node[above] {0.84}
}
child {node {$1$}
edge from parent
node[above] {0.16}
}
edge from parent
node[above] {0.84}
}
child[missing] {}
child {node {$1$}
child {node {$0$}
edge from parent
node[above] {0.84}
}
child {node {$1$}
edge from parent
node[above] {0.16}
}
edge from parent
node[above] {0.84}
}
edge from parent
node[above] {0.84}
}
child[missing] {}
child[missing] {}
child[missing] {}
child { node {$1$}
child {node {$0$}
child {node {$0$}
edge from parent
node[above] {0.84}
}
child {node {$1$}
edge from parent
node[above] {0.16}
}
edge from parent
node[above] {0.84}
}
child[missing] {}
child {node {$1$}
child {node {$0$}
edge from parent
node[above] {0.84}
}
child {node {$1$}
edge from parent
node[above] {0.16}
}
edge from parent
node[above] {0.84}
}
edge from parent
node[above] {0.16}
} ;
\end{tikzpicture}
\item Chaque personne a 2 possibilités (1: fait sonner ou 2: ne fait pas sonner) et l'on fait passer 3 personnes ce qui correspond à une répétition identique et aléatoire. On peut donc modéliser la situation par une loi binomiale.
\[
X \sim \mathcal{B}(3; 0.76)
\]
\item Probabilité qu'une seule personne fasse sonner le portique. On voit qu'il y a 3 branches qui correspondent à cette situation dont
\[
P(X = 1) = 3 \times 0.16^1 \times 0.84^2 \approx 0.339
\]
\item
\[
P(X = 0) = 0.84^3 \approx 0.593
\]
\[
P(X \geq 2) = P(X = 2) + P(X = 3) = 3 \times 0.16^2 \times 0.84^1 + 0.16^3 \approx 0.069
\]
\item Il faut d'abord tracer le tableau résumant la loi de probabilité:
\begin{center}
\begin{tabular}{|c|*{4}{c|}}
\hline
Valeur & 0 & 1 & 2 & 3 \\
\hline
Probabilité & $0.593$ & $0.339$ & $0.065$ &$0.004$ \\
\hline
\end{tabular}
\end{center}
On peut alors calculer l'espérance
\[
E[X] = 0 \times 0.593 + 1 \times 0.339 + 2 \times 0.065 + 3 \times 0.004 = 0.48
\]
On peut donc estimer qu'il y aura en moyenne $0.48$ personnes qui feront sonner le portique sur les 3 personnes.
\end{enumerate}
\end{solution}
\begin{exercise}[subtitle={Équation puissance}]
Résoudre les équations et inéquations suivantes
\begin{multicols}{2}
\begin{enumerate}
\item $10^x = 35$
\item $14^x = 11$
\item $0.39^x \leq 48$
\item $3 \times 0.07^x = 4$
\end{enumerate}
\end{multicols}
\end{exercise}
\begin{solution}
Les solutions ci-dessous ne sont pas justifiée car l'ordinateur ne sait pas faire. Par contre, vous vous devez savoir justifier vos réponses!
\begin{enumerate}
\item $x = \log(35)$
\item $x = \frac{\log(11)}{\log(14)}$
\item Il faut faire attention quand on divise par un log car ce dernier peut être négatif ce qui est le cas ici. Il faut donc pense à changer le sens de l'inégalité.
$x \geq \frac{\log(48)}{\log(0.39)}$
\item Il faut penser à faire la division à par $3$ avant d'utiliser le log car sinon, on ne peut pas utiliser la formule $\log(a^n) = n\times \log(a)$.
$x = \frac{\log(1.33)}{\log(0.07)}$
\end{enumerate}
\end{solution}
\begin{exercise}[subtitle={Étude de fonctions}]
Soit $f(x) = x^3 - 54x^2 + 780x + 36$ une fonction définie sur $\R$.
\begin{enumerate}
\item Calculer $f'(x)$ la dérivée de $f(x)$.
\item Calculer $f'(26)$ et $f'(10)$.
\item En déduire une forme factorisée de $f'(x)$.
\item Étudier le signe de $f'(x)$ et en déduire les variations de $f(x)$.
\item Est-ce que la fonction $f(x)$ admet un maximum ou un minimum? Si oui, calculer sa valeur.
\end{enumerate}
\end{exercise}
\begin{solution}
\begin{enumerate}
\item Dérivée de $f(x)$: $f'(x) = 3x^2 - 108x + 780$
\item
\begin{align*}
f'(26) &= 3 \times 26^{2} - 108 \times 26 + 780\\&= 3 \times 676 - 2808 + 780\\&= 2028 - 2028\\&= 0
\end{align*}
\begin{align*}
f'(10) &= 3 \times 10^{2} - 108 \times 10 + 780\\&= 3 \times 100 - 1080 + 780\\&= 300 - 300\\&= 0
\end{align*}
Donc $x = 26$ et $x=10$ sont des racines de $f'(x) = 3x^2 - 108x + 780$.
\item On en déduit la forme factorisée suivante
\[
f'(x) = 3 (x - 26)(x-10)
\]
\item Pas de correction disponible
\item À causes des branches extérieurs, la fonction $f(x)$ n'a pas de maximum ou de minimum.
\end{enumerate}
\end{solution}
%\printsolutionstype{exercise}
\end{document}
%%% Local Variables:
%%% mode: latex
%%% TeX-master: "master"
%%% End:

View File

@ -0,0 +1,203 @@
\documentclass[a5paper,10pt]{article}
\usepackage{myXsim}
\usepackage{tasks}
% Title Page
\title{DM2 \hfill DEBRAS Noémie}
\tribe{TST}
\date{\hfillÀ render pour le Mercredi 24 février}
\xsimsetup{
solution/print = true
}
\begin{document}
\maketitle
\begin{exercise}[subtitle={Loi binomiale}]
Trois personnes s'apprêtent à passer le portique de sécurité. On suppose que pour chaque personne la probabilité que le portique sonne est égale à $0.1$.
Soit $X$ la variable aléatoire donnant le nombre de personnes faisant sonner le portique, parmi les 3 personnes de ce groupe.
\begin{enumerate}
\item Tracer l'arbre représentant le situation.
\item Justifier que $X$ suit une loi binomiale dont on précisera les paramètres.
\item Quelle est la probabilité qu'une seule personne fasse sonner le portique?
\item Calculer puis interpréter les probabilités suivantes
\[
P(X = 0) \qquad \qquad P(X \geq 2)
\]
\item Calculer l'espérance de $X$ et interpréter le résultat.
\end{enumerate}
\end{exercise}
\begin{solution}
\begin{enumerate}
\item
\begin{tikzpicture}[sloped]
\node {.}
child {node {$0$}
child {node {$0$}
child {node {$0$}
edge from parent
node[above] {0.9}
}
child {node {$1$}
edge from parent
node[above] {0.1}
}
edge from parent
node[above] {0.9}
}
child[missing] {}
child {node {$1$}
child {node {$0$}
edge from parent
node[above] {0.9}
}
child {node {$1$}
edge from parent
node[above] {0.1}
}
edge from parent
node[above] {0.9}
}
edge from parent
node[above] {0.9}
}
child[missing] {}
child[missing] {}
child[missing] {}
child { node {$1$}
child {node {$0$}
child {node {$0$}
edge from parent
node[above] {0.9}
}
child {node {$1$}
edge from parent
node[above] {0.1}
}
edge from parent
node[above] {0.9}
}
child[missing] {}
child {node {$1$}
child {node {$0$}
edge from parent
node[above] {0.9}
}
child {node {$1$}
edge from parent
node[above] {0.1}
}
edge from parent
node[above] {0.9}
}
edge from parent
node[above] {0.1}
} ;
\end{tikzpicture}
\item Chaque personne a 2 possibilités (1: fait sonner ou 2: ne fait pas sonner) et l'on fait passer 3 personnes ce qui correspond à une répétition identique et aléatoire. On peut donc modéliser la situation par une loi binomiale.
\[
X \sim \mathcal{B}(3; 0.76)
\]
\item Probabilité qu'une seule personne fasse sonner le portique. On voit qu'il y a 3 branches qui correspondent à cette situation dont
\[
P(X = 1) = 3 \times 0.1^1 \times 0.9^2 \approx 0.243
\]
\item
\[
P(X = 0) = 0.9^3 \approx 0.729
\]
\[
P(X \geq 2) = P(X = 2) + P(X = 3) = 3 \times 0.1^2 \times 0.9^1 + 0.1^3 \approx 0.028
\]
\item Il faut d'abord tracer le tableau résumant la loi de probabilité:
\begin{center}
\begin{tabular}{|c|*{4}{c|}}
\hline
Valeur & 0 & 1 & 2 & 3 \\
\hline
Probabilité & $0.729$ & $0.243$ & $0.027$ &$0.001$ \\
\hline
\end{tabular}
\end{center}
On peut alors calculer l'espérance
\[
E[X] = 0 \times 0.729 + 1 \times 0.243 + 2 \times 0.027 + 3 \times 0.001 = 0.3
\]
On peut donc estimer qu'il y aura en moyenne $0.3$ personnes qui feront sonner le portique sur les 3 personnes.
\end{enumerate}
\end{solution}
\begin{exercise}[subtitle={Équation puissance}]
Résoudre les équations et inéquations suivantes
\begin{multicols}{2}
\begin{enumerate}
\item $10^x = 25$
\item $12^x = 21$
\item $0.77^x \leq 22$
\item $6 \times 0.4^x = 40$
\end{enumerate}
\end{multicols}
\end{exercise}
\begin{solution}
Les solutions ci-dessous ne sont pas justifiée car l'ordinateur ne sait pas faire. Par contre, vous vous devez savoir justifier vos réponses!
\begin{enumerate}
\item $x = \log(25)$
\item $x = \frac{\log(21)}{\log(12)}$
\item Il faut faire attention quand on divise par un log car ce dernier peut être négatif ce qui est le cas ici. Il faut donc pense à changer le sens de l'inégalité.
$x \geq \frac{\log(22)}{\log(0.77)}$
\item Il faut penser à faire la division à par $6$ avant d'utiliser le log car sinon, on ne peut pas utiliser la formule $\log(a^n) = n\times \log(a)$.
$x = \frac{\log(6.67)}{\log(0.4)}$
\end{enumerate}
\end{solution}
\begin{exercise}[subtitle={Étude de fonctions}]
Soit $f(x) = 4x^3 - 132x^2 - 5460x + 39$ une fonction définie sur $\R$.
\begin{enumerate}
\item Calculer $f'(x)$ la dérivée de $f(x)$.
\item Calculer $f'(35)$ et $f'(-13)$.
\item En déduire une forme factorisée de $f'(x)$.
\item Étudier le signe de $f'(x)$ et en déduire les variations de $f(x)$.
\item Est-ce que la fonction $f(x)$ admet un maximum ou un minimum? Si oui, calculer sa valeur.
\end{enumerate}
\end{exercise}
\begin{solution}
\begin{enumerate}
\item Dérivée de $f(x)$: $f'(x) = 12x^2 - 264x - 5460$
\item
\begin{align*}
f'(35) &= 12 \times 35^{2} - 264 \times 35 - 5460\\&= 12 \times 1225 - 9240 - 5460\\&= 14700 - 14700\\&= 0
\end{align*}
\begin{align*}
f'(-13) &= 12 \times - 13^{2} - 264(- 13) - 5460\\&= 12 \times 169 + 3432 - 5460\\&= 2028 - 2028\\&= 0
\end{align*}
Donc $x = 35$ et $x=-13$ sont des racines de $f'(x) = 12x^2 - 264x - 5460$.
\item On en déduit la forme factorisée suivante
\[
f'(x) = 12 (x - 35)(x--13)
\]
\item Pas de correction disponible
\item À causes des branches extérieurs, la fonction $f(x)$ n'a pas de maximum ou de minimum.
\end{enumerate}
\end{solution}
%\printsolutionstype{exercise}
\end{document}
%%% Local Variables:
%%% mode: latex
%%% TeX-master: "master"
%%% End:

View File

@ -0,0 +1,203 @@
\documentclass[a5paper,10pt]{article}
\usepackage{myXsim}
\usepackage{tasks}
% Title Page
\title{DM2 \hfill GERMAIN Anaïs}
\tribe{TST}
\date{\hfillÀ render pour le Mercredi 24 février}
\xsimsetup{
solution/print = true
}
\begin{document}
\maketitle
\begin{exercise}[subtitle={Loi binomiale}]
Trois personnes s'apprêtent à passer le portique de sécurité. On suppose que pour chaque personne la probabilité que le portique sonne est égale à $0.68$.
Soit $X$ la variable aléatoire donnant le nombre de personnes faisant sonner le portique, parmi les 3 personnes de ce groupe.
\begin{enumerate}
\item Tracer l'arbre représentant le situation.
\item Justifier que $X$ suit une loi binomiale dont on précisera les paramètres.
\item Quelle est la probabilité qu'une seule personne fasse sonner le portique?
\item Calculer puis interpréter les probabilités suivantes
\[
P(X = 0) \qquad \qquad P(X \geq 2)
\]
\item Calculer l'espérance de $X$ et interpréter le résultat.
\end{enumerate}
\end{exercise}
\begin{solution}
\begin{enumerate}
\item
\begin{tikzpicture}[sloped]
\node {.}
child {node {$0$}
child {node {$0$}
child {node {$0$}
edge from parent
node[above] {0.32}
}
child {node {$1$}
edge from parent
node[above] {0.68}
}
edge from parent
node[above] {0.32}
}
child[missing] {}
child {node {$1$}
child {node {$0$}
edge from parent
node[above] {0.32}
}
child {node {$1$}
edge from parent
node[above] {0.68}
}
edge from parent
node[above] {0.32}
}
edge from parent
node[above] {0.32}
}
child[missing] {}
child[missing] {}
child[missing] {}
child { node {$1$}
child {node {$0$}
child {node {$0$}
edge from parent
node[above] {0.32}
}
child {node {$1$}
edge from parent
node[above] {0.68}
}
edge from parent
node[above] {0.32}
}
child[missing] {}
child {node {$1$}
child {node {$0$}
edge from parent
node[above] {0.32}
}
child {node {$1$}
edge from parent
node[above] {0.68}
}
edge from parent
node[above] {0.32}
}
edge from parent
node[above] {0.68}
} ;
\end{tikzpicture}
\item Chaque personne a 2 possibilités (1: fait sonner ou 2: ne fait pas sonner) et l'on fait passer 3 personnes ce qui correspond à une répétition identique et aléatoire. On peut donc modéliser la situation par une loi binomiale.
\[
X \sim \mathcal{B}(3; 0.76)
\]
\item Probabilité qu'une seule personne fasse sonner le portique. On voit qu'il y a 3 branches qui correspondent à cette situation dont
\[
P(X = 1) = 3 \times 0.68^1 \times 0.32^2 \approx 0.209
\]
\item
\[
P(X = 0) = 0.32^3 \approx 0.033
\]
\[
P(X \geq 2) = P(X = 2) + P(X = 3) = 3 \times 0.68^2 \times 0.32^1 + 0.68^3 \approx 0.758
\]
\item Il faut d'abord tracer le tableau résumant la loi de probabilité:
\begin{center}
\begin{tabular}{|c|*{4}{c|}}
\hline
Valeur & 0 & 1 & 2 & 3 \\
\hline
Probabilité & $0.033$ & $0.209$ & $0.444$ &$0.314$ \\
\hline
\end{tabular}
\end{center}
On peut alors calculer l'espérance
\[
E[X] = 0 \times 0.033 + 1 \times 0.209 + 2 \times 0.444 + 3 \times 0.314 = 2.04
\]
On peut donc estimer qu'il y aura en moyenne $2.04$ personnes qui feront sonner le portique sur les 3 personnes.
\end{enumerate}
\end{solution}
\begin{exercise}[subtitle={Équation puissance}]
Résoudre les équations et inéquations suivantes
\begin{multicols}{2}
\begin{enumerate}
\item $10^x = 48$
\item $2^x = 42$
\item $0.47^x \leq 46$
\item $7 \times 0.37^x = 45$
\end{enumerate}
\end{multicols}
\end{exercise}
\begin{solution}
Les solutions ci-dessous ne sont pas justifiée car l'ordinateur ne sait pas faire. Par contre, vous vous devez savoir justifier vos réponses!
\begin{enumerate}
\item $x = \log(48)$
\item $x = \frac{\log(42)}{\log(2)}$
\item Il faut faire attention quand on divise par un log car ce dernier peut être négatif ce qui est le cas ici. Il faut donc pense à changer le sens de l'inégalité.
$x \geq \frac{\log(46)}{\log(0.47)}$
\item Il faut penser à faire la division à par $7$ avant d'utiliser le log car sinon, on ne peut pas utiliser la formule $\log(a^n) = n\times \log(a)$.
$x = \frac{\log(6.43)}{\log(0.37)}$
\end{enumerate}
\end{solution}
\begin{exercise}[subtitle={Étude de fonctions}]
Soit $f(x) = - 7x^3 + 472.5x^2 + 4116x + 48$ une fonction définie sur $\R$.
\begin{enumerate}
\item Calculer $f'(x)$ la dérivée de $f(x)$.
\item Calculer $f'(49)$ et $f'(-4)$.
\item En déduire une forme factorisée de $f'(x)$.
\item Étudier le signe de $f'(x)$ et en déduire les variations de $f(x)$.
\item Est-ce que la fonction $f(x)$ admet un maximum ou un minimum? Si oui, calculer sa valeur.
\end{enumerate}
\end{exercise}
\begin{solution}
\begin{enumerate}
\item Dérivée de $f(x)$: $f'(x) = - 21x^2 + 945x + 4116$
\item
\begin{align*}
f'(49) &= - 21 \times 49^{2} + 945 \times 49 + 4116\\&= - 21 \times 2401 + 46305 + 4116\\&= - 50421 + 50421\\&= 0
\end{align*}
\begin{align*}
f'(-4) &= - 21 \times - 4^{2} + 945(- 4) + 4116\\&= - 21 \times 16 - 3780 + 4116\\&= - 336 + 336\\&= 0
\end{align*}
Donc $x = 49$ et $x=-4$ sont des racines de $f'(x) = - 21x^2 + 945x + 4116$.
\item On en déduit la forme factorisée suivante
\[
f'(x) = -21 (x - 49)(x--4)
\]
\item Pas de correction disponible
\item À causes des branches extérieurs, la fonction $f(x)$ n'a pas de maximum ou de minimum.
\end{enumerate}
\end{solution}
%\printsolutionstype{exercise}
\end{document}
%%% Local Variables:
%%% mode: latex
%%% TeX-master: "master"
%%% End:

View File

@ -0,0 +1,203 @@
\documentclass[a5paper,10pt]{article}
\usepackage{myXsim}
\usepackage{tasks}
% Title Page
\title{DM2 \hfill HADJRAS Mohcine}
\tribe{TST}
\date{\hfillÀ render pour le Mercredi 24 février}
\xsimsetup{
solution/print = true
}
\begin{document}
\maketitle
\begin{exercise}[subtitle={Loi binomiale}]
Trois personnes s'apprêtent à passer le portique de sécurité. On suppose que pour chaque personne la probabilité que le portique sonne est égale à $0.31$.
Soit $X$ la variable aléatoire donnant le nombre de personnes faisant sonner le portique, parmi les 3 personnes de ce groupe.
\begin{enumerate}
\item Tracer l'arbre représentant le situation.
\item Justifier que $X$ suit une loi binomiale dont on précisera les paramètres.
\item Quelle est la probabilité qu'une seule personne fasse sonner le portique?
\item Calculer puis interpréter les probabilités suivantes
\[
P(X = 0) \qquad \qquad P(X \geq 2)
\]
\item Calculer l'espérance de $X$ et interpréter le résultat.
\end{enumerate}
\end{exercise}
\begin{solution}
\begin{enumerate}
\item
\begin{tikzpicture}[sloped]
\node {.}
child {node {$0$}
child {node {$0$}
child {node {$0$}
edge from parent
node[above] {0.69}
}
child {node {$1$}
edge from parent
node[above] {0.31}
}
edge from parent
node[above] {0.69}
}
child[missing] {}
child {node {$1$}
child {node {$0$}
edge from parent
node[above] {0.69}
}
child {node {$1$}
edge from parent
node[above] {0.31}
}
edge from parent
node[above] {0.69}
}
edge from parent
node[above] {0.69}
}
child[missing] {}
child[missing] {}
child[missing] {}
child { node {$1$}
child {node {$0$}
child {node {$0$}
edge from parent
node[above] {0.69}
}
child {node {$1$}
edge from parent
node[above] {0.31}
}
edge from parent
node[above] {0.69}
}
child[missing] {}
child {node {$1$}
child {node {$0$}
edge from parent
node[above] {0.69}
}
child {node {$1$}
edge from parent
node[above] {0.31}
}
edge from parent
node[above] {0.69}
}
edge from parent
node[above] {0.31}
} ;
\end{tikzpicture}
\item Chaque personne a 2 possibilités (1: fait sonner ou 2: ne fait pas sonner) et l'on fait passer 3 personnes ce qui correspond à une répétition identique et aléatoire. On peut donc modéliser la situation par une loi binomiale.
\[
X \sim \mathcal{B}(3; 0.76)
\]
\item Probabilité qu'une seule personne fasse sonner le portique. On voit qu'il y a 3 branches qui correspondent à cette situation dont
\[
P(X = 1) = 3 \times 0.31^1 \times 0.69^2 \approx 0.443
\]
\item
\[
P(X = 0) = 0.69^3 \approx 0.329
\]
\[
P(X \geq 2) = P(X = 2) + P(X = 3) = 3 \times 0.31^2 \times 0.69^1 + 0.31^3 \approx 0.229
\]
\item Il faut d'abord tracer le tableau résumant la loi de probabilité:
\begin{center}
\begin{tabular}{|c|*{4}{c|}}
\hline
Valeur & 0 & 1 & 2 & 3 \\
\hline
Probabilité & $0.329$ & $0.443$ & $0.199$ &$0.03$ \\
\hline
\end{tabular}
\end{center}
On peut alors calculer l'espérance
\[
E[X] = 0 \times 0.329 + 1 \times 0.443 + 2 \times 0.199 + 3 \times 0.03 = 0.93
\]
On peut donc estimer qu'il y aura en moyenne $0.93$ personnes qui feront sonner le portique sur les 3 personnes.
\end{enumerate}
\end{solution}
\begin{exercise}[subtitle={Équation puissance}]
Résoudre les équations et inéquations suivantes
\begin{multicols}{2}
\begin{enumerate}
\item $10^x = 10$
\item $11^x = 20$
\item $0.09^x \leq 22$
\item $6 \times 0.16^x = 45$
\end{enumerate}
\end{multicols}
\end{exercise}
\begin{solution}
Les solutions ci-dessous ne sont pas justifiée car l'ordinateur ne sait pas faire. Par contre, vous vous devez savoir justifier vos réponses!
\begin{enumerate}
\item $x = \log(10)$
\item $x = \frac{\log(20)}{\log(11)}$
\item Il faut faire attention quand on divise par un log car ce dernier peut être négatif ce qui est le cas ici. Il faut donc pense à changer le sens de l'inégalité.
$x \geq \frac{\log(22)}{\log(0.09)}$
\item Il faut penser à faire la division à par $6$ avant d'utiliser le log car sinon, on ne peut pas utiliser la formule $\log(a^n) = n\times \log(a)$.
$x = \frac{\log(7.5)}{\log(0.16)}$
\end{enumerate}
\end{solution}
\begin{exercise}[subtitle={Étude de fonctions}]
Soit $f(x) = 3x^3 - 225x^2 + 4896x + 16$ une fonction définie sur $\R$.
\begin{enumerate}
\item Calculer $f'(x)$ la dérivée de $f(x)$.
\item Calculer $f'(34)$ et $f'(16)$.
\item En déduire une forme factorisée de $f'(x)$.
\item Étudier le signe de $f'(x)$ et en déduire les variations de $f(x)$.
\item Est-ce que la fonction $f(x)$ admet un maximum ou un minimum? Si oui, calculer sa valeur.
\end{enumerate}
\end{exercise}
\begin{solution}
\begin{enumerate}
\item Dérivée de $f(x)$: $f'(x) = 9x^2 - 450x + 4896$
\item
\begin{align*}
f'(34) &= 9 \times 34^{2} - 450 \times 34 + 4896\\&= 9 \times 1156 - 15300 + 4896\\&= 10404 - 10404\\&= 0
\end{align*}
\begin{align*}
f'(16) &= 9 \times 16^{2} - 450 \times 16 + 4896\\&= 9 \times 256 - 7200 + 4896\\&= 2304 - 2304\\&= 0
\end{align*}
Donc $x = 34$ et $x=16$ sont des racines de $f'(x) = 9x^2 - 450x + 4896$.
\item On en déduit la forme factorisée suivante
\[
f'(x) = 9 (x - 34)(x-16)
\]
\item Pas de correction disponible
\item À causes des branches extérieurs, la fonction $f(x)$ n'a pas de maximum ou de minimum.
\end{enumerate}
\end{solution}
%\printsolutionstype{exercise}
\end{document}
%%% Local Variables:
%%% mode: latex
%%% TeX-master: "master"
%%% End:

View File

@ -0,0 +1,203 @@
\documentclass[a5paper,10pt]{article}
\usepackage{myXsim}
\usepackage{tasks}
% Title Page
\title{DM2 \hfill HENRIST Maxime}
\tribe{TST}
\date{\hfillÀ render pour le Mercredi 24 février}
\xsimsetup{
solution/print = true
}
\begin{document}
\maketitle
\begin{exercise}[subtitle={Loi binomiale}]
Trois personnes s'apprêtent à passer le portique de sécurité. On suppose que pour chaque personne la probabilité que le portique sonne est égale à $0.41$.
Soit $X$ la variable aléatoire donnant le nombre de personnes faisant sonner le portique, parmi les 3 personnes de ce groupe.
\begin{enumerate}
\item Tracer l'arbre représentant le situation.
\item Justifier que $X$ suit une loi binomiale dont on précisera les paramètres.
\item Quelle est la probabilité qu'une seule personne fasse sonner le portique?
\item Calculer puis interpréter les probabilités suivantes
\[
P(X = 0) \qquad \qquad P(X \geq 2)
\]
\item Calculer l'espérance de $X$ et interpréter le résultat.
\end{enumerate}
\end{exercise}
\begin{solution}
\begin{enumerate}
\item
\begin{tikzpicture}[sloped]
\node {.}
child {node {$0$}
child {node {$0$}
child {node {$0$}
edge from parent
node[above] {0.59}
}
child {node {$1$}
edge from parent
node[above] {0.41}
}
edge from parent
node[above] {0.59}
}
child[missing] {}
child {node {$1$}
child {node {$0$}
edge from parent
node[above] {0.59}
}
child {node {$1$}
edge from parent
node[above] {0.41}
}
edge from parent
node[above] {0.59}
}
edge from parent
node[above] {0.59}
}
child[missing] {}
child[missing] {}
child[missing] {}
child { node {$1$}
child {node {$0$}
child {node {$0$}
edge from parent
node[above] {0.59}
}
child {node {$1$}
edge from parent
node[above] {0.41}
}
edge from parent
node[above] {0.59}
}
child[missing] {}
child {node {$1$}
child {node {$0$}
edge from parent
node[above] {0.59}
}
child {node {$1$}
edge from parent
node[above] {0.41}
}
edge from parent
node[above] {0.59}
}
edge from parent
node[above] {0.41}
} ;
\end{tikzpicture}
\item Chaque personne a 2 possibilités (1: fait sonner ou 2: ne fait pas sonner) et l'on fait passer 3 personnes ce qui correspond à une répétition identique et aléatoire. On peut donc modéliser la situation par une loi binomiale.
\[
X \sim \mathcal{B}(3; 0.76)
\]
\item Probabilité qu'une seule personne fasse sonner le portique. On voit qu'il y a 3 branches qui correspondent à cette situation dont
\[
P(X = 1) = 3 \times 0.41^1 \times 0.59^2 \approx 0.428
\]
\item
\[
P(X = 0) = 0.59^3 \approx 0.205
\]
\[
P(X \geq 2) = P(X = 2) + P(X = 3) = 3 \times 0.41^2 \times 0.59^1 + 0.41^3 \approx 0.367
\]
\item Il faut d'abord tracer le tableau résumant la loi de probabilité:
\begin{center}
\begin{tabular}{|c|*{4}{c|}}
\hline
Valeur & 0 & 1 & 2 & 3 \\
\hline
Probabilité & $0.205$ & $0.428$ & $0.298$ &$0.069$ \\
\hline
\end{tabular}
\end{center}
On peut alors calculer l'espérance
\[
E[X] = 0 \times 0.205 + 1 \times 0.428 + 2 \times 0.298 + 3 \times 0.069 = 1.23
\]
On peut donc estimer qu'il y aura en moyenne $1.23$ personnes qui feront sonner le portique sur les 3 personnes.
\end{enumerate}
\end{solution}
\begin{exercise}[subtitle={Équation puissance}]
Résoudre les équations et inéquations suivantes
\begin{multicols}{2}
\begin{enumerate}
\item $10^x = 24$
\item $14^x = 16$
\item $0.35^x \leq 34$
\item $8 \times 0.25^x = 36$
\end{enumerate}
\end{multicols}
\end{exercise}
\begin{solution}
Les solutions ci-dessous ne sont pas justifiée car l'ordinateur ne sait pas faire. Par contre, vous vous devez savoir justifier vos réponses!
\begin{enumerate}
\item $x = \log(24)$
\item $x = \frac{\log(16)}{\log(14)}$
\item Il faut faire attention quand on divise par un log car ce dernier peut être négatif ce qui est le cas ici. Il faut donc pense à changer le sens de l'inégalité.
$x \geq \frac{\log(34)}{\log(0.35)}$
\item Il faut penser à faire la division à par $8$ avant d'utiliser le log car sinon, on ne peut pas utiliser la formule $\log(a^n) = n\times \log(a)$.
$x = \frac{\log(4.5)}{\log(0.25)}$
\end{enumerate}
\end{solution}
\begin{exercise}[subtitle={Étude de fonctions}]
Soit $f(x) = 7x^3 - 346.5x^2 - 4914x + 30$ une fonction définie sur $\R$.
\begin{enumerate}
\item Calculer $f'(x)$ la dérivée de $f(x)$.
\item Calculer $f'(39)$ et $f'(-6)$.
\item En déduire une forme factorisée de $f'(x)$.
\item Étudier le signe de $f'(x)$ et en déduire les variations de $f(x)$.
\item Est-ce que la fonction $f(x)$ admet un maximum ou un minimum? Si oui, calculer sa valeur.
\end{enumerate}
\end{exercise}
\begin{solution}
\begin{enumerate}
\item Dérivée de $f(x)$: $f'(x) = 21x^2 - 693x - 4914$
\item
\begin{align*}
f'(39) &= 21 \times 39^{2} - 693 \times 39 - 4914\\&= 21 \times 1521 - 27027 - 4914\\&= 31941 - 31941\\&= 0
\end{align*}
\begin{align*}
f'(-6) &= 21 \times - 6^{2} - 693(- 6) - 4914\\&= 21 \times 36 + 4158 - 4914\\&= 756 - 756\\&= 0
\end{align*}
Donc $x = 39$ et $x=-6$ sont des racines de $f'(x) = 21x^2 - 693x - 4914$.
\item On en déduit la forme factorisée suivante
\[
f'(x) = 21 (x - 39)(x--6)
\]
\item Pas de correction disponible
\item À causes des branches extérieurs, la fonction $f(x)$ n'a pas de maximum ou de minimum.
\end{enumerate}
\end{solution}
%\printsolutionstype{exercise}
\end{document}
%%% Local Variables:
%%% mode: latex
%%% TeX-master: "master"
%%% End:

View File

@ -0,0 +1,203 @@
\documentclass[a5paper,10pt]{article}
\usepackage{myXsim}
\usepackage{tasks}
% Title Page
\title{DM2 \hfill INFANTES Antoine}
\tribe{TST}
\date{\hfillÀ render pour le Mercredi 24 février}
\xsimsetup{
solution/print = true
}
\begin{document}
\maketitle
\begin{exercise}[subtitle={Loi binomiale}]
Trois personnes s'apprêtent à passer le portique de sécurité. On suppose que pour chaque personne la probabilité que le portique sonne est égale à $0.58$.
Soit $X$ la variable aléatoire donnant le nombre de personnes faisant sonner le portique, parmi les 3 personnes de ce groupe.
\begin{enumerate}
\item Tracer l'arbre représentant le situation.
\item Justifier que $X$ suit une loi binomiale dont on précisera les paramètres.
\item Quelle est la probabilité qu'une seule personne fasse sonner le portique?
\item Calculer puis interpréter les probabilités suivantes
\[
P(X = 0) \qquad \qquad P(X \geq 2)
\]
\item Calculer l'espérance de $X$ et interpréter le résultat.
\end{enumerate}
\end{exercise}
\begin{solution}
\begin{enumerate}
\item
\begin{tikzpicture}[sloped]
\node {.}
child {node {$0$}
child {node {$0$}
child {node {$0$}
edge from parent
node[above] {0.42}
}
child {node {$1$}
edge from parent
node[above] {0.58}
}
edge from parent
node[above] {0.42}
}
child[missing] {}
child {node {$1$}
child {node {$0$}
edge from parent
node[above] {0.42}
}
child {node {$1$}
edge from parent
node[above] {0.58}
}
edge from parent
node[above] {0.42}
}
edge from parent
node[above] {0.42}
}
child[missing] {}
child[missing] {}
child[missing] {}
child { node {$1$}
child {node {$0$}
child {node {$0$}
edge from parent
node[above] {0.42}
}
child {node {$1$}
edge from parent
node[above] {0.58}
}
edge from parent
node[above] {0.42}
}
child[missing] {}
child {node {$1$}
child {node {$0$}
edge from parent
node[above] {0.42}
}
child {node {$1$}
edge from parent
node[above] {0.58}
}
edge from parent
node[above] {0.42}
}
edge from parent
node[above] {0.58}
} ;
\end{tikzpicture}
\item Chaque personne a 2 possibilités (1: fait sonner ou 2: ne fait pas sonner) et l'on fait passer 3 personnes ce qui correspond à une répétition identique et aléatoire. On peut donc modéliser la situation par une loi binomiale.
\[
X \sim \mathcal{B}(3; 0.76)
\]
\item Probabilité qu'une seule personne fasse sonner le portique. On voit qu'il y a 3 branches qui correspondent à cette situation dont
\[
P(X = 1) = 3 \times 0.58^1 \times 0.42^2 \approx 0.307
\]
\item
\[
P(X = 0) = 0.42^3 \approx 0.074
\]
\[
P(X \geq 2) = P(X = 2) + P(X = 3) = 3 \times 0.58^2 \times 0.42^1 + 0.58^3 \approx 0.619
\]
\item Il faut d'abord tracer le tableau résumant la loi de probabilité:
\begin{center}
\begin{tabular}{|c|*{4}{c|}}
\hline
Valeur & 0 & 1 & 2 & 3 \\
\hline
Probabilité & $0.074$ & $0.307$ & $0.424$ &$0.195$ \\
\hline
\end{tabular}
\end{center}
On peut alors calculer l'espérance
\[
E[X] = 0 \times 0.074 + 1 \times 0.307 + 2 \times 0.424 + 3 \times 0.195 = 1.74
\]
On peut donc estimer qu'il y aura en moyenne $1.74$ personnes qui feront sonner le portique sur les 3 personnes.
\end{enumerate}
\end{solution}
\begin{exercise}[subtitle={Équation puissance}]
Résoudre les équations et inéquations suivantes
\begin{multicols}{2}
\begin{enumerate}
\item $10^x = 1$
\item $11^x = 10$
\item $0.52^x \leq 14$
\item $5 \times 0.49^x = 45$
\end{enumerate}
\end{multicols}
\end{exercise}
\begin{solution}
Les solutions ci-dessous ne sont pas justifiée car l'ordinateur ne sait pas faire. Par contre, vous vous devez savoir justifier vos réponses!
\begin{enumerate}
\item $x = \log(1)$
\item $x = \frac{\log(10)}{\log(11)}$
\item Il faut faire attention quand on divise par un log car ce dernier peut être négatif ce qui est le cas ici. Il faut donc pense à changer le sens de l'inégalité.
$x \geq \frac{\log(14)}{\log(0.52)}$
\item Il faut penser à faire la division à par $5$ avant d'utiliser le log car sinon, on ne peut pas utiliser la formule $\log(a^n) = n\times \log(a)$.
$x = \frac{\log(9.0)}{\log(0.49)}$
\end{enumerate}
\end{solution}
\begin{exercise}[subtitle={Étude de fonctions}]
Soit $f(x) = - 9x^3 + 364.5x^2 + 756x - 21$ une fonction définie sur $\R$.
\begin{enumerate}
\item Calculer $f'(x)$ la dérivée de $f(x)$.
\item Calculer $f'(28)$ et $f'(-1)$.
\item En déduire une forme factorisée de $f'(x)$.
\item Étudier le signe de $f'(x)$ et en déduire les variations de $f(x)$.
\item Est-ce que la fonction $f(x)$ admet un maximum ou un minimum? Si oui, calculer sa valeur.
\end{enumerate}
\end{exercise}
\begin{solution}
\begin{enumerate}
\item Dérivée de $f(x)$: $f'(x) = - 27x^2 + 729x + 756$
\item
\begin{align*}
f'(28) &= - 27 \times 28^{2} + 729 \times 28 + 756\\&= - 27 \times 784 + 20412 + 756\\&= - 21168 + 21168\\&= 0
\end{align*}
\begin{align*}
f'(-1) &= - 27 \times - 1^{2} + 729(- 1) + 756\\&= - 27 \times 1 - 729 + 756\\&= - 27 + 27\\&= 0
\end{align*}
Donc $x = 28$ et $x=-1$ sont des racines de $f'(x) = - 27x^2 + 729x + 756$.
\item On en déduit la forme factorisée suivante
\[
f'(x) = -27 (x - 28)(x--1)
\]
\item Pas de correction disponible
\item À causes des branches extérieurs, la fonction $f(x)$ n'a pas de maximum ou de minimum.
\end{enumerate}
\end{solution}
%\printsolutionstype{exercise}
\end{document}
%%% Local Variables:
%%% mode: latex
%%% TeX-master: "master"
%%% End:

View File

@ -0,0 +1,203 @@
\documentclass[a5paper,10pt]{article}
\usepackage{myXsim}
\usepackage{tasks}
% Title Page
\title{DM2 \hfill MAGRO Robin}
\tribe{TST}
\date{\hfillÀ render pour le Mercredi 24 février}
\xsimsetup{
solution/print = true
}
\begin{document}
\maketitle
\begin{exercise}[subtitle={Loi binomiale}]
Trois personnes s'apprêtent à passer le portique de sécurité. On suppose que pour chaque personne la probabilité que le portique sonne est égale à $0.59$.
Soit $X$ la variable aléatoire donnant le nombre de personnes faisant sonner le portique, parmi les 3 personnes de ce groupe.
\begin{enumerate}
\item Tracer l'arbre représentant le situation.
\item Justifier que $X$ suit une loi binomiale dont on précisera les paramètres.
\item Quelle est la probabilité qu'une seule personne fasse sonner le portique?
\item Calculer puis interpréter les probabilités suivantes
\[
P(X = 0) \qquad \qquad P(X \geq 2)
\]
\item Calculer l'espérance de $X$ et interpréter le résultat.
\end{enumerate}
\end{exercise}
\begin{solution}
\begin{enumerate}
\item
\begin{tikzpicture}[sloped]
\node {.}
child {node {$0$}
child {node {$0$}
child {node {$0$}
edge from parent
node[above] {0.41}
}
child {node {$1$}
edge from parent
node[above] {0.59}
}
edge from parent
node[above] {0.41}
}
child[missing] {}
child {node {$1$}
child {node {$0$}
edge from parent
node[above] {0.41}
}
child {node {$1$}
edge from parent
node[above] {0.59}
}
edge from parent
node[above] {0.41}
}
edge from parent
node[above] {0.41}
}
child[missing] {}
child[missing] {}
child[missing] {}
child { node {$1$}
child {node {$0$}
child {node {$0$}
edge from parent
node[above] {0.41}
}
child {node {$1$}
edge from parent
node[above] {0.59}
}
edge from parent
node[above] {0.41}
}
child[missing] {}
child {node {$1$}
child {node {$0$}
edge from parent
node[above] {0.41}
}
child {node {$1$}
edge from parent
node[above] {0.59}
}
edge from parent
node[above] {0.41}
}
edge from parent
node[above] {0.59}
} ;
\end{tikzpicture}
\item Chaque personne a 2 possibilités (1: fait sonner ou 2: ne fait pas sonner) et l'on fait passer 3 personnes ce qui correspond à une répétition identique et aléatoire. On peut donc modéliser la situation par une loi binomiale.
\[
X \sim \mathcal{B}(3; 0.76)
\]
\item Probabilité qu'une seule personne fasse sonner le portique. On voit qu'il y a 3 branches qui correspondent à cette situation dont
\[
P(X = 1) = 3 \times 0.59^1 \times 0.41^2 \approx 0.298
\]
\item
\[
P(X = 0) = 0.41^3 \approx 0.069
\]
\[
P(X \geq 2) = P(X = 2) + P(X = 3) = 3 \times 0.59^2 \times 0.41^1 + 0.59^3 \approx 0.633
\]
\item Il faut d'abord tracer le tableau résumant la loi de probabilité:
\begin{center}
\begin{tabular}{|c|*{4}{c|}}
\hline
Valeur & 0 & 1 & 2 & 3 \\
\hline
Probabilité & $0.069$ & $0.298$ & $0.428$ &$0.205$ \\
\hline
\end{tabular}
\end{center}
On peut alors calculer l'espérance
\[
E[X] = 0 \times 0.069 + 1 \times 0.298 + 2 \times 0.428 + 3 \times 0.205 = 1.77
\]
On peut donc estimer qu'il y aura en moyenne $1.77$ personnes qui feront sonner le portique sur les 3 personnes.
\end{enumerate}
\end{solution}
\begin{exercise}[subtitle={Équation puissance}]
Résoudre les équations et inéquations suivantes
\begin{multicols}{2}
\begin{enumerate}
\item $10^x = 5$
\item $15^x = 1$
\item $0.57^x \leq 45$
\item $3 \times 0.51^x = 21$
\end{enumerate}
\end{multicols}
\end{exercise}
\begin{solution}
Les solutions ci-dessous ne sont pas justifiée car l'ordinateur ne sait pas faire. Par contre, vous vous devez savoir justifier vos réponses!
\begin{enumerate}
\item $x = \log(5)$
\item $x = \frac{\log(1)}{\log(15)}$
\item Il faut faire attention quand on divise par un log car ce dernier peut être négatif ce qui est le cas ici. Il faut donc pense à changer le sens de l'inégalité.
$x \geq \frac{\log(45)}{\log(0.57)}$
\item Il faut penser à faire la division à par $3$ avant d'utiliser le log car sinon, on ne peut pas utiliser la formule $\log(a^n) = n\times \log(a)$.
$x = \frac{\log(7.0)}{\log(0.51)}$
\end{enumerate}
\end{solution}
\begin{exercise}[subtitle={Étude de fonctions}]
Soit $f(x) = - x^3 + 43.5x^2 + 510x - 37$ une fonction définie sur $\R$.
\begin{enumerate}
\item Calculer $f'(x)$ la dérivée de $f(x)$.
\item Calculer $f'(34)$ et $f'(-5)$.
\item En déduire une forme factorisée de $f'(x)$.
\item Étudier le signe de $f'(x)$ et en déduire les variations de $f(x)$.
\item Est-ce que la fonction $f(x)$ admet un maximum ou un minimum? Si oui, calculer sa valeur.
\end{enumerate}
\end{exercise}
\begin{solution}
\begin{enumerate}
\item Dérivée de $f(x)$: $f'(x) = - 3x^2 + 87x + 510$
\item
\begin{align*}
f'(34) &= - 3 \times 34^{2} + 87 \times 34 + 510\\&= - 3 \times 1156 + 2958 + 510\\&= - 3468 + 3468\\&= 0
\end{align*}
\begin{align*}
f'(-5) &= - 3 \times - 5^{2} + 87(- 5) + 510\\&= - 3 \times 25 - 435 + 510\\&= - 75 + 75\\&= 0
\end{align*}
Donc $x = 34$ et $x=-5$ sont des racines de $f'(x) = - 3x^2 + 87x + 510$.
\item On en déduit la forme factorisée suivante
\[
f'(x) = -3 (x - 34)(x--5)
\]
\item Pas de correction disponible
\item À causes des branches extérieurs, la fonction $f(x)$ n'a pas de maximum ou de minimum.
\end{enumerate}
\end{solution}
%\printsolutionstype{exercise}
\end{document}
%%% Local Variables:
%%% mode: latex
%%% TeX-master: "master"
%%% End:

View File

@ -0,0 +1,203 @@
\documentclass[a5paper,10pt]{article}
\usepackage{myXsim}
\usepackage{tasks}
% Title Page
\title{DM2 \hfill MORFIN Chloé}
\tribe{TST}
\date{\hfillÀ render pour le Mercredi 24 février}
\xsimsetup{
solution/print = true
}
\begin{document}
\maketitle
\begin{exercise}[subtitle={Loi binomiale}]
Trois personnes s'apprêtent à passer le portique de sécurité. On suppose que pour chaque personne la probabilité que le portique sonne est égale à $0.96$.
Soit $X$ la variable aléatoire donnant le nombre de personnes faisant sonner le portique, parmi les 3 personnes de ce groupe.
\begin{enumerate}
\item Tracer l'arbre représentant le situation.
\item Justifier que $X$ suit une loi binomiale dont on précisera les paramètres.
\item Quelle est la probabilité qu'une seule personne fasse sonner le portique?
\item Calculer puis interpréter les probabilités suivantes
\[
P(X = 0) \qquad \qquad P(X \geq 2)
\]
\item Calculer l'espérance de $X$ et interpréter le résultat.
\end{enumerate}
\end{exercise}
\begin{solution}
\begin{enumerate}
\item
\begin{tikzpicture}[sloped]
\node {.}
child {node {$0$}
child {node {$0$}
child {node {$0$}
edge from parent
node[above] {0.04}
}
child {node {$1$}
edge from parent
node[above] {0.96}
}
edge from parent
node[above] {0.04}
}
child[missing] {}
child {node {$1$}
child {node {$0$}
edge from parent
node[above] {0.04}
}
child {node {$1$}
edge from parent
node[above] {0.96}
}
edge from parent
node[above] {0.04}
}
edge from parent
node[above] {0.04}
}
child[missing] {}
child[missing] {}
child[missing] {}
child { node {$1$}
child {node {$0$}
child {node {$0$}
edge from parent
node[above] {0.04}
}
child {node {$1$}
edge from parent
node[above] {0.96}
}
edge from parent
node[above] {0.04}
}
child[missing] {}
child {node {$1$}
child {node {$0$}
edge from parent
node[above] {0.04}
}
child {node {$1$}
edge from parent
node[above] {0.96}
}
edge from parent
node[above] {0.04}
}
edge from parent
node[above] {0.96}
} ;
\end{tikzpicture}
\item Chaque personne a 2 possibilités (1: fait sonner ou 2: ne fait pas sonner) et l'on fait passer 3 personnes ce qui correspond à une répétition identique et aléatoire. On peut donc modéliser la situation par une loi binomiale.
\[
X \sim \mathcal{B}(3; 0.76)
\]
\item Probabilité qu'une seule personne fasse sonner le portique. On voit qu'il y a 3 branches qui correspondent à cette situation dont
\[
P(X = 1) = 3 \times 0.96^1 \times 0.04^2 \approx 0.005
\]
\item
\[
P(X = 0) = 0.04^3 \approx 0.0
\]
\[
P(X \geq 2) = P(X = 2) + P(X = 3) = 3 \times 0.96^2 \times 0.04^1 + 0.96^3 \approx 0.996
\]
\item Il faut d'abord tracer le tableau résumant la loi de probabilité:
\begin{center}
\begin{tabular}{|c|*{4}{c|}}
\hline
Valeur & 0 & 1 & 2 & 3 \\
\hline
Probabilité & $0.0$ & $0.005$ & $0.111$ &$0.885$ \\
\hline
\end{tabular}
\end{center}
On peut alors calculer l'espérance
\[
E[X] = 0 \times 0.0 + 1 \times 0.005 + 2 \times 0.111 + 3 \times 0.885 = 2.88
\]
On peut donc estimer qu'il y aura en moyenne $2.88$ personnes qui feront sonner le portique sur les 3 personnes.
\end{enumerate}
\end{solution}
\begin{exercise}[subtitle={Équation puissance}]
Résoudre les équations et inéquations suivantes
\begin{multicols}{2}
\begin{enumerate}
\item $10^x = 17$
\item $3^x = 31$
\item $0.44^x \leq 45$
\item $9 \times 1.0^x = 17$
\end{enumerate}
\end{multicols}
\end{exercise}
\begin{solution}
Les solutions ci-dessous ne sont pas justifiée car l'ordinateur ne sait pas faire. Par contre, vous vous devez savoir justifier vos réponses!
\begin{enumerate}
\item $x = \log(17)$
\item $x = \frac{\log(31)}{\log(3)}$
\item Il faut faire attention quand on divise par un log car ce dernier peut être négatif ce qui est le cas ici. Il faut donc pense à changer le sens de l'inégalité.
$x \geq \frac{\log(45)}{\log(0.44)}$
\item Il faut penser à faire la division à par $9$ avant d'utiliser le log car sinon, on ne peut pas utiliser la formule $\log(a^n) = n\times \log(a)$.
$x = \frac{\log(1.89)}{\log(1.0)}$
\end{enumerate}
\end{solution}
\begin{exercise}[subtitle={Étude de fonctions}]
Soit $f(x) = - 8x^3 + 324x^2 + 1392x - 9$ une fonction définie sur $\R$.
\begin{enumerate}
\item Calculer $f'(x)$ la dérivée de $f(x)$.
\item Calculer $f'(29)$ et $f'(-2)$.
\item En déduire une forme factorisée de $f'(x)$.
\item Étudier le signe de $f'(x)$ et en déduire les variations de $f(x)$.
\item Est-ce que la fonction $f(x)$ admet un maximum ou un minimum? Si oui, calculer sa valeur.
\end{enumerate}
\end{exercise}
\begin{solution}
\begin{enumerate}
\item Dérivée de $f(x)$: $f'(x) = - 24x^2 + 648x + 1392$
\item
\begin{align*}
f'(29) &= - 24 \times 29^{2} + 648 \times 29 + 1392\\&= - 24 \times 841 + 18792 + 1392\\&= - 20184 + 20184\\&= 0
\end{align*}
\begin{align*}
f'(-2) &= - 24 \times - 2^{2} + 648(- 2) + 1392\\&= - 24 \times 4 - 1296 + 1392\\&= - 96 + 96\\&= 0
\end{align*}
Donc $x = 29$ et $x=-2$ sont des racines de $f'(x) = - 24x^2 + 648x + 1392$.
\item On en déduit la forme factorisée suivante
\[
f'(x) = -24 (x - 29)(x--2)
\]
\item Pas de correction disponible
\item À causes des branches extérieurs, la fonction $f(x)$ n'a pas de maximum ou de minimum.
\end{enumerate}
\end{solution}
%\printsolutionstype{exercise}
\end{document}
%%% Local Variables:
%%% mode: latex
%%% TeX-master: "master"
%%% End:

View File

@ -0,0 +1,203 @@
\documentclass[a5paper,10pt]{article}
\usepackage{myXsim}
\usepackage{tasks}
% Title Page
\title{DM2 \hfill PERES RAMALHO Emeric}
\tribe{TST}
\date{\hfillÀ render pour le Mercredi 24 février}
\xsimsetup{
solution/print = true
}
\begin{document}
\maketitle
\begin{exercise}[subtitle={Loi binomiale}]
Trois personnes s'apprêtent à passer le portique de sécurité. On suppose que pour chaque personne la probabilité que le portique sonne est égale à $0.19$.
Soit $X$ la variable aléatoire donnant le nombre de personnes faisant sonner le portique, parmi les 3 personnes de ce groupe.
\begin{enumerate}
\item Tracer l'arbre représentant le situation.
\item Justifier que $X$ suit une loi binomiale dont on précisera les paramètres.
\item Quelle est la probabilité qu'une seule personne fasse sonner le portique?
\item Calculer puis interpréter les probabilités suivantes
\[
P(X = 0) \qquad \qquad P(X \geq 2)
\]
\item Calculer l'espérance de $X$ et interpréter le résultat.
\end{enumerate}
\end{exercise}
\begin{solution}
\begin{enumerate}
\item
\begin{tikzpicture}[sloped]
\node {.}
child {node {$0$}
child {node {$0$}
child {node {$0$}
edge from parent
node[above] {0.81}
}
child {node {$1$}
edge from parent
node[above] {0.19}
}
edge from parent
node[above] {0.81}
}
child[missing] {}
child {node {$1$}
child {node {$0$}
edge from parent
node[above] {0.81}
}
child {node {$1$}
edge from parent
node[above] {0.19}
}
edge from parent
node[above] {0.81}
}
edge from parent
node[above] {0.81}
}
child[missing] {}
child[missing] {}
child[missing] {}
child { node {$1$}
child {node {$0$}
child {node {$0$}
edge from parent
node[above] {0.81}
}
child {node {$1$}
edge from parent
node[above] {0.19}
}
edge from parent
node[above] {0.81}
}
child[missing] {}
child {node {$1$}
child {node {$0$}
edge from parent
node[above] {0.81}
}
child {node {$1$}
edge from parent
node[above] {0.19}
}
edge from parent
node[above] {0.81}
}
edge from parent
node[above] {0.19}
} ;
\end{tikzpicture}
\item Chaque personne a 2 possibilités (1: fait sonner ou 2: ne fait pas sonner) et l'on fait passer 3 personnes ce qui correspond à une répétition identique et aléatoire. On peut donc modéliser la situation par une loi binomiale.
\[
X \sim \mathcal{B}(3; 0.76)
\]
\item Probabilité qu'une seule personne fasse sonner le portique. On voit qu'il y a 3 branches qui correspondent à cette situation dont
\[
P(X = 1) = 3 \times 0.19^1 \times 0.81^2 \approx 0.374
\]
\item
\[
P(X = 0) = 0.81^3 \approx 0.531
\]
\[
P(X \geq 2) = P(X = 2) + P(X = 3) = 3 \times 0.19^2 \times 0.81^1 + 0.19^3 \approx 0.095
\]
\item Il faut d'abord tracer le tableau résumant la loi de probabilité:
\begin{center}
\begin{tabular}{|c|*{4}{c|}}
\hline
Valeur & 0 & 1 & 2 & 3 \\
\hline
Probabilité & $0.531$ & $0.374$ & $0.088$ &$0.007$ \\
\hline
\end{tabular}
\end{center}
On peut alors calculer l'espérance
\[
E[X] = 0 \times 0.531 + 1 \times 0.374 + 2 \times 0.088 + 3 \times 0.007 = 0.57
\]
On peut donc estimer qu'il y aura en moyenne $0.57$ personnes qui feront sonner le portique sur les 3 personnes.
\end{enumerate}
\end{solution}
\begin{exercise}[subtitle={Équation puissance}]
Résoudre les équations et inéquations suivantes
\begin{multicols}{2}
\begin{enumerate}
\item $10^x = 5$
\item $4^x = 39$
\item $0.95^x \leq 21$
\item $10 \times 0.74^x = 16$
\end{enumerate}
\end{multicols}
\end{exercise}
\begin{solution}
Les solutions ci-dessous ne sont pas justifiée car l'ordinateur ne sait pas faire. Par contre, vous vous devez savoir justifier vos réponses!
\begin{enumerate}
\item $x = \log(5)$
\item $x = \frac{\log(39)}{\log(4)}$
\item Il faut faire attention quand on divise par un log car ce dernier peut être négatif ce qui est le cas ici. Il faut donc pense à changer le sens de l'inégalité.
$x \geq \frac{\log(21)}{\log(0.95)}$
\item Il faut penser à faire la division à par $10$ avant d'utiliser le log car sinon, on ne peut pas utiliser la formule $\log(a^n) = n\times \log(a)$.
$x = \frac{\log(1.6)}{\log(0.74)}$
\end{enumerate}
\end{solution}
\begin{exercise}[subtitle={Étude de fonctions}]
Soit $f(x) = - 3x^3 + 108x^2 - 396x - 26$ une fonction définie sur $\R$.
\begin{enumerate}
\item Calculer $f'(x)$ la dérivée de $f(x)$.
\item Calculer $f'(22)$ et $f'(2)$.
\item En déduire une forme factorisée de $f'(x)$.
\item Étudier le signe de $f'(x)$ et en déduire les variations de $f(x)$.
\item Est-ce que la fonction $f(x)$ admet un maximum ou un minimum? Si oui, calculer sa valeur.
\end{enumerate}
\end{exercise}
\begin{solution}
\begin{enumerate}
\item Dérivée de $f(x)$: $f'(x) = - 9x^2 + 216x - 396$
\item
\begin{align*}
f'(22) &= - 9 \times 22^{2} + 216 \times 22 - 396\\&= - 9 \times 484 + 4752 - 396\\&= - 4356 + 4356\\&= 0
\end{align*}
\begin{align*}
f'(2) &= - 9 \times 2^{2} + 216 \times 2 - 396\\&= - 9 \times 4 + 432 - 396\\&= - 36 + 36\\&= 0
\end{align*}
Donc $x = 22$ et $x=2$ sont des racines de $f'(x) = - 9x^2 + 216x - 396$.
\item On en déduit la forme factorisée suivante
\[
f'(x) = -9 (x - 22)(x-2)
\]
\item Pas de correction disponible
\item À causes des branches extérieurs, la fonction $f(x)$ n'a pas de maximum ou de minimum.
\end{enumerate}
\end{solution}
%\printsolutionstype{exercise}
\end{document}
%%% Local Variables:
%%% mode: latex
%%% TeX-master: "master"
%%% End:

View File

@ -0,0 +1,203 @@
\documentclass[a5paper,10pt]{article}
\usepackage{myXsim}
\usepackage{tasks}
% Title Page
\title{DM2 \hfill RADOUAA Saleh}
\tribe{TST}
\date{\hfillÀ render pour le Mercredi 24 février}
\xsimsetup{
solution/print = true
}
\begin{document}
\maketitle
\begin{exercise}[subtitle={Loi binomiale}]
Trois personnes s'apprêtent à passer le portique de sécurité. On suppose que pour chaque personne la probabilité que le portique sonne est égale à $0.08$.
Soit $X$ la variable aléatoire donnant le nombre de personnes faisant sonner le portique, parmi les 3 personnes de ce groupe.
\begin{enumerate}
\item Tracer l'arbre représentant le situation.
\item Justifier que $X$ suit une loi binomiale dont on précisera les paramètres.
\item Quelle est la probabilité qu'une seule personne fasse sonner le portique?
\item Calculer puis interpréter les probabilités suivantes
\[
P(X = 0) \qquad \qquad P(X \geq 2)
\]
\item Calculer l'espérance de $X$ et interpréter le résultat.
\end{enumerate}
\end{exercise}
\begin{solution}
\begin{enumerate}
\item
\begin{tikzpicture}[sloped]
\node {.}
child {node {$0$}
child {node {$0$}
child {node {$0$}
edge from parent
node[above] {0.92}
}
child {node {$1$}
edge from parent
node[above] {0.08}
}
edge from parent
node[above] {0.92}
}
child[missing] {}
child {node {$1$}
child {node {$0$}
edge from parent
node[above] {0.92}
}
child {node {$1$}
edge from parent
node[above] {0.08}
}
edge from parent
node[above] {0.92}
}
edge from parent
node[above] {0.92}
}
child[missing] {}
child[missing] {}
child[missing] {}
child { node {$1$}
child {node {$0$}
child {node {$0$}
edge from parent
node[above] {0.92}
}
child {node {$1$}
edge from parent
node[above] {0.08}
}
edge from parent
node[above] {0.92}
}
child[missing] {}
child {node {$1$}
child {node {$0$}
edge from parent
node[above] {0.92}
}
child {node {$1$}
edge from parent
node[above] {0.08}
}
edge from parent
node[above] {0.92}
}
edge from parent
node[above] {0.08}
} ;
\end{tikzpicture}
\item Chaque personne a 2 possibilités (1: fait sonner ou 2: ne fait pas sonner) et l'on fait passer 3 personnes ce qui correspond à une répétition identique et aléatoire. On peut donc modéliser la situation par une loi binomiale.
\[
X \sim \mathcal{B}(3; 0.76)
\]
\item Probabilité qu'une seule personne fasse sonner le portique. On voit qu'il y a 3 branches qui correspondent à cette situation dont
\[
P(X = 1) = 3 \times 0.08^1 \times 0.92^2 \approx 0.203
\]
\item
\[
P(X = 0) = 0.92^3 \approx 0.779
\]
\[
P(X \geq 2) = P(X = 2) + P(X = 3) = 3 \times 0.08^2 \times 0.92^1 + 0.08^3 \approx 0.019
\]
\item Il faut d'abord tracer le tableau résumant la loi de probabilité:
\begin{center}
\begin{tabular}{|c|*{4}{c|}}
\hline
Valeur & 0 & 1 & 2 & 3 \\
\hline
Probabilité & $0.779$ & $0.203$ & $0.018$ &$0.001$ \\
\hline
\end{tabular}
\end{center}
On peut alors calculer l'espérance
\[
E[X] = 0 \times 0.779 + 1 \times 0.203 + 2 \times 0.018 + 3 \times 0.001 = 0.24
\]
On peut donc estimer qu'il y aura en moyenne $0.24$ personnes qui feront sonner le portique sur les 3 personnes.
\end{enumerate}
\end{solution}
\begin{exercise}[subtitle={Équation puissance}]
Résoudre les équations et inéquations suivantes
\begin{multicols}{2}
\begin{enumerate}
\item $10^x = 8$
\item $17^x = 11$
\item $0.84^x \leq 28$
\item $8 \times 0.96^x = 22$
\end{enumerate}
\end{multicols}
\end{exercise}
\begin{solution}
Les solutions ci-dessous ne sont pas justifiée car l'ordinateur ne sait pas faire. Par contre, vous vous devez savoir justifier vos réponses!
\begin{enumerate}
\item $x = \log(8)$
\item $x = \frac{\log(11)}{\log(17)}$
\item Il faut faire attention quand on divise par un log car ce dernier peut être négatif ce qui est le cas ici. Il faut donc pense à changer le sens de l'inégalité.
$x \geq \frac{\log(28)}{\log(0.84)}$
\item Il faut penser à faire la division à par $8$ avant d'utiliser le log car sinon, on ne peut pas utiliser la formule $\log(a^n) = n\times \log(a)$.
$x = \frac{\log(2.75)}{\log(0.96)}$
\end{enumerate}
\end{solution}
\begin{exercise}[subtitle={Étude de fonctions}]
Soit $f(x) = 3x^3 - 207x^2 + 1161x + 46$ une fonction définie sur $\R$.
\begin{enumerate}
\item Calculer $f'(x)$ la dérivée de $f(x)$.
\item Calculer $f'(43)$ et $f'(3)$.
\item En déduire une forme factorisée de $f'(x)$.
\item Étudier le signe de $f'(x)$ et en déduire les variations de $f(x)$.
\item Est-ce que la fonction $f(x)$ admet un maximum ou un minimum? Si oui, calculer sa valeur.
\end{enumerate}
\end{exercise}
\begin{solution}
\begin{enumerate}
\item Dérivée de $f(x)$: $f'(x) = 9x^2 - 414x + 1161$
\item
\begin{align*}
f'(43) &= 9 \times 43^{2} - 414 \times 43 + 1161\\&= 9 \times 1849 - 17802 + 1161\\&= 16641 - 16641\\&= 0
\end{align*}
\begin{align*}
f'(3) &= 9 \times 3^{2} - 414 \times 3 + 1161\\&= 9 \times 9 - 1242 + 1161\\&= 81 - 81\\&= 0
\end{align*}
Donc $x = 43$ et $x=3$ sont des racines de $f'(x) = 9x^2 - 414x + 1161$.
\item On en déduit la forme factorisée suivante
\[
f'(x) = 9 (x - 43)(x-3)
\]
\item Pas de correction disponible
\item À causes des branches extérieurs, la fonction $f(x)$ n'a pas de maximum ou de minimum.
\end{enumerate}
\end{solution}
%\printsolutionstype{exercise}
\end{document}
%%% Local Variables:
%%% mode: latex
%%% TeX-master: "master"
%%% End:

View File

@ -0,0 +1,203 @@
\documentclass[a5paper,10pt]{article}
\usepackage{myXsim}
\usepackage{tasks}
% Title Page
\title{DM2 \hfill TAY Ummuhan}
\tribe{TST}
\date{\hfillÀ render pour le Mercredi 24 février}
\xsimsetup{
solution/print = true
}
\begin{document}
\maketitle
\begin{exercise}[subtitle={Loi binomiale}]
Trois personnes s'apprêtent à passer le portique de sécurité. On suppose que pour chaque personne la probabilité que le portique sonne est égale à $0.72$.
Soit $X$ la variable aléatoire donnant le nombre de personnes faisant sonner le portique, parmi les 3 personnes de ce groupe.
\begin{enumerate}
\item Tracer l'arbre représentant le situation.
\item Justifier que $X$ suit une loi binomiale dont on précisera les paramètres.
\item Quelle est la probabilité qu'une seule personne fasse sonner le portique?
\item Calculer puis interpréter les probabilités suivantes
\[
P(X = 0) \qquad \qquad P(X \geq 2)
\]
\item Calculer l'espérance de $X$ et interpréter le résultat.
\end{enumerate}
\end{exercise}
\begin{solution}
\begin{enumerate}
\item
\begin{tikzpicture}[sloped]
\node {.}
child {node {$0$}
child {node {$0$}
child {node {$0$}
edge from parent
node[above] {0.28}
}
child {node {$1$}
edge from parent
node[above] {0.72}
}
edge from parent
node[above] {0.28}
}
child[missing] {}
child {node {$1$}
child {node {$0$}
edge from parent
node[above] {0.28}
}
child {node {$1$}
edge from parent
node[above] {0.72}
}
edge from parent
node[above] {0.28}
}
edge from parent
node[above] {0.28}
}
child[missing] {}
child[missing] {}
child[missing] {}
child { node {$1$}
child {node {$0$}
child {node {$0$}
edge from parent
node[above] {0.28}
}
child {node {$1$}
edge from parent
node[above] {0.72}
}
edge from parent
node[above] {0.28}
}
child[missing] {}
child {node {$1$}
child {node {$0$}
edge from parent
node[above] {0.28}
}
child {node {$1$}
edge from parent
node[above] {0.72}
}
edge from parent
node[above] {0.28}
}
edge from parent
node[above] {0.72}
} ;
\end{tikzpicture}
\item Chaque personne a 2 possibilités (1: fait sonner ou 2: ne fait pas sonner) et l'on fait passer 3 personnes ce qui correspond à une répétition identique et aléatoire. On peut donc modéliser la situation par une loi binomiale.
\[
X \sim \mathcal{B}(3; 0.76)
\]
\item Probabilité qu'une seule personne fasse sonner le portique. On voit qu'il y a 3 branches qui correspondent à cette situation dont
\[
P(X = 1) = 3 \times 0.72^1 \times 0.28^2 \approx 0.169
\]
\item
\[
P(X = 0) = 0.28^3 \approx 0.022
\]
\[
P(X \geq 2) = P(X = 2) + P(X = 3) = 3 \times 0.72^2 \times 0.28^1 + 0.72^3 \approx 0.808
\]
\item Il faut d'abord tracer le tableau résumant la loi de probabilité:
\begin{center}
\begin{tabular}{|c|*{4}{c|}}
\hline
Valeur & 0 & 1 & 2 & 3 \\
\hline
Probabilité & $0.022$ & $0.169$ & $0.435$ &$0.373$ \\
\hline
\end{tabular}
\end{center}
On peut alors calculer l'espérance
\[
E[X] = 0 \times 0.022 + 1 \times 0.169 + 2 \times 0.435 + 3 \times 0.373 = 2.16
\]
On peut donc estimer qu'il y aura en moyenne $2.16$ personnes qui feront sonner le portique sur les 3 personnes.
\end{enumerate}
\end{solution}
\begin{exercise}[subtitle={Équation puissance}]
Résoudre les équations et inéquations suivantes
\begin{multicols}{2}
\begin{enumerate}
\item $10^x = 41$
\item $2^x = 7$
\item $0.44^x \leq 20$
\item $3 \times 0.08^x = 24$
\end{enumerate}
\end{multicols}
\end{exercise}
\begin{solution}
Les solutions ci-dessous ne sont pas justifiée car l'ordinateur ne sait pas faire. Par contre, vous vous devez savoir justifier vos réponses!
\begin{enumerate}
\item $x = \log(41)$
\item $x = \frac{\log(7)}{\log(2)}$
\item Il faut faire attention quand on divise par un log car ce dernier peut être négatif ce qui est le cas ici. Il faut donc pense à changer le sens de l'inégalité.
$x \geq \frac{\log(20)}{\log(0.44)}$
\item Il faut penser à faire la division à par $3$ avant d'utiliser le log car sinon, on ne peut pas utiliser la formule $\log(a^n) = n\times \log(a)$.
$x = \frac{\log(8.0)}{\log(0.08)}$
\end{enumerate}
\end{solution}
\begin{exercise}[subtitle={Étude de fonctions}]
Soit $f(x) = 7x^3 - 472.5x^2 + 10374x + 15$ une fonction définie sur $\R$.
\begin{enumerate}
\item Calculer $f'(x)$ la dérivée de $f(x)$.
\item Calculer $f'(26)$ et $f'(19)$.
\item En déduire une forme factorisée de $f'(x)$.
\item Étudier le signe de $f'(x)$ et en déduire les variations de $f(x)$.
\item Est-ce que la fonction $f(x)$ admet un maximum ou un minimum? Si oui, calculer sa valeur.
\end{enumerate}
\end{exercise}
\begin{solution}
\begin{enumerate}
\item Dérivée de $f(x)$: $f'(x) = 21x^2 - 945x + 10374$
\item
\begin{align*}
f'(26) &= 21 \times 26^{2} - 945 \times 26 + 10374\\&= 21 \times 676 - 24570 + 10374\\&= 14196 - 14196\\&= 0
\end{align*}
\begin{align*}
f'(19) &= 21 \times 19^{2} - 945 \times 19 + 10374\\&= 21 \times 361 - 17955 + 10374\\&= 7581 - 7581\\&= 0
\end{align*}
Donc $x = 26$ et $x=19$ sont des racines de $f'(x) = 21x^2 - 945x + 10374$.
\item On en déduit la forme factorisée suivante
\[
f'(x) = 21 (x - 26)(x-19)
\]
\item Pas de correction disponible
\item À causes des branches extérieurs, la fonction $f(x)$ n'a pas de maximum ou de minimum.
\end{enumerate}
\end{solution}
%\printsolutionstype{exercise}
\end{document}
%%% Local Variables:
%%% mode: latex
%%% TeX-master: "master"
%%% End:

View File

@ -0,0 +1,203 @@
\documentclass[a5paper,10pt]{article}
\usepackage{myXsim}
\usepackage{tasks}
% Title Page
\title{DM2 \hfill VIALON-DUPERRON Victorien}
\tribe{TST}
\date{\hfillÀ render pour le Mercredi 24 février}
\xsimsetup{
solution/print = true
}
\begin{document}
\maketitle
\begin{exercise}[subtitle={Loi binomiale}]
Trois personnes s'apprêtent à passer le portique de sécurité. On suppose que pour chaque personne la probabilité que le portique sonne est égale à $0.18$.
Soit $X$ la variable aléatoire donnant le nombre de personnes faisant sonner le portique, parmi les 3 personnes de ce groupe.
\begin{enumerate}
\item Tracer l'arbre représentant le situation.
\item Justifier que $X$ suit une loi binomiale dont on précisera les paramètres.
\item Quelle est la probabilité qu'une seule personne fasse sonner le portique?
\item Calculer puis interpréter les probabilités suivantes
\[
P(X = 0) \qquad \qquad P(X \geq 2)
\]
\item Calculer l'espérance de $X$ et interpréter le résultat.
\end{enumerate}
\end{exercise}
\begin{solution}
\begin{enumerate}
\item
\begin{tikzpicture}[sloped]
\node {.}
child {node {$0$}
child {node {$0$}
child {node {$0$}
edge from parent
node[above] {0.82}
}
child {node {$1$}
edge from parent
node[above] {0.18}
}
edge from parent
node[above] {0.82}
}
child[missing] {}
child {node {$1$}
child {node {$0$}
edge from parent
node[above] {0.82}
}
child {node {$1$}
edge from parent
node[above] {0.18}
}
edge from parent
node[above] {0.82}
}
edge from parent
node[above] {0.82}
}
child[missing] {}
child[missing] {}
child[missing] {}
child { node {$1$}
child {node {$0$}
child {node {$0$}
edge from parent
node[above] {0.82}
}
child {node {$1$}
edge from parent
node[above] {0.18}
}
edge from parent
node[above] {0.82}
}
child[missing] {}
child {node {$1$}
child {node {$0$}
edge from parent
node[above] {0.82}
}
child {node {$1$}
edge from parent
node[above] {0.18}
}
edge from parent
node[above] {0.82}
}
edge from parent
node[above] {0.18}
} ;
\end{tikzpicture}
\item Chaque personne a 2 possibilités (1: fait sonner ou 2: ne fait pas sonner) et l'on fait passer 3 personnes ce qui correspond à une répétition identique et aléatoire. On peut donc modéliser la situation par une loi binomiale.
\[
X \sim \mathcal{B}(3; 0.76)
\]
\item Probabilité qu'une seule personne fasse sonner le portique. On voit qu'il y a 3 branches qui correspondent à cette situation dont
\[
P(X = 1) = 3 \times 0.18^1 \times 0.82^2 \approx 0.363
\]
\item
\[
P(X = 0) = 0.82^3 \approx 0.551
\]
\[
P(X \geq 2) = P(X = 2) + P(X = 3) = 3 \times 0.18^2 \times 0.82^1 + 0.18^3 \approx 0.086
\]
\item Il faut d'abord tracer le tableau résumant la loi de probabilité:
\begin{center}
\begin{tabular}{|c|*{4}{c|}}
\hline
Valeur & 0 & 1 & 2 & 3 \\
\hline
Probabilité & $0.551$ & $0.363$ & $0.08$ &$0.006$ \\
\hline
\end{tabular}
\end{center}
On peut alors calculer l'espérance
\[
E[X] = 0 \times 0.551 + 1 \times 0.363 + 2 \times 0.08 + 3 \times 0.006 = 0.54
\]
On peut donc estimer qu'il y aura en moyenne $0.54$ personnes qui feront sonner le portique sur les 3 personnes.
\end{enumerate}
\end{solution}
\begin{exercise}[subtitle={Équation puissance}]
Résoudre les équations et inéquations suivantes
\begin{multicols}{2}
\begin{enumerate}
\item $10^x = 22$
\item $4^x = 6$
\item $0.01^x \leq 35$
\item $10 \times 0.36^x = 19$
\end{enumerate}
\end{multicols}
\end{exercise}
\begin{solution}
Les solutions ci-dessous ne sont pas justifiée car l'ordinateur ne sait pas faire. Par contre, vous vous devez savoir justifier vos réponses!
\begin{enumerate}
\item $x = \log(22)$
\item $x = \frac{\log(6)}{\log(4)}$
\item Il faut faire attention quand on divise par un log car ce dernier peut être négatif ce qui est le cas ici. Il faut donc pense à changer le sens de l'inégalité.
$x \geq \frac{\log(35)}{\log(0.01)}$
\item Il faut penser à faire la division à par $10$ avant d'utiliser le log car sinon, on ne peut pas utiliser la formule $\log(a^n) = n\times \log(a)$.
$x = \frac{\log(1.9)}{\log(0.36)}$
\end{enumerate}
\end{solution}
\begin{exercise}[subtitle={Étude de fonctions}]
Soit $f(x) = - 4x^3 + 72x^2 + 1296x - 22$ une fonction définie sur $\R$.
\begin{enumerate}
\item Calculer $f'(x)$ la dérivée de $f(x)$.
\item Calculer $f'(18)$ et $f'(-6)$.
\item En déduire une forme factorisée de $f'(x)$.
\item Étudier le signe de $f'(x)$ et en déduire les variations de $f(x)$.
\item Est-ce que la fonction $f(x)$ admet un maximum ou un minimum? Si oui, calculer sa valeur.
\end{enumerate}
\end{exercise}
\begin{solution}
\begin{enumerate}
\item Dérivée de $f(x)$: $f'(x) = - 12x^2 + 144x + 1296$
\item
\begin{align*}
f'(18) &= - 12 \times 18^{2} + 144 \times 18 + 1296\\&= - 12 \times 324 + 2592 + 1296\\&= - 3888 + 3888\\&= 0
\end{align*}
\begin{align*}
f'(-6) &= - 12 \times - 6^{2} + 144(- 6) + 1296\\&= - 12 \times 36 - 864 + 1296\\&= - 432 + 432\\&= 0
\end{align*}
Donc $x = 18$ et $x=-6$ sont des racines de $f'(x) = - 12x^2 + 144x + 1296$.
\item On en déduit la forme factorisée suivante
\[
f'(x) = -12 (x - 18)(x--6)
\]
\item Pas de correction disponible
\item À causes des branches extérieurs, la fonction $f(x)$ n'a pas de maximum ou de minimum.
\end{enumerate}
\end{solution}
%\printsolutionstype{exercise}
\end{document}
%%% Local Variables:
%%% mode: latex
%%% TeX-master: "master"
%%% End:

View File

@ -0,0 +1,203 @@
\documentclass[a5paper,10pt]{article}
\usepackage{myXsim}
\usepackage{tasks}
% Title Page
\title{DM2 \hfill ZENAGUI Yanis}
\tribe{TST}
\date{\hfillÀ render pour le Mercredi 24 février}
\xsimsetup{
solution/print = true
}
\begin{document}
\maketitle
\begin{exercise}[subtitle={Loi binomiale}]
Trois personnes s'apprêtent à passer le portique de sécurité. On suppose que pour chaque personne la probabilité que le portique sonne est égale à $0.23$.
Soit $X$ la variable aléatoire donnant le nombre de personnes faisant sonner le portique, parmi les 3 personnes de ce groupe.
\begin{enumerate}
\item Tracer l'arbre représentant le situation.
\item Justifier que $X$ suit une loi binomiale dont on précisera les paramètres.
\item Quelle est la probabilité qu'une seule personne fasse sonner le portique?
\item Calculer puis interpréter les probabilités suivantes
\[
P(X = 0) \qquad \qquad P(X \geq 2)
\]
\item Calculer l'espérance de $X$ et interpréter le résultat.
\end{enumerate}
\end{exercise}
\begin{solution}
\begin{enumerate}
\item
\begin{tikzpicture}[sloped]
\node {.}
child {node {$0$}
child {node {$0$}
child {node {$0$}
edge from parent
node[above] {0.77}
}
child {node {$1$}
edge from parent
node[above] {0.23}
}
edge from parent
node[above] {0.77}
}
child[missing] {}
child {node {$1$}
child {node {$0$}
edge from parent
node[above] {0.77}
}
child {node {$1$}
edge from parent
node[above] {0.23}
}
edge from parent
node[above] {0.77}
}
edge from parent
node[above] {0.77}
}
child[missing] {}
child[missing] {}
child[missing] {}
child { node {$1$}
child {node {$0$}
child {node {$0$}
edge from parent
node[above] {0.77}
}
child {node {$1$}
edge from parent
node[above] {0.23}
}
edge from parent
node[above] {0.77}
}
child[missing] {}
child {node {$1$}
child {node {$0$}
edge from parent
node[above] {0.77}
}
child {node {$1$}
edge from parent
node[above] {0.23}
}
edge from parent
node[above] {0.77}
}
edge from parent
node[above] {0.23}
} ;
\end{tikzpicture}
\item Chaque personne a 2 possibilités (1: fait sonner ou 2: ne fait pas sonner) et l'on fait passer 3 personnes ce qui correspond à une répétition identique et aléatoire. On peut donc modéliser la situation par une loi binomiale.
\[
X \sim \mathcal{B}(3; 0.76)
\]
\item Probabilité qu'une seule personne fasse sonner le portique. On voit qu'il y a 3 branches qui correspondent à cette situation dont
\[
P(X = 1) = 3 \times 0.23^1 \times 0.77^2 \approx 0.409
\]
\item
\[
P(X = 0) = 0.77^3 \approx 0.457
\]
\[
P(X \geq 2) = P(X = 2) + P(X = 3) = 3 \times 0.23^2 \times 0.77^1 + 0.23^3 \approx 0.134
\]
\item Il faut d'abord tracer le tableau résumant la loi de probabilité:
\begin{center}
\begin{tabular}{|c|*{4}{c|}}
\hline
Valeur & 0 & 1 & 2 & 3 \\
\hline
Probabilité & $0.457$ & $0.409$ & $0.122$ &$0.012$ \\
\hline
\end{tabular}
\end{center}
On peut alors calculer l'espérance
\[
E[X] = 0 \times 0.457 + 1 \times 0.409 + 2 \times 0.122 + 3 \times 0.012 = 0.69
\]
On peut donc estimer qu'il y aura en moyenne $0.69$ personnes qui feront sonner le portique sur les 3 personnes.
\end{enumerate}
\end{solution}
\begin{exercise}[subtitle={Équation puissance}]
Résoudre les équations et inéquations suivantes
\begin{multicols}{2}
\begin{enumerate}
\item $10^x = 15$
\item $7^x = 38$
\item $0.66^x \leq 14$
\item $9 \times 0.36^x = 21$
\end{enumerate}
\end{multicols}
\end{exercise}
\begin{solution}
Les solutions ci-dessous ne sont pas justifiée car l'ordinateur ne sait pas faire. Par contre, vous vous devez savoir justifier vos réponses!
\begin{enumerate}
\item $x = \log(15)$
\item $x = \frac{\log(38)}{\log(7)}$
\item Il faut faire attention quand on divise par un log car ce dernier peut être négatif ce qui est le cas ici. Il faut donc pense à changer le sens de l'inégalité.
$x \geq \frac{\log(14)}{\log(0.66)}$
\item Il faut penser à faire la division à par $9$ avant d'utiliser le log car sinon, on ne peut pas utiliser la formule $\log(a^n) = n\times \log(a)$.
$x = \frac{\log(2.33)}{\log(0.36)}$
\end{enumerate}
\end{solution}
\begin{exercise}[subtitle={Étude de fonctions}]
Soit $f(x) = - 5x^3 + 375x^2 - 4515x + 9$ une fonction définie sur $\R$.
\begin{enumerate}
\item Calculer $f'(x)$ la dérivée de $f(x)$.
\item Calculer $f'(43)$ et $f'(7)$.
\item En déduire une forme factorisée de $f'(x)$.
\item Étudier le signe de $f'(x)$ et en déduire les variations de $f(x)$.
\item Est-ce que la fonction $f(x)$ admet un maximum ou un minimum? Si oui, calculer sa valeur.
\end{enumerate}
\end{exercise}
\begin{solution}
\begin{enumerate}
\item Dérivée de $f(x)$: $f'(x) = - 15x^2 + 750x - 4515$
\item
\begin{align*}
f'(43) &= - 15 \times 43^{2} + 750 \times 43 - 4515\\&= - 15 \times 1849 + 32250 - 4515\\&= - 27735 + 27735\\&= 0
\end{align*}
\begin{align*}
f'(7) &= - 15 \times 7^{2} + 750 \times 7 - 4515\\&= - 15 \times 49 + 5250 - 4515\\&= - 735 + 735\\&= 0
\end{align*}
Donc $x = 43$ et $x=7$ sont des racines de $f'(x) = - 15x^2 + 750x - 4515$.
\item On en déduit la forme factorisée suivante
\[
f'(x) = -15 (x - 43)(x-7)
\]
\item Pas de correction disponible
\item À causes des branches extérieurs, la fonction $f(x)$ n'a pas de maximum ou de minimum.
\end{enumerate}
\end{solution}
%\printsolutionstype{exercise}
\end{document}
%%% Local Variables:
%%% mode: latex
%%% TeX-master: "master"
%%% End:

Binary file not shown.

View File

@ -0,0 +1,226 @@
\documentclass[a5paper,10pt]{article}
\usepackage{myXsim}
\usepackage{tasks}
% Title Page
\title{DM2 \hfill \Var{Nom}}
\tribe{TST}
\date{\hfillÀ render pour le Mercredi 24 février}
\xsimsetup{
solution/print = false
}
\begin{document}
\maketitle
\begin{exercise}[subtitle={Loi binomiale}]
%- set p = round(random(), 2)
Trois personnes s'apprêtent à passer le portique de sécurité. On suppose que pour chaque personne la probabilité que le portique sonne est égale à $\Var{p}$.
Soit $X$ la variable aléatoire donnant le nombre de personnes faisant sonner le portique, parmi les 3 personnes de ce groupe.
\begin{enumerate}
\item Tracer l'arbre représentant le situation.
\item Justifier que $X$ suit une loi binomiale dont on précisera les paramètres.
\item Quelle est la probabilité qu'une seule personne fasse sonner le portique?
\item Calculer puis interpréter les probabilités suivantes
\[
P(X = 0) \qquad \qquad P(X \geq 2)
\]
\item Calculer l'espérance de $X$ et interpréter le résultat.
\end{enumerate}
\end{exercise}
\begin{solution}
%- set q = round(1-p, 2)
\begin{enumerate}
\item
\begin{tikzpicture}[sloped]
\node {.}
child {node {$0$}
child {node {$0$}
child {node {$0$}
edge from parent
node[above] {\Var{q}}
}
child {node {$1$}
edge from parent
node[above] {\Var{p}}
}
edge from parent
node[above] {\Var{q}}
}
child[missing] {}
child {node {$1$}
child {node {$0$}
edge from parent
node[above] {\Var{q}}
}
child {node {$1$}
edge from parent
node[above] {\Var{p}}
}
edge from parent
node[above] {\Var{q}}
}
edge from parent
node[above] {\Var{q}}
}
child[missing] {}
child[missing] {}
child[missing] {}
child { node {$1$}
child {node {$0$}
child {node {$0$}
edge from parent
node[above] {\Var{q}}
}
child {node {$1$}
edge from parent
node[above] {\Var{p}}
}
edge from parent
node[above] {\Var{q}}
}
child[missing] {}
child {node {$1$}
child {node {$0$}
edge from parent
node[above] {\Var{q}}
}
child {node {$1$}
edge from parent
node[above] {\Var{p}}
}
edge from parent
node[above] {\Var{q}}
}
edge from parent
node[above] {\Var{p}}
} ;
\end{tikzpicture}
\item Chaque personne a 2 possibilités (1: fait sonner ou 2: ne fait pas sonner) et l'on fait passer 3 personnes ce qui correspond à une répétition identique et aléatoire. On peut donc modéliser la situation par une loi binomiale.
\[
X \sim \mathcal{B}(3; \Var{0.76})
\]
%- set p0 = round(q**3, 3)
%- set p1 = round(3*p*q**2, 3)
%- set p2 = round(3*p**2*q, 3)
%- set p3 = round(p**3, 3)
\item Probabilité qu'une seule personne fasse sonner le portique. On voit qu'il y a 3 branches qui correspondent à cette situation dont
\[
P(X = 1) = 3 \times \Var{p}^1 \times \Var{q}^2 \approx \Var{p1}
\]
\item
\[
P(X = 0) = \Var{q}^3 \approx \Var{p0}
\]
\[
P(X \geq 2) = P(X = 2) + P(X = 3) = 3 \times \Var{p}^2 \times \Var{q}^1 + \Var{p}^3 \approx \Var{round(p2 + p3, 3)}
\]
\item Il faut d'abord tracer le tableau résumant la loi de probabilité:
\begin{center}
\begin{tabular}{|c|*{4}{c|}}
\hline
Valeur & 0 & 1 & 2 & 3 \\
\hline
Probabilité & $\Var{p0}$ & $\Var{p1}$ & $\Var{p2}$ &$\Var{p3}$ \\
\hline
\end{tabular}
\end{center}
On peut alors calculer l'espérance
%- set E = round(3*p, 3)
\[
E[X] = 0 \times \Var{p0} + 1 \times \Var{p1} + 2 \times \Var{p2} + 3 \times \Var{p3} = \Var{E}
\]
On peut donc estimer qu'il y aura en moyenne $\Var{E}$ personnes qui feront sonner le portique sur les 3 personnes.
\end{enumerate}
\end{solution}
\begin{exercise}[subtitle={Équation puissance}]
Résoudre les équations et inéquations suivantes
\begin{multicols}{2}
\begin{enumerate}
%- set a1 = randint(1, 50)
\item $10^x = \Var{a1}$
%- set a2 = randint(1, 50)
%- set b2 = randint(2, 20)
\item $\Var{b2}^x = \Var{a2}$
%- set a3 = randint(2, 50)
%- set b3 = round(random(), 2)
\item $\Var{b3}^x \leq \Var{a3}$
%- set a4 = randint(2, 50)
%- set b4 = round(random(), 2)
%- set c4 = randint(2, 10)
\item $\Var{c4} \times \Var{b4}^x = \Var{a4}$
\end{enumerate}
\end{multicols}
\end{exercise}
\begin{solution}
Les solutions ci-dessous ne sont pas justifiée car l'ordinateur ne sait pas faire. Par contre, vous vous devez savoir justifier vos réponses!
\begin{enumerate}
\item $x = \log(\Var{a1})$
\item $x = \frac{\log(\Var{a2})}{\log(\Var{b2})}$
\item Il faut faire attention quand on divise par un log car ce dernier peut être négatif ce qui est le cas ici. Il faut donc pense à changer le sens de l'inégalité.
$x \geq \frac{\log(\Var{a3})}{\log(\Var{b3})}$
\item Il faut penser à faire la division à par $\Var{c4}$ avant d'utiliser le log car sinon, on ne peut pas utiliser la formule $\log(a^n) = n\times \log(a)$.
$x = \frac{\log(\Var{round(a4/c4, 2)})}{\log(\Var{b4})}$
\end{enumerate}
\end{solution}
\begin{exercise}[subtitle={Étude de fonctions}]
%- set a_ = randint(-10, 10)
%- set a = a_*3
%- set x1 = randint(0, 50)
%- set x2 = randint(-20, 20)
%- set b = randint(-50, 50)
%- set f = Polynom.from_coefficients([b, a*x1*x2, -a*(x1+x2)/2, a_])
Soit $f(x) = \Var{f}$ une fonction définie sur $\R$.
\begin{enumerate}
\item Calculer $f'(x)$ la dérivée de $f(x)$.
\item Calculer $f'(\Var{x1})$ et $f'(\Var{x2})$.
\item En déduire une forme factorisée de $f'(x)$.
\item Étudier le signe de $f'(x)$ et en déduire les variations de $f(x)$.
\item Est-ce que la fonction $f(x)$ admet un maximum ou un minimum? Si oui, calculer sa valeur.
\end{enumerate}
\end{exercise}
\begin{solution}
\begin{enumerate}
%- set fp = f.differentiate()
\item Dérivée de $f(x)$: $f'(x) = \Var{fp}$
\item
\begin{align*}
f'(\Var{x1}) &= \Var{fp(x1).explain() |join('\\\\&= ')}
\end{align*}
\begin{align*}
f'(\Var{x2}) &= \Var{fp(x2).explain() |join('\\\\&= ')}
\end{align*}
Donc $x = \Var{x1}$ et $x=\Var{x2}$ sont des racines de $f'(x) = \Var{fp}$.
\item On en déduit la forme factorisée suivante
\[
f'(x) = \Var{a} (x - \Var{x1})(x-\Var{x2})
\]
\item Pas de correction disponible
\item À causes des branches extérieurs, la fonction $f(x)$ n'a pas de maximum ou de minimum.
\end{enumerate}
\end{solution}
%\printsolutionstype{exercise}
\end{document}
%%% Local Variables:
%%% mode: latex
%%% TeX-master: "master"
%%% End: